Download as pdf or txt
Download as pdf or txt
You are on page 1of 195

TELEGRAM CHANNELS (Earth,

Faith, Jointly, smle B)


Answers highlighted in YELLOW
Answers in doubt highlighted in GREEN
Notes (if needed) in RED
ICQ = Incomplete Question

1. A multipara woman presented with midline abdominal bulge that becomes more prominent
when she leans forward, cough test is negative. What is the most likely diagnosis?
⁃ Incisional hernia
⁃ Transverse muscle weakness
⁃ Rectus muscle divarication

2. Pregnant lady with vaginal warts asking about treatment


A. Cryotherapy
B. Electrocautery
C. intralesional interferon something

3. A 25 years old male presented with progressive SOB and wheezing. On auscultation:
wheezy chest, no crackles.
History of previous hospitalization and intubation last year.
Was given systemic steroids but no benefit which of the following is the
most appropriate rescue (something like that) drug
A. IV SABA
B. Inhaled budesonide
C. IV Epinephrine
D. IV MgSo

4. Which of the following corresponds to a 4 years old child ?

1-write his first name


2- copies square and triangle
3- says complete sentences
4- plays parallel or plays together

5. Female patient (forget her age )with HTN with incidental finding of Gallbladder polyp
measuring 0.8 mm Asymptomatic & no stones. What will you do ?
A- referral for surgery
B- US follow up after 6 months

6. Male doctor wants to examine the female pt, so he brings a female nurse with him. What
ethical principle is this?
A- Justice
B- privacy
C- i don’t remember
D- nonmaleficence

7. 6 Years old child with nausea, vomiting, headache & visual disturbance worsening over
the past 6 months. Symptoms were provoked at early morning & late night, and improved
by walking. What is the diagnostic test?
A- Brain MRI.
B- Abdominal US.
C- Muscle biopsy.
D- CT (i forgot where).

8. MRI picture of fibroid, they mentioned the diagnosis as (subserosal fibroid), asking how
are you going to manage?
A- Myomectomy.
B- Hysterectomy.
C- COPs.

9. 32 y/o Lt breast mass with blood discharge, what is initial diagnostic test:
- mammogram
- US
- MRI
According to uptodate:
Pathological discharge
>=40 we do mammogram + ultrasound
30-39 we do mammogram +- ultrasound
Less than 30 Ultrasound +- Mammogram

10. Old patient fell from 2 stories, patient is confused open eye to speech and localise pain,
what is degree of brain injury:
-minimal
-mild
-moderate
- severe

Confused 4
Open to speech 3
Localise pain is 5
5+3+4 = 12

Mild 13-15
Moderate 9-12
Severe 3-8

11. Most common side effect of Dtap vaccine?


- fever 38
- erythema around the shot area
I don’t remember
12. Patient came with symptoms of TB
And isolated with open TB in next day found also has HIV what your actions
1 treat both
2 Consider is pneumocystis carinii
3 treat HIV until cd 4 improved
4 treat TB alone to prevent inflammatory reconstruction syndrome

‫بالتجميعات كان فيه خيار و االغلب كان مجاوبه انه الجواب الصح و ما كان موجود باالختبار‬
‫ كان‬treat TB then HIV after 3 month

13. pt w morning arthritis (specific features of rheumatoid arth), but with Malar rash, most
diagnostic test?
Anti smith
anti ds dna
anti ccb
Anti-rnp

14. Definitive diagnosis of Diffuse esophageal spasm?


Barium
manometry

15. Dilated cardiomyopathy taking many drug, what to add? ICQ


Amlodipine
Spiro

Not enough info but it is similar to HF so I will choose spironolactone

16. Pt did splenectomy or any spleen surgery, then after 1 week i think, had left upper
quadrant pain, lungs exam on lower left area: dullness, decreased or absent lung sound. I
think fever, forget if theres cough or not, dx?
Subphrenic abscess
lower left lobe pneumonia

17. Male stabbed on chest, Then developed Subcutaneous emphysema, tracheal deviation,
decreased breath sound on left side, dulness on one side, dx?
Tension pneumothorax
Massive hemothorax

18. A woman had MVA, tender abdomen & I think ecchymosis, but there's no peritonitis signs,
her BP is fine. What's the next step?
A. IV crystalloid
B. FAST
C. Surgical exploration

19. Pain legs, Absent foot pulse, ABI<0.3 . + on imaging: embolus obstructing artery, (i think
acute limb ischemia) i think he was already on enoxaparin
A. Heparin or thrombolytic
B. ThromboEmbolectomy
C. Thrombolysis

20. Child pale + eats dirt (maybe pica) + hepatomegaly + labs shows small size RBC + What
will u order?
Lead screening
CBC and blood smear

21. Female in the first trimester pregnant had a history of DVT (didn’t mention when). and he
asked if she needed any medication right now?
A) Enoxaparin
B) Heparin
C) Warfarin
D) no need anticoagulant

22. Female pregnant patient (didn’t mention what trimester) came with leg swelling, and after
few days she had shortness of breath. Labs was done (I don’t remember exactly sorry).
Which of the following is the initial test that should be done?

A) D-dimer
B) CT angio
C) Ultrasound doppler
D) I don’t remember
No any V/Q scan in the choices , if its there its the answer.

23. Adult patient forgot age presented complaining of heartburn that he tried to treat using
antacids from the pharmacy but nothing worked Which of the following is the next best
treatment?

Ppi
Life style modification
Fundoplication
H2 blocker

24. 31 yrd old 10 months history of diarrhea with anemia and unintentional wt loss no change
appetite no blood in stool What to give ?
A-anti diarrhea
B-anti biotic
C-anti parasite
D-gluten free diet

25. Which of the following is the highest risk factor for cerebral palsy
• A-Hypoxia
• B-Preterm
• C-Gestational diabetes
• D-Neonatal sepsis
26. Patient with psychiatric symptoms and you decide to start neuroleptic medication. What
is the side effect that can develop?
A-Hyperthermia
B-Myocardial infarction
C-Seizure Akathisia

27. 42 years old female complaining of amenorrhea, night sweat and flushing for the last 6
months. What is the most likely diagnosis? ICQ
A- Hypothyroid
B- Hyperprolactinemia
C- Congenital adrenal Hyperplasia
D- Pheochromocytoma

28. There was a question about 20 yrs female c/o intermittent crampy abdominal pain and
alerted bowel habits pain relive by defection which can be constipations and other times
loose and watery content, and after defecation still feel not empty her gut, what is the
appropriate management

A/ h.pylori test
B/ abd ct
I don’t recall other options

Case is most likely IBS , i would choose fecal calprotectin if its there.

29. Infant diagnosed Kawasaki syndrome Which treatment


A. Aspirine
B. Cyclosporine
C. Prednisolone
D. Immunoglobulin

30. 5 month old infant with coughing, sneezing and circumoral cyanosis for 3 days. Recurrent
frothy sputum around the mouth. On examination there are diffuse rhonchi all over the
chest and basal crackles but good air entry bilaterally. Chest x-ray shows bilateral
infiltrates. What is the most likely diagnosis?
1- Heart failure
2- Pneumonia
3- Bronchiolitis
4- Bronchiolitis Obliterans

31. 47 years old man can't swallow liquids what is the high significant diagnostic?

A monomety
B barium swallow
C gastrointestinal endoscopy
D ct with contrast

32. side hot nodular goiter 2*3 management with exophthalmos (eye protrusion)
A near total thyroidectomy
B right hemithyroidectomy
C radioactive ablation

33. Long question about patient having pneumonia and when you calculate his CURP-65 it’s
zero how would you treat him?

A- inpatient with azithromycin


B- inpatient with azithromycin and cefuroxime
C- outpatient with azithromycin
D- outpatient with azithromycin and ceftriaxone

If with comorbidities D , if not C

34. Pediatric with burnt sugar smell


Ans: Maple syrup urine disease

35. Pt post mi developed R HF he is on ACEI what should give him ?


A-Bb
B-Ccb
C-Arb
D-Heparin

36. A case of adrenal carcinoma is planned for surgery. What to do before the surgery?

A. PET Scan
B. 24-hour urine metanephrines
C. Serum Cortisol level
D. Urinary Cortisol level

37. A toddler presents with cough and wheeze. When he cries, it looks like he is having
breath-holding spells. What is the most likely diagnosis?

A. Asthma
B. Pneumonia
C. Croup
D. Bronchiolitis

38. A lady is pregnant with twins. What is the most common risk factor associated with
increased mortality in multiple gestation?

A. Birth defects
B. Birth trauma
C. Prematurity
D. Placental insufficiency

39. A 39 years old male patient present to ER with productive cough and fever.
Tempreture 39 O2 96 What is the appropriate management?
A-Moxifloxacin IV (dose written)
B-one of 4th generation cephalosporin IV (dose written)
C-meropen IV (dose written)
D-Pepracillin tazobactam 4.5 g every 6 hours

40. The most common type of hepatitis in KSA


A
B
C
D

41. 42 YO mother presented with paraumbilical mass, On exam you found the mass tender,
irreducible, and negative cough impulse. What is the appropriate next step ?
- Diagnostic Laparoscopy
- Abdominal Us
- Open repair
- Biopsy

42. Alderly pt come with severe headache to ER then suddenly deteriorated and decrease
level of consciousness ct done was hematoma next
A_admit to icu
B_give mannitol
C_evacuate in ER
Most likey this is epidural hematoma with loss of consciousness (lucid interval) so can be treated
with burr hole in ER but if craniotomy is present then it is the answer

43. 28 years Gestational 20 week her blood group negative and she come with vaginal
spotting what you will do
A observation
B give anti d now

44. 16 years old boy come to ER with history of euphoria aggitation visual hallucination What
he take
A_ cannabis
B_amphetamine

45. Female come complain of abdominal pain with watery and green vaginal discharge she
recently entered iud diagnosis
A_PID
B_uterine rupture
C_bacterial vaginosis

46. Most common delivery position in which 4th , 3rd tear happens ?

47. Old man with oxygen 74% what to do ?


A. mechanical ventilation
B. oxygen mask
C. send home

49. Pediatric case female has sore throat Pain upon swallowing and neck
lymphadenopathy since 2days Now complaining of fever
When examine her throat swollen tonsils whit exudate Whats the important next step to
establish the diagnosis :
ECHO
Biopsy the lymph node
Throat culture
Blood test looking for agent ( not sao titer)

Throat culture or Rapid antigen test

50. Women K/C of liver cirrhosis, + acites , confused labs: low Na , high glucose
A) Fursomide
B) 0.5 NS
C) NS
D) D5

I had the same question on my exam and they mentioned Elevated jvp, if it comes on the exam
then the answer is Furosemide.

51. 32 GA came with preterm labor then had preterm rupture of membrane and was given
Abx and steroids. What to give
Nifedipine
Mg sulfate

52. Pediatric pt with abdominal pain radiating to the back increased with eating.
Tenderness on the epigastric area. Amylase and lipase high
Next step?
NPO, IVF, analgesia
MRCP

‎‫ السؤال حق‬prognostic ‫ كان فيه‬hepatological case ‫ وفيه‬LFTs ‫ بعدين يسأل‬which of these has the
highest prognostic value،

Best prognostic value?


Bilirubin
Albumin
ALT
INR

Child present with severe dehydration and fever ( sunken eyes and decreased skin turgor
) septic work up ordered, what to do next ?
A- 20ml/kg IVF bolus
B- Maintenance IVF until results come back

Pt with severe epigastric pain and vomiting blood , organomegaly , finding of gastric
varicocele on endoscopy, portal vein normal splenic or liver congestion I can’t remember
He doesn’t use med or drink alcohol and he has normal liver
Splenic artery Aneurysm
Pancreatitis
Marry tear
Esophageal varicele

Bilateral distal and proxsimal pharangeal joints pain with nodule no morning
c reactive normal
Rmatoid normal ?
1-polyarticular gout
2-Reactive arthritis
3-S... rematoid arthritis
4-Osteoarthritis

GERD What’s the most associated risk (something like this)


Smoking
Barrett
Obesity
Smth unrelated

If they mention in the case patient is a smoker only, I’ll go with smoking
If they mention smoking and GERD then I’ll choose Barrett
If the question doesn’t have a case and it’s “most IMPORTANT” then Barrett
If the question doesn’t have a case and mentions “most COMMON” then I’ll go with smoking


post 7 cycle chemotherapy fever with high neutrophils 70% next
A- give him NSIAD and antibiotic
B- wait for cuture sensitivity
C- give empiric Abx

After herniotomy surgery for 5 years boy came with fever and pus discharge and part of
mesh seen?
A. Give IV antibiotics
B. Draining of pus
C. Draining of pus and remove mesh
D. Observation
Biliary stone and 12mm cbd wall , what to do?
Ercp
Mrcp
Lap choly
Open choly

Temporal fracture caused what? >> epidural hematoma

Patient MVA with pelvic fracture managed with pelvic binder, what Fluid you’ll manage her
with:
⁃ albumin
⁃ RL
⁃ Hypertonic saline
⁃ Dextrose

Pt I think he known case of Alzheimer came to ER with severe agitation the doctor give
him a big dose of Haloperidol and he develops side effects I don’t remember what r they
exactly, what u should give him now?
They mean what is the antidote of haloperidol?
A-Naloxone
B-bromocriptine
C-Glycogen

Old after blood transfusion has leukocytes without fever,dx?! ICQ


Hemolytic disorder
Non hemolytic
Bacterial infection
45 year old female 6m missed period
She is not pregnant she want to know what is the most valuable and significant lab for
postpartum :
LH
Cortisol
Calcitriol
Testosterone

Teenager with irregular rhythm otherwise completely normal what to do


1) normal growth
2) dysfunctional myocardium

Full MCQs /
14 years old boy came for routine clinic. Does not have any major medical issues. On
examination you find irregular pulse (ECG picture provided with irregular rhythm but
normal P wave)
Vital signs all normal including HR
Height >95th percentile
Weight 25th percentile

Mother is asking you about the prognosis?


A- Multiple syncopal episodes
B- Normal development
C- Will need pacemaker
D- myocardial dysfunction
Pt had prosthetic valve disease for 10 days. Underwent dental procedure and developed
infective endocarditis What organism caused infective endocarditis?

- Staph aureus
- Staph epidermidis
- Strep viridans

Pt with high cholesterol, triglycerides and TSH. T4 was normal


What is the best medication for her Dyslipidemia?
- Statin
- Thyroxin
- Niacin

Pt have Rt testes swelling for 6 months , in the last 4 months it increase in size , in
examination , it was separable from the testes and doesn’t reach the inguinal. ICQ
A - spermatocele
B- epididymal cyst
C - indirect hernia
Spermatocele is the same as Epididymal cyst.

An elderly man on NSAIDs developed dyspepsia. Endoscopy showed gastritis. Labs


showed iron deficiency anemia with Hb= 9. What is the best method of treatment?
A- Iv iron
B- Blood transfusion
C- Oral iron
D- Erythropoietin

Male patient diabetic and smoker presented with ulcer on 2nd , 3rd and 4th left foot toes.
He gave a history of superficial thrombophlebitis. Upon examination: he has absent
dorsalis pedis and posterior tibialis pulse in both limbs , while popliteal pulse was intact.
How would you manage the patient?
A- Amputation of toes
B- Longterm anticoagulant
C- immediate Surgical intervention
D- Ask patient to stop smoking with lifestyle modifications

25 YO pregnant lady at 30 weeks presenting contractions no bleeding no pooling. CTG


shows assuring status and good contractions.
How to diagnose preterm labor:
1. Digital exam to see cervical dilatation
2. From the history
3. CTG showing assuring status
4. If there is no leaking then it is not preterm labor

The lady was G2 P1, her labor was 2 hours long (baby came out before she barely made it
to the hospital). Baby’s weight was 3000. She started bleeding after delivery of the
placenta.
Question asking about the cause of PPH.
1 Baby’s weight
2 Prolonged labor
3 Precipitous labor
4 Large multigravida

Kid with cervical lymphadenopathy and sore throat (now). Has Coca Cola red urine.
1 IGA Nephropathy
2 Acute glomerulonephritis
3 Acute cystitis

Woman came with symptoms of pyelonephritis. Fever, pain, vomiting chills. High fever.
She has history of multiple UTIs.
A. Septic screen
B. Urinary tract US

open fracture He is already on antibiotics


A.Surgical debridement with intramedullary nail
B. Closed reduction with external fexation
C antibiotics

12 YO patient with recurrent epigastric pain and vomiting and his mother the same.
Stool showed blood and mucus.
Best for diagnosis:
A. Endoscopy
B. CT
C. Amylase and lipase
D. US (not sure)

Postpartum by one week came with shortness of breath and displaced apex beat.
A. Postpartum cardiomyopathy
B. Pulmonary embolism
Should also have HF signs and symptoms

Copd Patient came with raised JVP and bilateral edema and hepatic pain. What is the next step in
management you do for him ?!
A) spirometry
B) ECHO

DVT then became PE. What is the high diagnostic test ?!


A) spiral CT
B) ECHO

Patient with mid systolic murmur at RIGHT sternal border with S4 but no symptoms of heart
failure or displaced apex of heart asking about diagnosis ?!
A) aortic stenosis
B) aortic regurgitation
C) mitral regurgitation
20 years old lady, recently delivered, presents with symptoms of epigastric pain which radiates to
the back and nausea and vomiting(not sure about vomiting), on examination there’s 8x8cm mass
in the epigastric area. What is the most likely diagnosis?
A-Gallbladder polyp
B-Pancreatic pseudocysts
C-Gastrointestinal stromal tumor
Not sure missing lot of info but would choose it because it Gallbladder Stones and its complications can
happen in pregnancy

Male patient recieve 4 liter blood transfusion, what is the most important complications?
1) hypokalemia
2) hypocalcemia
3) citrate toxicity
4) hyper albunemia

Pt recently diagnosed with congestive heart failure, what is most important intial management?
1) bb and diuretics
2) ace and diuretic
3) nitrate and bb

13 weeks of gestation + during pelvic examination found cervical lesion


Next step?
1) pap
2) colposcopy
3) cone biopsy
4) endocervical curettage

Child has diarrhea without vomiting ( no other sx), his mother concerned about dehydration What
is the appropriate intervention?
1) oral rehydration
2) anti diarrhea
3) iv saline infusion
4)Change milk

Mother recently delivered her baby, the doctor encouraged her for breast feeding
What is the most significant response?
1) decrease centrocranial infection
2) reduce primary immunity disorder
3) increase bonding between mother and baby
4) reduce the hemorrhagic disease for newborns

Female with postpartum depression for 5 days, the baby is breastfed well, but she is sad and cry,
what is appropriate intial step?

1) discharge follow up after 1 week


2) SSRI
3) benzodiazepines
4) multivitamin

36w fundal height 38 active labor pain tender tense uterus what to do next:
Check ctg
Us

the patient was stabbed in multiple areas, came to the ER and was awake,
his clothes were full of blood, his bp is around 80/60, which of the following is
best next step?
A) oxygen supplementation.
B) iv crystalloid infusion
C) Intubation

milestone fear of strangers?


6 months

-atelectasis and pneumonia dx?


Rigid bronchoscope
Flexible
Fluoroscopy

It should be CXR

71 y/o patient post subtotal colectomy, after days his urine output becomes 0.5mL/hr. What is the
most appropriate drug to be given intravenously?
A- Antibiotics
B- Diuretics
C- 500 ml NS challenge test
D-...

Patient had gastric bypass then weeks later develop abdominal pain rebound tenderness
abdominal X ray showed small bowel multiple air fluid levels
What is the next step
-Exploratory
-Conservative with ABX

Patient has obstruction with Rebound tenderness could mean perforation and peritonitis.

11 years old with exercise intolerance


Echo normal
Vital shows tachycardia tachypnea and 95% sat
What is most appropriate to order ?
-TFT
-CBC
1- minimal days per week for exercising muscles?
A-3
B-4
C-5

If aerobic or didnt specify then 5days


If said strengh muscle excercies It should be 2

2 - what expected pt with dm2 in urine investigation?


A- hematuria
B- protein urea

4- long case but in the x ray showed silhouette signs (i think ask about the cause)
A-pericardial
B-myocardium

Q- 34 years old woman presented to the hospital with dyspnea and cough 10 days after her
discharge from hospital, she was diagnosed as pneumonia. Culture shows MRSA, What
antibiotics are most appropriate? ICQ
- Pipra/tazo.
- Ceftriaxone + Azithromycin.
* no Vancomycin in options
I would choose A if no option like linezolid,daptomycin or clindamycin is available as it appears to be
hospital aquired infection..

Celiac disease pt noncompliant to gluten free diet, has


Iron deficiency anemia, took supplements for 3 months, still very anemic

A- IV iron
B- Continue same therapy (oral supplements)
C- Dissolve pills in water
D- Crush pills and take them orally?

Female in 30s or 40s has excised fibroadenoma, histopathology showed (forgot type of cells) with
hyperplasia and atypia, which factor suggests malignancy?
A- Age
B- presence of Atypia
C- presence of hyperplasia
D- can’t recall

Ptn did a lumbar spinal surgery through back approach, in the 4th post op day he was sent to the
GS due to severely distended abdomen with RIF tenderness, barium enema was done and showed
hugely all bowel distention (with cecal diameter of 13 cm) but there was no extravasation or gas in
the peritoneum. Which of the following most appropriate management ?
- conservative
- colonoscopy
- cecactomy
- total colectomy
This is a case of oglive syndrome which is treatem consevative until diameter reaches more than
12 then decompressive colonoscopy or Neostigmine is the answer. If colonoscopy fails then
cecostomy is the answer

Pt with liver stigmata and hematemesis . Bp 80/60, Hemoglobin (8.5) normal was (130-150) and and
resuscitative begin , what to do after initial resuscitation?
- EGD
- Bl transfusion

Female pregnant with recurrent late deceleration and dilated 6cm what to do ?
- Emergency CS
- Reassure
- Induction

30 years old female with history of 3 years infertility, semine analysis normal, history of chlamydia
traetment for past pelvic inflammatory disease , what to order :

LH/FSH
TSH
US PELVIC
hysterosalpingogram

Sle pt with neurological symptoms asking about first line in the management?

-steroid with cyclophosphamide


- iv cyclophosphamide

Pt trauma head and paralysis found have Hge stroke , what is best treatment:
- Carniotomy and decompresion

Old with retrosternal pain radiates to the back and then sudden become dyspnea and confused ,
bp low :

CTA
TEE
US
Laparotomy

Most likely aortic dissection. Is unstable -> TEE , if stable -> CTA

a 45 y old male had an accident by a wood to his right leg. A portion of the wood infiltrated the
skin causing a greyish white semi purulent discharge to ooze out.
Asking about the treatment.

1- CBC.
2- Take culture and sensitivity ( most likely because the WBC were high and his presentation is
suggestive of an infection ).
3- unrelated.
4- unrelated.

Most likely its necrotizing fascitis and needs surgical debridment

Management of comminuted tibial condylar fracture.

23 years old female, 28 gestational age with tricuspid regurgitation. She has slight physical
limitations. What is her New York Heart Association score?
A. I
B. II
C. III
D. IV

Patient presented to the ER with a history of 3 day fever and cough and rash all over the body.
There is consolidation and crepitation.
What is the treatment?
⁃ Antibiotic
⁃ Antipyretic ⁃ Acyclovir

Easy Photo and case of stroke MCA which occur after sleep before 2 hours. What is the treatment
? ICQ
⁃ tPA
⁃ Clopidogrel
⁃ Aspirin
The question is not written clearly, however, if they said that the symptoms accord after 2 hours of
sleeping, then she is within window and Thrombolysis is indicated

I‫‏‬nfantile colic. What is the recommended measure in this case ? ( no initial - no best)
‫ ⁃‏‬Change formula
‫ ⁃‏‬Behavioral therapy
‫ (‏‬there is no improve feeding technique in choices )

‫‏‬Female Patient with right iliac pain , radiate to shoulder, what’s the highest diagnostic test ?

‫ ‏‬- pregnancy test


A
‫‏‬B-CT
*‫ ‏‬female with right cervical lump Upon examination she has enlarged cervical lymph node with
normal thyroid gland Percutaneous biopsy taken from the lump Histopathology report: follicular
thyroid cells, which of the following is the most likely name of cervical lump?
‫‏‬A-metastasis of thyroid gland.
‫‏‬B-Follicular thyroid carcinoma.
‫‏‬C-Papillary thyroid carcinoma.
‫‏‬D-aberrant thyroid tissue.

40-‫ ‏‬yo girl presented complaining of SOB and exercise intolerance and 11
‫!‏‬weight. on examination there is gallops rhythm
‫‏‬BP: high 150systolic
‫‏‬Hr: 160
‫‏‬RR:40
‫‏‬whats the best next management ?
‫‏‬A-TSH
‫‏‬B- cBC
‫‏‬C- Electrolytes
Depends on age and weight if weight is low for age I would choose A but if normal cbc is the most likely
answer because gallop rhythm happens with both anemia and hyperthyroidism

4‫‏‬year old can stand on 1 foot for less than 10 sec and can tell story but not ise future tense and
draw square but can’t put cubes together…
‫‏‬-Normal development
‫‏‬-Only language delay
‫‏‬-Global development

‫ ‏‬irst indicator for hemorrhagic shock :


F
pulse pressure

‫ ‏‬/ 55-year-old k/c of bronchial asthma , DM , HTN came to the ER with an acute episode of
Q
palpitation . HR : 160 bpm with irregular rhythm , BP 120/80 SPO2 92% on RA
‫‏‬what of the following is most appropriate next step :

‫ ) ‏‬Cardioversion
A
‫‏‬B) Amiodarone
‫‏‬C) Adenosin
‫‏‬D) Propranolol

‫ ‏‬TA with dull percussion on exam and chest tube was inserted. Which of the following is an
R
indication for surgical intervention?
‫‏‬A. initial output > 500 ml
‫‏‬B. Ongoing output more than 200 ml/hr
‫‏‬C. combined hemothorax with pneumothorax
D. drop in Hemoglobin and hematocrit

a 28 years old patient came with severe peri-anal pain and swelling. On examination, there is 1x1
cm perianal swelling with tenderness.
Vitals: normal, no fever.
Labs: WBC 8 (normal).
Which of the following is the most likely diagnosis ?
A. Anal fistula
B. Anal fissure
C. Perianal abscess
D. Perianal hematoma.

Pt with trauma came with unable to move legs and arms, conscious oriented with shallow
breathing chest exam Normally, having low BP, what best to secure airway ?
Tracheostomy
oropharyngeal airway
nasopharyngeal airway
endotracheal airway

25 y Patient with scalp lacerations came after 6 h , what to do ??


Leave with granulation healing
debridement and granulation tissue healing
debridement with primary repair

Concern mother asking advice for her baby who has only diarrhea ?
A- Oral replacement fluid
B- I.v saline

-A child with yellowish discoloration


Labs:
Ind. bilirubin: 140
Total: 240
Other labs within normal. What is the Dx?
1- viral hepatitis
2- Gilbert syndrome
3- obstructive jaundice
4- acute pancreatitis

25 yr old female with right iliac fossa pain and suprapubic pain without tenderness us done was
inconclusive
What will do next?
A- CT
B- transvaginal us

Female pt pregnant at 41 weeks


CTG show deceleration (written like this) wt you will do
Delivery
CTG daily
Biophysical daily
Pregnant( ‫ ماني متأكد‬٣٠ ‫ )اعتقد فوق‬came with fever and rigor what to do ?
Iv antibiotics
oral antibiotics

Pt hit on abdomen.. by MVA. No bleeding.On examination raised jvp, hypotension pulse 120.
What type of shock ?
Cardiogenic
Hypovolemic

Patient with nausea, vomiting and diarrhea developed postural hypotension. Fluid deficit is:
a) Intracellular
b) Extracellular
c) Interstitial

Q2 PT k/c of congestive heart failure, came for F/U he was asymptomatic, he is on ACEI , diuretic,
what do you want to add?
A- nothing
B- BB
C- ARB
D-CCB
If EF<50 then b if not a is the answer

Q3 most anti-malaria resistance?


A- chloroquine
B- mefloquine
C- atovaquone
D- malarone

Q2 28y/o female medically free presented to ER with 2 days Hx of fever, lethargy, today she
developed seizure, how will you treat her?
A- vancomycin + ceftriaxone + steroid
B- vancomycin + ceftriaxone
C- steroid
D- tazocin

Q3 78y/o male presented with change level of consciousness and fever , lumber puncture was doe
show CSF gram +ve bacilli , hem catalase +ve , how will you treat ?
A-ceftriaxone + vancomycin + steroid
B- ceftriaxone + ampicillin + vancomycin
C- ampicillin

Q4 CSF analysis of 59y/o male showed lymphocyte 90%, -ve gram stain, how will you treat? ICQ
A- acyclovir
B- ampicillin + ceftriaxone + vancomycin
C- supportive
D- ampicillin + steroid

Missing information about glucose and protein it could be tuberculous meningitis or viral

23 year old female patient, history of ASD when she was 3 Years, now he has
decrescendo diastolic murmur, 2/6, on the left sternum, what is your diagnosis?
A. Mitral stenosis
B. Aortic regurge
C. Tricuspid regurg

COPD case comes with dyspnea cough and bilateral lower limb edema Next most appropriate
investigation?
A- Spirometry
B- CT chest
C- Lower limb duplex ultrasound
D- CXR

If there is ECHO its the answer


Pt with mid diastolic rumble With decrescendo murmur.
What is this?
1. Pe
2. Aortic regurgitation
3. Mitral regurgitation
4. Pulmonary insufficiency
Answer should be mitral stenosis or tricuspid stenosis
Pt came with back pain
They have done an MRI and it showed something ( Spondylitis as far as I remember ) on the T6
vertebra . What to do next =
A-biopsy T6
B-bone marrow aspiration + culture
C-PPD test
D-Brucella titre

elderly male k/c of BPH, asymptomatic, not on medications, came to clinic with multiple readings
of high Bp investigations: normal UA and no signs of ventricular systolic dysfunction on echo,
what is the most appropriate advice?
A- book appointment for Bp determination
B- advice exercise
C- start him on amlodipine CCB
D- start him on BB

elderly k/c of DM and RA presented to ER complaining of lethargy and back pain, lab
investigations: Ca slightly elevated , Cr/albumin ratio was high, what is your diagnosis?
A- membranous glomerulonephritis
B- diabetic nephropathy
C- primary amyloidosis
D- secondary amyloidosis

high TSH, low FT4 in an asymptomatic 30y/o female, what is your diagnosis?
A- hypothyroidism
B- hyperthyroidism
C- subclinical hypothyroidism
D- subclinical hyperthyroidism

PT with lethargy all labs are normal except: hyperkalemia and low bicarbonate, how would you
manage this PT?
A- insulin and dextrose infusion
B- bicarbonate
C- hemodialysis
D- ACEI
24y/o male presented with *diarrhea and fatigue* What electrolyte abnormality would you
suspect?
A- hypokalemia
B- hypocalcemia
C- hyponatremia
D- hypomagnesemia

PT with refractory hyperkalemia, what is your next step?


A- asses aldosterone and renin ratio
B- start spironolactone
C- check magnesium
D- give another potassium replacement

If refractory HYPOkalemia then C

1- 16 weeks Pregnant outbreak illness what vaccine?


A-Rubella
B-Influenza
C-smallpox
D-chicken pox

2- Obese has sleep obstructive apnea


best treatment:
A-mindofil
B-BMI less than 30
C-*continuous positive airway pressure*

3- drugs cause irritation and anxiety and insomnia:


A-Tricyclic antidepressant
B-tetracyclic antidepressant
C-MOI
D-SSRI

4- 15 years worker in a chemical factory wants to screen for cancer, what is the most sensitive
test?
A-cell transformation
B-AMES
C-Unscheduled DNA
D-cytogenetic

7- case of hyperkalemia, what ECG changes?


A-*Peak T wave*
B-Peak P wave
C-Narrow complex
9-patient complain of fever and malaise with headache and on Vaginal examination it shows
multiple bilateral vesicles:
A-Genital warts
B-Genital herpes
C-Charnacle
D-Chlamydia

- patient complains of heavy bleeding and picture shows uterine fibroids asking about the name of
the procedure?
A-Hysteroscopy
2-laparoscopy
3-cystoscope

Bleeding fibroid usually submucosal treated by hysteroscopic myomectomy

post defecation painless fresh bleeding?


A-Anal fissure
B-hemorrhoids
C-Perianal abscess

Smoker what the percent of loss her life ‫او انه ينقص عمره او شيء زي كذا ناسية بالضبط‬
5
10
15
20

Asthmatic child has 1 exacerbation per month that treat well in ER with SABA , what give the child
as daily mx in home ?
- ICS
- IV theophylline
- NO SABA

Pt with asthma symptoms mild improvement after SABA and ICS , symptoms worsen at night and
when he lays down he develops hoarseness of voice , what to give ?
- PPI

Child diagnosed with croup clinically and tx given accordingly but no improvement , next ?
- Lateral neck X Ray
- Chest CT
- bronchoscope

WHO report that the Case-fatality is 37.4% for Mers Cov , what does the percentage mean ?
- Number of population at risk for Mers Cov infection
- Number of Infant who will catch the infection if their mother is +ve
- Number of death among infected people with Mers Cov

Pt present with bilateral green nipple discharge , US done showing duct dilatation ( BIRADS score
II ) , what is next ?
- Follow up
- Breast conserving surgery
- Surgical excision

Pt with pelvic fracture , he is vitally stable , what to do until he fully assessed by neurosurgeon ?
- Pelvic binder
- CT with contrast
- Restrict movement of the spine

55 years Old age case of ischemic colitis , vitals are stable but examination revealed tender and
rigid abdomen , next ?
- Colonoscopy
- Laparotomy

Pregnant came with a routine visit , US showed oligohydramnios , what is associated with it ?
- DM
- Duodenal atresia
- Placental insufficiency

Pregnant presented in 2nd trimester with signs of anemia + had severe vomiting in 1st trimester.
Labs showed mcv 112, hgb 9. Dx:
- physiological anemia
- folate deficiency
- b12 deficiency
- iron deficiency

Pt presented with palpitations, diagnosed now with AFib ( given). Next most appropriate
investigation?
1. Carotid US
2. Thyroid Function test
3. Exercise tolerance test
4. 24 ECG monitoring

Asthma exacerbation not responding to salbutamol, o2 and steroid.


1. Magnesium sulfate
2. Aminophylline
3. Albuterol
4. Not sure maybe intubation?

Pt RTA came with tracheal deviation, Chest X ray expanded lungs, wide mediastinum:
1. Tension pneumothorax
2. Hemothorax
3. Torn thoracic aorta
4. Cardiac tamponade

18 month Child came with shortness of breath.decrease air entry in right side ( forgot the rest
sorry) + they was a finding in chest x ray(forgot) parents tell you there is no prior infections or hx
of fever or choking, past medical hx unremarkable: ICQ

1. Foreign body aspiration


2. Brochiolitis obliterans
3. Asthma
4. Pneumonia

8 y.o patient immunization schedule unknown, came with fever and neuro symptoms (not sure
what they were) examination reveals bilateral tender and enlarged parotid glands and pain with
neck flexion:
1. EBV
2. CMV
3. Mumps
4. Measles

Morbid obesity which is high diagnosis for procedure?


Barium swallow
Endoscopy
Ct
Uss

Old patient with Left lower quadrant pain. Found large collection 11*_ and diverticulosis. What to
do:
- sigmoidoscopy
- laparotomy
- percutaneous drainage
- resection and anastomosis

6 days post abdominal surgery patient had obstruction. X-ray: multiple air fluid levels. Dx:
- paralytic ileus
- volvulus
- adhesion

Lady post cholecystectomy presented with right upper quadrant pain. Found to have collection in
right upper quadrant:
- us guided percutaneous drainage

Patient after rectal surgery presented with sob. Sinus tachycardia in ecg. What to do?
- d-dimer
-ct

Patient with unilateral lower limb non-pitting edema. What will you give:
- aspirin
- heparin
- warfarin

Pt 60 new onset of spotting minimal amount of bleeding from genitalia


What is the source of bleeding?
Tubal
Ovary
Uterus
Lower Genitalia

Pt not child alert with shallow breathing


Tracheostomy
Cricothyroidotomy
Orotracheal intubation
Nasotracheal intubation

Microcytic anemia with target cells and inclusion bodies what’s the Dx?
SCD

Pt with MR ( written severe rheumatic mitral regurge) and sx of HF


on lasix , spironolactone, ACEI, BB
What's appropriate for management?
Digoxin
Mitral valve replacement

Patient developed SOB and chest pain after long flight, Ex:hyperresonance chest with decrease
breathing sound ,medical and surgical Hx clear, he is TALL
VS:hypotensive
What is your Next Mx?
-thoracostomy
-thoracotomy
-I can’t remember the others choices

50 y/o (I think) post surgery (I think it’s gastric sleeve) by 5 days developed obstruction Sx
(vomiting, bloating and abdominal pain) when they were going to push the Pt for CT they noticed
in the NG tube coffee ground vomitus and fresh blood, what will you do?
A- CT abdomen
B- Upper endoscopy
C- Colonoscopy

Pt known to have MR from Rheumatic heart disease came to the ER with dyspnea, orthopnea and
PND, she was given medication (I think ACEi and diuretics) and her Sx improved.
ECHO:
Severe MR, EF=45
What will you do?

A- follow up
B- MV replacement/repair

Lung LN with caseating granuloma ‫ لكن ما اذكر وش الخيارات اللي اذكره ان فيه‬TB
‫ يبغى‬diagnosis

MVA patient with decreased air entry in left hemithorax was tympanic on percussion (+ was
unstable) what’s next:
- chest tube
- needle decompression
- intubation and ventilation
- x-ray

Patient was treated conservatively for appendicitis then presented with appendiceal mass. Next?
ICQ
- appendectomy after 12weeks
- colonoscopy after 6 weeks

Depend on age if old age B , young age A

A patient presented with an infection. Which one of the following diseases the doctor need to
report to the ministry of health?
Chlamydia trachomatis
Bacterial vaginosis
Lobar pneumonia
Infectious mononucleosis

Vaccine pneumonia
‫سوال مرة‬
When to give splenectomy after 2 weeks
‫وسوال مرة وش النوع الي ينعطى‬
After 65 pneumonia
‫قصده‬
pcv 13 ‫االنواع وكذا وكم مرة‬

- ‫ سوال عن طفل عنده جوندس ووو وبالنهايه قال عنده حمى البحر المتوسط ايش االدويه ال‬CI
‫الجواب‬: ABX(amoxicillin)
If the patient has jaundice then its G6PDD dont give aspirin, if no jaundice then its Mediterranean fevere
dont give amoxacillin

11- fibroid felt on pelvic examination. What's the location ?


intramural

10- old female e back pain, relieved e leaning forward on walker and walking uphill, Peripheral
pulses are intact
Spinal stenosis

9- A child presents with a salmon patch macule, hepatosplenomegaly, and multiple


symmetric and asymmetric joint pain. Rheumatoid factor was negative. What is
the most likely diagnosis ?
a.Juvenile psoriatic arthritis
C. Systmeic Juvenile idiopathic arthritis

Prognosis of schizophrenia ?
5% remission
33% reduction of symptoms
70% satisfied with their life

Long case child complain itching his eyes and ..nasal congestion on examination there is
periorbital swelling and enlarged mucous turbinate
A . allergic rhinitis
B. rhinitis medicamentosa

1- Patients with bleeding peptic ulcer and history of long use of aspirin. What is the
management?
-High dose oral PPI BID
-IV PPI for 24hr followed by oral PPI
-IV PPI for 48hr followed by oral PPI
-IV PPI for 72 hours

3- What contraceptive med is used for ovarian cyst


Progesterone pills
OCP
Depo

4- Medial ulcer, what is the risk factor?


DM
Atherosclerosis
Venous HTN
Copd and cor pulmonale
Which of the following Improve mortality ICQ
A-Multaknin
B-Inhaler steroid
C-LABA

Mortality in COPD improved by


1- smoking cessation
2- Oxygen
3- Vaccination

-Pt has fever and didn’t eat for 1 day


BP : 64/42 ‫متأكد‬
A- discharge the pt
B- admission for observation fever and discharge if become afabril
C-advice the mother to come back if happen again
D- admission for Iv antibiotic and culture

-female with suprapubic pain with purulent discharge . Vaginal Ex tenderness in fornix
A-Acute cervicitis
B-Acute salpingitis
C-Acute appendicitis
D-chronic appendicitis

Most important initial investigation for PE?


1- D-dimer
2- VP scan
3- Doppler US

70 Y.O male complain of lower abdominal pain and desire for urination and has a history of
progressive urinary tract obstruction most likely diagnosis
1- prostatic cancer
2- BPH
3- UTI

Lung LN with caseating granuloma otherwise normal,, management;


1- observe?
2- steroid
3- ABx

Caseating granuloma anywhere -> TB


Non-caseating granuloma in lung -> Sarcoidosis
Non-caseating granuloma in ileum -> Crohns
A question about a young girl (7-9 years) her mother brought her because she was worried about
her developing genital hair , but no muscles enlargement, no breast development or axillary hair
and the girl is normal socially and interact with people
On examination there is increase height
Dx
1- Turner syndrome
2- congenital adrenal hyperplasia
3- premature adrenarche

Elderly pt come with severe headache to ER then suddenly deteriorated and decrease level of
consciousness ct done was hematoma next:
A.admit to icu
B.give mannitol
C. evacuate in ER

Baby with supracondylar fracture with absent radial pulse management?


A- Urgent K wire
B- Elevate arm
C- Observe
D- Surgery

‎‫اتوقع (بعد اسبوع) جاته‬، ‫ جاء سؤال يقول واحد ماخذ كيموثربي‬varicella ‫وش نسوي له؟‬
Give him acyclovir
Stop chemotherapy for ( not sure if 2 weeks or 2 months)
Give him vaccine
Give immunoglobulin

The question isnt clear. If he got the infection then start IV acyclovir, if didnt get the infection give IVIG

child came with post URTI complaining of SOB Wheezing, RR 33, HR 100, O2 90%, other vitals
normal. What’s most appropriate management?
A. ABx and ventolin
B. IV fluid and ventolin
C. Ventolin and systemic steroid

Which of the following is the earliest plain radiographic finding of rheumatoid arthritis?
A- Juxta-articular osteopenia
B- No abnormality
C- Soft tissue swelling
D- Subchondral erosions
E- Symmetric joint space loss

If you have a disease that has high incidence female and equal prevalence in male and female
How did that happen ?
A- Case fatality is high in female
B- Male patient die more from this disease
C- Female carry this disease for longer period
D- Male carry this disease for shorter period
20 y male pt came to the ER with 2 months hx of bad personal hygiene , echolalia , echopraxia ,
muttering , good memory what drug to give him ?

A. oxcarbazepine
B. venlafaxine
C. lithium
D. amisulpride

‫ا‬Amisulpride one of the atypical antipsychotics effective in argumentation


But remember the first line therapy for cataonic schizophrenia is benzodiazepine

2)49 year old man heavy smoker , otherwise he’s healthy came for lung cancer screen what do
you do for him?
- low dose CT
-sputum cytology
-chest x ray
-maybe something biopsy

Lung cancer screening begin 55-80 in currently smoker or stopped less than 15y

64 old male Known case of DM2 , recently Diagnosed with Symptomatic peripheral Artery
Disease,, started Supervised exercise program
Which of the following is recommended to prevent cardiac event :
- Aspirin
- warfarin
- SC heparin
- non related

Child who have URTI recently resolved but still C/O dry cough at night given Alopuritol but no
relief
What to give next :
- Cough sedative
- ICS
- oral steroids
- LTRA

- p.t after MVA At ER , Blah blah


Vitally stable O/E ( Lift hypochondrium tenderness and ecchmosis )

What is the MOST APPROPRIATE TEST :

- CT
- FAST
-DPL
- Laprotomy
17 yrs old primigravida came to ER c/o vaginal bleeding and abdominal cramps , pregnancy test
positive at home US : shows its 9wks and closed cervix with blood through ,as well as positive
heart beat (they describe Treatened Abortion)
“No vitals given”

Mx :
A- Reassure and F/U after 1 wk
B- admit,stabilize, prepare for possible termination of pregnancy
C- oxytocin

P.t known case of RA controlled on HCQ 400mg and Prednisone 15mg


Well controlled for period ( forget )
All labs include LFT ( N )
EXCEPT ( high FBG and HbA1c )

- what’s your next action :


A- taper Steroid and start MTX
B- taper steroid and start Ibuprofen
C- continue same Mx

the patient was stabbed in multiple areas, came to the ER and was awake,
his clothes were full of blood, his bp is around 80/60, (O2 sat 88%)which of the following is
best next step?
A) oxygen supplementation.
B) iv crystalloid infusion
C) Intubation

Female with urine incontinence (during coughing and laughing)


What will you do next ?

A- Bonny test
B- retrograde urethrogram
C- voiding cystourethrogram
D-Pelvic examination

Pt in ICU with Euthyroid sick syndrome/ non thyroidal illness (Given the dx in scenario). what his
labs would be like?
-Elevated T3 & T4, low reverse T3
-Elevated T3 & T4, High reverse T3
-Low T3 & T4, Low reverse T3
-All of them r low

Which of the following indicate hemophilia workup?


-Age above 50
-On OCP
-Hx of connective tissue disorders
-negative Family hx of coagulopathy

Post menopausal woman with itching & watery secretion & scaling & tender vulva. What’s likely
the dx?
-Atrophic vaginitis
-trichomonas vaginalis
-Vulvovaginal candidiasis
-Bacterial vulvovaginitis

Q about which of the following is the best initial test for Celiac pt?
-Iga tissue transglutaminase antibody
-Anti-endomysial antibody
-Endoscopy

Patient came with epigastric pain radiating to the back for 6 days , he has history of multiple
gallstones, now he is complaining of abdominal pain. Physical exam confirm he has abdominal
distention and sluggish bowel sounds. All labs are normal ( LFT and Bilirubin level).
Erect cxr: left pleural effusion.
What is the initial next step?
- abdominal ultrasound
- abdominal CT scan
- ex lap

506-Pt in his 60’s came with flank pain, tender mass in lumber region his vital BP= 160/90
What’s your investigation ?
A-US
B-CT abdomen
C-MRI
D-Radionuclides

Women did hernia repair whats the minimal time that she can get pregnant?
A-1 month
B-4 month
C-6 months
D-12 months

Minimal 6 months , best 12 months

Patient presented with SOB after trauma , upon exam there was dullness in the left side
What is the most likely diagnosis?
- Tensions pneumothorax
- cardiac temponade
- massive hemothorax
- pulmonary contusion

a 30 years old male medically free complaining of chest pain that is localized and tender in touch
and aggravated by movement the patient noticed that the pain started after intense exercise
program last week.physical exam is normal.
What is the appropriate management?
A-atenolol.
B-ibuprofen.
C-reassure and reassess
D-nitroglycerine

patient kc of copd came complaining of UGI bleeding stabilized after endoscopy.after that he
became dyspnic and cyanosed.
his workup is as follows (approximation)
ph 7.24
pco2 7.4 not sure of the number but higher than normal
What is the most appropriate initial management?
A-dexamethasone
B-IV mg
C-high flow O2
D-intubate and hyperventilate

Traumatic patient presented to ER with Profuse bleeding from nose and mouth , cyanosed with
decreased breath sound on right side of the chest. Mostly was unstable Which of the following is
the most appropriate next step.

A-Right chest thoracostomy


B-Intubation
C-IV fluid resuscitation and O type blood transfusion.

Femal with heavy veginal bleeding. Hypotensive, Hb 7 . Proper next step?


A. hysterectomy
B. Treat anemia
C.Progesterone
D. D&C

After Vaginal delivery . Pt had vaginal bleeding . doctor see multiple venipanctur bleeding . Which
of following is appropriate management:
A- Coagulation correction
B- internal iliac ligation
C- paking the uterus
D- B lynch

-normocytic normochromic + inclusion body


A- lead poisoning
B- SCD
C&D not related

unilateral neck swelling in the RT side by investigations :


hot thyroid nodule remaining of the gland cold
TSH is low,
T3, T4 high
No LN enlargement (dx hyperthyroidism toxic nodule)
What is the initial Treatment?
A- antithyroid drug
B- RT thyroidectomy
C- Hemithyroidectomy
D- radioactive iodine

2 couple came to infertility clinic because of 2 years no pregnancy the husband look healthy and
well while wife Dm obese BMI 30?
What to do
A- infertility test for wife only
B-lifestyle change for wife as she get pregnant
C- test for both
D-I don’t remember

Female with genital warts: what's the management?


A- sclerotherapy
B-electrocautery
C- injectable steroid
D- Podophyllin resin
Both B/D correct but I chose B

seizure for 3 days,


lab
serum osmolarity and NA: low
K: normal
urine osmolarity( normal
⁃ conns
⁃ cushion
⁃ SIadh
_ addison

px with typical symptoms of DM( polyuria, thirst, ..)


on lab: the random blood glucose was normal
ask about the cause of symptoms
- Decrease renin
- decrease insulin
- increase insulin
- absent of glycogen

‫جاني سؤال عند‬posterior shoulder dislocation ‫وكانوا حاطيين اسم العظم‬


Subglenoid anterior
Subarachnoid anterior
‫ وكذا جواب بعد نفس الطريقه يغيرون الى‬post ‫و اسم العظم‬

Patient came to the ER with signs and symptoms of myocardial infarction he was going for PCI,
when the cardiologist was assessing his condition he notice the patient was depressed with low
mood, the patient refused the PCI he demonstrate good understanding of his problem to the
doctor what to do
A. Treat the patient regardless of the consent
B. Refer the patient for psychiatric assessment then take the consent
C. Respect the patient choice after discussing the reasons of refusing
D. Take the consent from the patient relative

First thing to do for a patient came with swelling of joint and arthritis
1- arthrocentesis
2- MRI
3- US

What type of fibroid causes heavy bleeding?


Submucosal

But the most common is Intramural

A patient came with lower limb pain and no pulses in right limb while the pulses in left limb are
intact
How to differentiate between acute and chronic of this case ?
1- history of intermittent claudication
2- absent pulse in the right limb

most important initial step to reduce ICP


1- hyperventilation
2- elevate head
3- IV mannitol

RTA patient multiple bruises in right side High blood pressure HR : 70 Most likely cause : ICQ
1- cardiac tamponade
2- spinal injury
3- pneumothorax
4- internal bleeding

when to check eradication of H.pylori


1- 4weeks
2- 2weeks
3- 3weeks
A 60-year-old man is admitted to the Coronary Care Unit with an acute myocardial Infarction. His
hemodynamic parameters 2 hours later are:
Blood pressure :80/50 mmHg
Heart rate 40 /min
Oxygen saturation °C 95% on room air Which of the following would be the appropriate
management?
A. IV0.6 mg atropine sulphate
B. Normal saline infusion
C. IV isoproterenol
D. IV dobutamine

Known case of something ( iforgot) came with peptic ulcer perforation and need urgent surgery
Inr :2
Plt 90
Hgb 90

What to do first ?
Cryoprecipitate
FfP
Plt transfusion
PRBC

Question about DM incidents and prevalence after insulin


Decrease prevalence
Increase prevalence
Decrease incidence
Increase incidence

Known case of COPD came with HF s&s


What to do next
Echo
Spirometer
I forgot the rest but irrelevant

Pt went for ovarian cystectomy and histo found that thick fatty material
Sorry forgot the choices but asking about the type . I chose thecoma

Similar recall
A
B

Pediatric 18 m introduced cow milk at 9m after stopping exclusive brest milk came with soft stool
⁃ cow milk allergy.
⁃ The rest irrelevant

Newborn with brother of immune deficiency. Regarding his vaccinations?


A- Dont give BCG
B-give vaccination as saudi schedule

Female pregnant patient she received anti-D at 20 weeks of gestation, what is the next does
A. 300 mcg
B. 1000 mcg
C. 2000 mcg
D. No need for further dose

Patient presented with headache, blurred vision and BP was 160/100. And she has seizures What
is the diagnosis?
Eclampsia

Senior of female 30 week gestation eclampsia, after resuscitation and Mg su was given, what is
your next step?
Delivery

A 24y.o female G2 P1, presented with premature labor at 34 weeks with twin pregnancy? What
increases the risk of having PPH?

Age of the patient


Multiparty
Multiple gestation

What decreases the pre eclampsia? Aspirin


What decreases eclampsia? Mg sulphate
Epidural anesthesia (not mentioned directly), what is speared?
Rectum

Patient has PPH, what you will give her?


5 units of oxytocin
20 units of oxytocin in dextrose

According to Dr. wafaa notes :


Medications used in PPH Management:
1) Oxytocin 10-40 units in 500/1000ml NS/RL or 10 units IM (1st line) (For patients in whom infusion of
sodium chloride must be avoided, 5 % dextrose solution may be used)
2) Methylergonovine 0.2 mg IM (2nd line, Contraindicated in preeclampsia and HTN)
3) Carboprost (hemabate) PG F2a 0.25mg IM (3rd line, Contraindicated in Asthma)
4)Misoprostol 600-1000 micrograms PO, or rectal (4th line)

2 day old infant came to hospital with complaints of seizure and decreased feeding since
yesterday. Inactive child with generalized increased muscle tone. CSF analysis is normal.

What is the most likely diagnosis?


Hypoxic ischemic encephalopathy
Neonatal sepsis
Neonatal tetanus
Pyogenic meningitis

32 year old 26 weeks pregnant with large gentiaal wart.


What is the most appropriate management?
Cryotherapy
Resin
Electrocautery
Intralesional interferon

2 year old with diarrhea, distension, growth delay, pallor and buttocks wasting.
What is the most approximate investigation?
A-Duodenal biopsy
B-Endomysial Ab
C-Colonoscopy
D-CBC

Initial would be anti-ttg or anti endomysial

3 year old came to clinic with lethargy and failure to thrive


Hg 88
Mcv 65
Normal RBCs
Reticulocyte 5
Mch low
What is the most appropriate management :

A. Blood transfusion
B. Im iron
C. Oral ferrous sulfate
D. Fortified cereal

A 20 years old male was hit by a car 2 hours ago, abdominal examination revealed distinded
temse tender abdomen
Bp: 100/70
RR: 30
HR: 120 T: 35 O2: 95
Which of the following is the most appropriate?
A) spinal
B) cardiogenic
C) hemorrhagic
D) anaphylactic

A 2 months visit well baby clinic. The mother is counseling about when she can start giving solid
food to her baby, which one of the following is generally recommended?
A) 3 months
B) 4 months
C) 5 months
D) 6 months

A 40 years old female is complaining of stress urinary incontinence. She is asking you about the
most effective management?
A) pessary
B) Kelly something
C)med-urethral sling
D)

A 50 years old women is complaining of amenorrhea for 6 and hot flushes.. which of the following
is going to help reach a diagnosis?
A) FSH
B) LH
C) progesterone
D) estrogen

Case of wilsons (decrease in intellectual ability decrease school performance seizure tremor) how
to diagnose
A-Liver biopsy
B-24 hr urine copper

34 year old with splinter hemorrhage and fever and pansystolic murmur
No history of surgery or medical illness
What is the empiric treatment will give
A - ceftriaxone
B- gentamicin
C - ceftriaxone and vancomycin
D- ‫نسيت وش هو الصدق بس ماله عالقه‬

40 years old female with menorrhagia for 3 months, menarche started at age of 13. Risk factor for
endometrial cancer?
1- late menarche
2- early menopause
3- diabetes mellitus
4- progesterone producing tumor

By exclusion C

Kawasaki what to give vaccine:


MMR

‎‫ جا سؤال فيه اعراض ال‬CF ‫وقال وش يجي بعد‬:


Nasal polyp

Artery involved in epidural helmatoma:


Middle meningial artery

72 ‫ جا سؤال‬YO
+TIA
+A fib
Treatment:
A-Aspirin
B-warfarin 2-3 INR

‫وجاني وحدة عطيتها بروفين وتحسنت الديسمينوريا والحين جايه زعالنه تقول ابغى مسكن أقوى‬
Counselling and educate about self care

Postpartum depression
What to do beside psychotherapy:
A-Small dose of antidepressants
B-Mother breast feed the baby
C- involve family in therapy

Q2: Sickle cell trait, what’s the most common complication during pregnancy?
A- IUGR
B- Chest infection
C- Preeclampsia
D- UTI

Pt did an operation on his leg and now what is needed?


-fasciotomy

🌟Diabetic type 1 had dka,, on discharge what is the best regimen:


⁃ Twice nph
⁃ Once mix??
⁃ Basal bolus with once glargine
⁃ Short acting before meals and no long acting agent

🌟38 year old lady history of weight LOSS and couldn’t breastfeed her last child and her menses
didn’t return since her delivery (they didn’t mention her delivery date or age of her last child), they
mentioned alot of hypothyroid symptoms.. labs showed low Hg, low sodium, Low T4. What is the
appropriate thing to do:
⁃ Give eatrogen
⁃ Order TSH
⁃ give thyroxine 50 micro daily

patient k/c of crohns, having refractory hypokalemia


⁃ Assess aldosterone angiotensin level
⁃ Give IV magnesium sulfate

pregnant 39 weeks, in latent phase of labor (didn’t mention for how long) cervix is 2 cm, CTG
shows basal HR 140, moderate variability, no acceleration or deceleration, mild and irregular
contraction.
⁃ observe
⁃ Induce labor
⁃ Augment labor
⁃ C/S (not if this choice was there)

positive secretin stimulation test


⁃ VIPoma
⁃ Gastrinoma
⁃ Carcinoid

Pregnant unbooked presented with vaginal bleeding no abdominal pain, fundal height 34. have no
transportation method to hospital. NEXT step:
⁃ US
⁃ Admit to ward
⁃ Cervical exam
Post CBD surgery complains of chills and rigors. BP 119/? HR 80 RR 28 T38. Normal WBC ( 11 i
think?).
⁃ sirs
⁃ Bacteremia
⁃ Sepsis
⁃ Severe sepsis

SLE patient having edema and dyspnea high JVP no gallop or heart abnormality. Lab: high
creatinine, low albumin, normal platelet, high ALT and AST. What is the likely diagnosis ?
⁃ liver failure
⁃ Renal failure
-constrictive pericarditis

what is the most common organism for infective endocarditis:


⁃ Staphylococcus epidermidis
⁃ Streptcoccus species
⁃ Enteroccous species
⁃ Forgot but no staph aureus

pregnant lady first antenatal visit. She is known case of diabetes and hypertension.
⁃ Review of her medication
⁃ Dietician

Abdominal pain and loose stool for months associated with weight loss. Biopsy from the ileum
showed non-caseating granuloma. What is the diagnosis?
⁃ Pseudomembranous colitis
⁃ Intestinal Tuberculosis
⁃ Crohns
⁃ Celiac

Pt with stroke, confirmed by imaging haemorrhagic asking Most definitive treatment of


hemorrhagic stroke
IV steroids
mannitol
Surgical decompression and evacuation

Colon polyp, turned out to be 1.3 cm tubular adenoma, when to repeat colonoscopy?
A. 3-6 months
B. 3 years
C. 10 years

1-2 tubular polyps less than 1cm -> every 7-10y


3-4 tubular polyps less than 1cm -> every 5y
More than 5 tubular polyps OR more than 1cm OR tubulovillus OR villous OR high grade dysplasia ->
every 3y
Female high prolactin with amenorrhea and white breast discharge imaging for what structure?
Sella turcica

Tall and joint hyper laxity , what investigation will prevent life threatening complications?
-echo
-ecg
Forget others, colonoscopy?

CTA is the answer -> Aortic dissection

Known SLE on medication, presents with Lower limb weakness,


Exam: paraplegia and hyperreflexia, investigation?
-Lumbar puncture and brain mri
-Lp and brain mri, mra, mrv
-Lp and ct brain
-Lp and mri spine

13 year old,
Seems to have IBS
-avoid peperment oil
-Avoid lentil
-Increase honey
-Increase (forgot)

Pica, low socioeconomic status, pale.


Lead levels: 2,
Digoxin level normal,
Iron levels normal ,
hbg low,
What to give?
-D penicillamine
-Iron
-Digibind

Pica, pale, (nothing about socioeconimic status) Investigation? ICQ


-Iron studies
-Lead level

Open fracture initial management? (Was dirty, )


-Antibiotics
-Debridment
-Internal fixation
-External fixation

Stab injury to arm, Nerve and tendon cut, Clean wound.


Management?
-Primary repair
-Debridment and secondary closure
-Debridment and prinary closure
-Debridment and secondary graft

Hard mass in right thyroid,


Turned out to be Papillary thyroid cancer management?
-Hemithyroidectomy
-Subtotal thyroidectomy
-Hemithyroidectomy with isthmusectomy
-Total thyroidectomy

Trauma patient, currently well. lucid interval reported by paramedics, now he is deteriorating.
Most likely diagnosis?
-Epidural hematoma
-Subdural hematoma
-Base of skull fracture

Placenta is deep in the uterus,


Which one is it?
Placenta increta
Acreta
Percreta

Went for myomectomy of subserosal fibroadenoma, but opened the uterus.


Risk for placenta accreta?
-increased
-decreased
-Unknown
-not affected

Elderly man complains of urinary retention, had back pain in the past months. High PSA.
Possible diagnosis?
-Prostate cancer
-Benign prostatic hyperplasia

Elderly, 6 moths history of parkinsonism, early dementia, (forgot the third thing).
-parkinson's disease
-lewy body dementia
-alzhiemer

Right sided weakness, sensation intact.


Which artery is affected?
-basilar
-vertebral
-branch of middle cerebral artery
-anterior cerebral artery

Prosthetic valve, now with endocarditis, treatment?


-Vancomycin, ceftriaxone
-Vanco, rifampin, gentamicin
-Vanco
-Vanco, rifampin

Pt with family history of diabetes,


Fasting glucose high, HbA1C high
initial management?
-Metformin
-Liraglutide

20days postpartum, vaginal discharge, white- yellow no smell no itchiness,


microscope: white cell and epithelial?
-Reassure
-Culture discharge
-Urine culture
-Metronidazole

20weeks Pregnant with rash and jaundice, Medically free, Elevated LFT.
Likely diagnosis?
-Cholestasis of pregnancy
-Hepatitis
-Budd chiari

The Patient was in a RTA 2 weeks back,


Now has symptoms of pneumothorax (Reduced air entry on right side). (I think the Radiograph
image showed reduced right lung size)
Management?
-Needle
-Chest tube

2 questions
Asking for Gestational age, unknown LMP?
Asking for Gestational age, (‫?)الدورة ملخبطة عندها‬
-Quantitative bhcg
-Progesterone
-Crown rump length
-Estradiol
-Fundal hight

One year old child, his sister has an immunodeficiency, what vaccine to avoid?
- Opv
-Varicella

Patient with CKD stage 2 with fatigue and pallor investigations show anemia and low MCV and
slight increased something MCH maybe. Next step

A (Serum) Iron study


B Reticulocyte count
C Serum Transferritin
D Serum erythropoietin

type of estrogen in pregnancy


Estriol

Estradiol -> in young bearing age


Estrol -> menopause

contraception methods
- in PE -> IUD
- in cardiac issue IUD
- breastfeeding not want getting pregnant for 2 years -> Depoprovera injections

Case about an old man with tender joints, forgets his friends name and celebrity names, his wife
is scared of Alzheimer’s
Alzheimer’s
Benign forgetfulness

This case came in two scenarios , one the patient himself saying he forgets then its benign forgetfulness.
The other scenario the wife complains that he forgets then its alzheimer

Reduced variability CTG (only pic they didn’t mention the diagnosis) asking about which
medication causes it

Mg Sulphate
Epidural
Oxytocin

RAI is absolutely contraindicated in pregnant

‫كيس بيشنت حامل عندها ثيرويد نديولز‬


‫ايش ممنوع نعطيها والجواب راديو اكتيف ايودين‬

Most common complication of hysteroscopy :


Infection
Adhesion
Bleeding

Perforation then infection


55 years old pt with family hx of type 2 DM
Fasting blood glucose= 7
HBA1C2 7.5

Dx?
1- prediabetic
2- impared glucose
3- diabetes mellitus

Scenario about placenta previa ( diagnosis is given) then asked: what is risk factor for this
condition?
1- HTN
2- DM
3- multiple gestation

Patient with malar rash, rynoid phenomenon, joint pain of MCP joints, healed ulcer in top of 2nd
finger, proximal muscle weakness pt can’t climb the stairs, what is diagnostic AB?
1- anti ccp
2- anti smith
3- Anti ds dna
4- anti RNP

Women 33 weeks with severe placental abruption(written exactly like this), with IUFD and DIC,
contraction with cervix is dilated 3 cm, how to manage?
( no vitals mentioned)
1- observation
2- augment labor
3- CS

Stable b , unstable C

After an accident found to have air under the diaphragm and spleen laceration and thoracic aortic
perforation [ patient hypotensive], what is the first thing to do?
A. Thoracotomy
B. Exploratory laparotomy

Female 47 y/o complaining of abnormal vaginal bleeding, what would be the most appropriate to
establish diagnosis ?
A-Endometrial sample
B-Pelvic US

Same question for other age, i think 51, and didn’t mention US
PT with Crohn
which of the following increases risk of malignancy
a. Cancers located within 15 cm from the anal verge are
B.Primary sclerosing cholangitis
c. duration of 3 years

A full-term C-section baby presents with intercostal retractions and cyanosis.


What is the most likely diagnosis?
A- Transient Tachypnea of Newborn
B- Meconium Aspiration Syndrome
C- Hyaline Membrane Disease
D- Bronchopulmonary dysplasia

DM with cardiac disease your goal in management?


A-triglyceride less than 150
B-BP less than 140/90
C- LDL less than 70
D - HDL more than 70

Female first visit ...found E coli 1000.. asymptomatic management ..


a.Nitrofuntoin
b.Cipropfloxacin
c.Trimethoprium
D. no need

If pregnant A, if not then D

What is the drug contraindicated in familial Mediterranean fever?


A)paracetamol
B) ibuprofen
C) amoxicillin
D) aspirin

Pt 65 years old came with sever diffuse abdominal pain 10/10 known case of a fib and dm and htn
Upon the examination the there is diffuse pain all over the abdomen with rigidity no vitals Only
lactate was 4 (above normal) what is your next step
Exp laprotomy
Colonscopy
Laproscopy

Pt elderly came with RUQ pain radiate to tip of the right shoulder with fever also complain of
shortness of breath he has hx of repaired perforated deudeenal 2 weeks ago The did chest x ray
they found lower lobe atelectasis. what is your next step
Us of abdomen
Ct of chest

Pt who have recent travel history to india c/o Right upper quadrants pain US show hypoechoic
mass?
amebiasis
hydatid cyst

Obese female concerning about GERD as a complication of gastric sleeve


Percentage:
A-5%
B-10%
C-20%
D-30%

Pt k/c IHD come After lap chole feel chest pain + lab show hypotension, what inx :
Ecg
Cta
Cxr

Which one CI of vaccine?


Abx
Sever acute illness
Recent infected by infectious disease

PPROM which one most imp in mx


Steroid
Tocol
Mgo4
Abx

Fetus 180 HR , early deceleration And other information, and ask about which one indicate
severity I think:
HR
early deceleration

Baby born with microcephaly (forget 2ed one but looks like CMV ) ask about next ?
congenital infections screening

An 8 years old healthy male, presented with a history of 2 months of abdominal pain with 2
episodes of bloody stool. Proctoscope was done and showed numerous polyps covered the
linings and multiple biopsies taken. What is the most likely diagnosis?
AFamilial polyposis
B Ulcerative Colitis
C Diverticulosis coli
D Human papillomavirus polyp

Child with neck nodule that moves up and down. And moves up with tongue protrusion. What’s
the most likely diagnosis?
A. Dermoid cyst
B. Thyroid nodule
C. Thyroglossal cyst
D. Parathyroid nodule

Patient was in the ward after gallbladder empyema surgery, deteriorated and needed mechanical
ventilation transferred to icu, has leucocytosis, fever.?
A-sirs
B-sepsis
C-severe sepsis

researchers are trying to collect data from multiple studies, how to apply that?
A. Meta analysis
B. Case control
C. Cohort stud

Case of trauma came with hypotension and pr:70 type of shock


Septic
Neurogenic
Carcinogenic
Hypovolemic

Hypotension + bradycardia = Neurogenic shock

Hx of paraplegia due to trauma one year ago presented with PE:


Alteplase
Enoxaparin
Warfarin

Unstable A
Stable B

Child I think 9yo with red urine and she had constipation weeks ago
Urine high leukocytes, protein +, RBC ++

UTI
Post Strep glomerulonephritis
O-ve mom, +ve baby (mom was give anti d at 28 weeks) now she gave birth, what the dose of anti
d she should receive?
300ug (or 30 maybe)
100ug
1000ug
No need

Treated HCV case with negative RNA. How do u FU?


Liver US every 6 months
AFP every 3 months

If patient has cirrhosis then A, if not then no need

HCV antigen negative, antibody positive and RNA negative. What to do next?
FU US
Reassurance

Child with beckwith Weidemann syndrome, doctor explained to the mom that it has risks of liver
tumors Screening for such a case?
Urine oxide
AFP

22-month infant with LEFT inflamed red hemi scrotum, on examination there was red firm
irreducible painful scrotal swelling which is extended to to left inguinal region. LEFT testis
couldn't be palpated. What is the diagnosis?
A. Torsion appendix testis.
B. testicular torsion
C. epididymo orchitis.
D. incarcerated inguinal hernia

2. Appendix perforation antibiotic coverage


Positive aerobes
Positive anerobes
Negative aerobes
Negative anerobes and aerobes

3. SLE with high creatinine, edema, heart sound far? High creatinine low albumin
MI
Liver failure
Renal failure
4. Pelvic fracture fluid.
RL
NS

‫سؤال عن لما نبغا نطلع بروبوزال لبحث كيف نطلعو‬


‫انو تاخدي رأي زمالئك الي بالجامعة وال بس كدا بكيفك تختاري الموضوع الي تبغيه و تشتغلي عليه وال تروحي جهة مختصة مسؤولة عن االثكال‬
‫ابروفال و تطلعي ابروفال بعدها يسير البروبوزال‬

What activate TB?


-Silicosis
-asbestosis

41 Ga .. position with vertix , labs normal ,ctg normal and cervix 4 cm


What to do?
-CS
-induction of labor
-augmentation of labor
-reassure

Prolonged labor in MULTI how many hours?


-18
-10
‫كان فاالختيارات‬

Child with noisy breathing in sitting and supine position, decreased when the child is prone, the
mother is worried, what will you do?

A reassure
B Flexible (Nasopharyngoscope)

Patients with UC exacerbation on medications 6 bloody diarrhea per day and abdominal pain.
Culture showed C. Difficile
How would you manage this patient?
A IV Cefuroxime
B IV Metronidazole
C Oral Vancomycin

Pregnant 28 weeks with +ve bacteriuria what’s tx


1-ciprofloxacin
2-nitrofurantoin
3-TMP/SMX

1- Trauma Pt with extra peritoneal bladder injury?


A) A-Suprapubic catheter
B) B-Urgent exploration and repair
C) C-Catheter repair and assess after 2 weeks
D) D-Catheter us after 2 weeks

2- A patient with intestinal obstruction and perforated and the doctor decided to do EX LAP what
is the contraindications?!
A) nitric oxid
B) propofol

3-Kid with cervical lymphadenopathy and sore throat 3 weeks ago. Has Coca Cola red urine.
A. IGA Nephropathy
B. Acute glomerulonephritis
C. Acute cystitis

4- Patients with bleeding peptic ulcer and history of long use of aspirin. What is the
managent?
-High dose oral PPI BID
-IV PPI for 24hr followed by oral PPI
-IV PPI for 48hr followed by oral PPI
-IV PPI for 72 hours

5-How to diagnose celiac


Anti-endomysial antibody
tissue transglutaminase antibody

Intestinal biopsy shows subtotal villous atrophy

6-Best way to diagnose endometriosis


Endometrial biopsy
Ultrasound
Laparoscopy

7-Calculate the GCS score of someone who opens his eyes to painful stimuli, says
incomprehensible sounds, and flex his arm due to pain

opens his eyes to painful stimuli = 2


says incomprehensible sounds = 2
and flex his arm due to pain = 3
7

8-What contraceptive med is used for ovarian cyst


Progesterone pills
OCP
Depo

9-Medial ulcer, what is rhe risk factor?


DM
Atherosclerosis
Venous HTN

10-pt wt systolic murmur , hepatomegaly?


vsd

11-Ecclampsia pt came to ER seizing what to give her


-1valpric acid
-Magnesium sulfate

12-Sandpaper rash what is the organism?


A- streptococcus varidans
B- streptococcus epidirmidis
C- streptococcus pyogenis
D- staphylococcus

13-Pregnant lady, and a mass. What to do?


A. Colposcopy
B. Pap smear.
C. Cone Bx.

14-A child presents with a salmon patch macule, hepatosplenomegaly, and multiple
symmetric and asymmetric joint pain. Rheumatoid factor was negative. What is
the most likely diagnosis ?
a.Juvenile psoriatic arthritis
C. Systmeic Juvenile idiopathic arthritis

15 -Child with chlamydia diplococci


(conjuctivitis etc.)
Asking about the causative organism

18-fibroid felt on pelvic examination whats the location


intramural

21-MOH reported the infant mortality rate in 2020 which was 4.81 .
Which of the following help in calculating this ratio

A-Knowing the childbearing women in the middle of 2020


B- Population percentage in Saudi Arabia
C-Knowing percent of those died before the first year of their life.

22-Why is acei CI in pregnancy


-Drop BP very low
-Cause congenital malformation

23-Abdominal mass cross midline


-Neuroblastoma
-Willams tumor

26-How to know fetal wt intrapartum


A-Femur length
B-Head circumflex
C-Biparietal diameter
D-Abdominal circumference

27 -Mother came for antenatal care and US shows week 32 reversed end diastolic blood
flow:
A. Follow up 2 week and reassess
B. Immediate delivery now
C. Administer steroids 1 week and delivery
D. NST

29- post roux-en-y developed shoulder pain whats the initial step of management
Stable -> ct with iv contrast
Unstable -> laparotomy

30- what congenital anomaly could be associated with oligohydramnios


Renal anomalies

2- 6 days old infant , had sob , dyspnea poor feeding , his mother was diabetic . Dx?
A- ARD
B- bronchitis
C-

3- 50s female with 2nd degree vaginal prolapse, urgency , incontinence. Tx

4- pregnant with pyelonephritis symptoms . High urine wbc . Tx?


A- oral abx
B- admission for iv abx
C- renal us

12 - Pt with neck swelling move with swallowing . In US the mass was solid and 2.5 cm
What is the most appropriate investigation

A- thyroid scan
B- FNA
C- Ct of chest
D- TSH
18- Pt with hepatitis C + liver cirrhosis treated , - ve HCV RNA, what to do next?
A- reassurance,F/U
B- FU with US
C- liver biopsy

Pediatric with abnormal movements + tea color urine+ piruritis + lower limb edema
+ slightly high blood pressure
A- HUS
B- ‫‏‬ITP
C- ‫‏‬PSGN
D- ‫‏‬Forget

HSP

50 years old c/p of severe bleeding failed medical management


UC show fibroid 7*3
What is the definitive treatment ?
Hysterectomy
Uterine artery embolization

If young and want to preserve fertility then B

45 man with hematuria and flank pain uss show dense echos And radioopaque I forgot rest
Uric acid stones
Sloughed renal papilla

Patient presented to the ER with a history of 3 day fever and cough and rash all over the body.
There is consolidation and crepitation.
What is the treatment?
⁃ Antibiotic
⁃ Acyclovir
⁃ Antipyretic

Male with gonorrhea , what is the BEST procedure to diagnose him:


Urethral swab

😢
11 yrs with fatigue and exercise intolerance and they mention the word “weight”‫ ا‬then didn’t
complete the sentence what to order?
CBC
TSH
Others unrelated

Patient drip urine on coughing ?


Stress incon
91 yrs old in a residential home for geriatrics no drug history was given came with 24 hrs of
severe abdominal pain on examination there was generalized nonspecific tenderness no bleeding
no vomiting most appropriate initial thing to do?
Colonoscopy
Sigmoidoscopy
Digital rectal examination

What drug cause patent ductus arteriosus?


Indomethacin

Long case about uncontrolled DM


Bp in sitting 168/90 I didn’t remember exactly but was very high then another reading on standing
115/60 what’s the must likely complication
*Autonomic neuropathy I think

- A Case of endometriosis (dx given) and you are asked what is considered an acceptable
diagnostic method?
- Us
- Laparoscopy
Gold Std: Laparoscopy, Acceptable US

Both are given and the questioner didn’t specify what answer is wanted , an initial or confirmatory
modality, just asked about an acceptable one!)

- A young man fell on an outstretched hand and there was pain at the anatomic snuff box:
-Hamate fracture
- Scaphoid fracture
- Colle’s fracture

- A case of dysphagia and other related s/s, there is a barium swallow report that would lead to
you to the answer:
Report: ( dilatation of the esophagus with loss of peristalsis and the lower sphincter won’t relax)
No picture was provided.
- GERD
- achalasia cardia

4 -1‫‏‬year old Child with recurrent chest infection, inspiratory crepitation and expiration weezing,
poor growth and fatigue, finger clubbing, on further examination he has offensive greasy stool,
what will you find on further examination?
‫‏‬nasal polyp
‫‏‬inguinal hernia
Dx: Cystic Fibrosis
Child presented to the ER with his parents as they were saying that their child and just said a
potentially toxic Medication. What is the next step in management ?
A- Active Charcoal
B- gastric lavage
C- Ipecac Syrup
‫السؤال ماكان فيه النوع الدواء وال الوقت اللي مضى‬.

Postop fever and other signs and symptoms, surgery were before 4 days (I think)?
1. Atelectasis
2. Pneumonia
3. UTI
4. SSI
‫ماني متأكده من الخيارات‬

‫ ‏‬ld age has severe abdominal pain just above umbilicus just say like that + Vital sign i think he is
o
mentioned it and there is no significant
‫‏‬What is the test needed to be done?
‫‏‬-Amylase
‫‏‬-ABG

Initial A, intial radiological US , best CT

32y/o
‫ ‏‬woman, vague abdominal pain 2 month no menstruales
‫‏‬Pregnancy test
‫‏‬US

‫ ‏‬rimigravida with pain post-defecation pain and blood stool after defecation, describe it as sharp
p
and.. What is the Dx?
‫‏‬-Anal fissure
‫‏‬-Perianal abscess

‫ ‏‬hild with dark urine and pain , in lab there is positive nitrate.. What first investigation to order?
C
‫‏‬-Urine analysis
‫‏‬-I think US .. i can’t remember:(

‫ ‏‬t with headache, fatigue..before menstrual ..ect >> Unfortunately i can’t remember the whole Q
p
but it was something about premenstruales syndrome and there are No options like PMS or PMDD

‫ ‏‬ld age with bleeding . What next investigation?


O
‫‏‬Biopsy
‫‏‬US
If vaginal bleeding intial US , definitive sample

‫‏‬Q about pt with .. ‫لها عالقة بالرئة او فحوصات للربو صراحة ناسيه‬
‫‏‬Intrabroncheal biopsy ICQ

‫ ‏‬ld age i think 52 y/o , last menstrual 12 month ago..ect


o
‫‏‬What is the responsible for this condition?
‫‏‬-FSH
‫‏‬-Estradiol
‫‏‬-progesterone
‫‏‬-testosterone

Most associated with weight gain


A- Olanzapine
B- colanzapine

Colanzapine -> olanzapine -> Quetipine -> Risperidone

Pediatric asthmatic, good control but not improve, his mother complain about baby weight is low
compared to other children at his age Examination :chest normal Weight and height below centile
A- screen for other dx mimicking asthma

man obese with somelence and fatigue + BMI high


What is the investigation?
A ECG .
B- Echo,
C- Chest X-RAY
D- sleep study

chest pain + palpitations HR: 270


A- chest X-ray
B- Echo
C- 12 lead ECG
D- stress test

how to remove the placenta if it takes more than 30 minutes after vaginal delivery ?
A) Manual removal
B) Wait
C) CS

How to differentiate between crohns and UC?


A. Extra intestinal manifestations
B. Non caseating granuloma
C. Aphthous ulcer

55 years old female menopause present with heavy vaginal bleeding associated with weight gain
BMI 35. TSH done was normal
Endometrial biopsy showed adenomatous hyperplasia
What is pathophysiology?
A. Adrenal hyperplasia
B. Peripheral conversion of precursors to estrogen
C. Genetic mutation of endometrium

Long scenario about a female with celiac and iron deficiency on iron ferrous 325mg but still
complains of symptoms what would you do?
A. Mix in oral solution
B. IV iron
C. Divide dose three times

17‫‏‬years old female present with jaundice and upper abdominal pain, she is single, does not take
any medication, not smoker, does not drink alcohol or a drug user, her liver function is elevated,
what is the most appropriate test should be ordered first.
‫‏‬A. HAV IgG
‫‏‬B. HAVIgM
‫‏‬C. Hepatitis B surface antigen
‫‏‬D. anti-HCV antibodies

‫ ‏‬hild came with mass on right quadrant (not reach the medline )not mentioned?
C
‫‏‬-nephroblastoma

‫ ‏‬hat is the earliest manifestation in fracture something like that ?


w
‫‏‬-pain
‫‏‬-cold
‫‏‬-paresthesia
‫‏‬-hotness or redness
‫‏‬

30‫‏‬year old female came to the ER with abdominal pain LMP was 6 weeks ago. US shows
abundant fluid in pouch of douglas. What to do ?
‫‏‬A. Laparoscopy
‫‏‬B. Terminate pregnancy
‫‏‬C. Drain fluid

‫ ‏‬lderly male with epigastric pain that ‫‏‬started after a meal and became “tearing”, with diaphoresis (
E
vitally stable) . Next step in management
‫‏‬A. Diagnostic laparotomy
‫‏‬B. US
‫‏‬C. CT

‫ ‏‬ewborn with respiratory distress and pulses of upper and lower limb are different. What to do
N
‫‏‬A. Surgery
‫‏‬B. Prostaglandin infusion
‫‏‬C. Epinephrine infusion

‫ ‏‬TA what would you most likely see on FAST ?


R
‫‏‬A. Hollow organ perforation
‫‏‬B. Fluid in peritoneum

‫ ‏‬atient DM HTN with lower limb edema. What is causative drugs


P
‫‏‬-Ramipril
‫‏‬-Amilodarone
‫‏‬-Verapamil

32‫‏‬weeks pregnant 7 cm dilated sudden loss of 700 ml of blood. What type of hemorrhage
‫‏‬A. Intrapartum
‫‏‬B. Antepartum
‫‏‬C. Postpartum

If associated with contractions -> A


No contractions or no dilatation -> B

30‫‏‬year old endometriosis on NSAID with no symptomatic relief. Next most appropriate
‫‏‬A. Combined OCP
‫‏‬B. Laparoscopic fulguration

‫ ‏‬atient complain of headache that described as the worst headache in live


‌P
‫‌‏‬Ct done and was normal; ‫‌‏‬What next step
‫‌‏‬-MRI brain
‫‌‏‬-MR Angio
‫‌‏‬-LP
‫‌‏‬-CT

‫ ‏‬ong scenario Elderly complaining of lower limb weakness and urinary incontinence
‌L
Hyperreflexia What is the diagnostic method?
‫‌‏‬-MRI spine
‫‌‏‬-lumbar puncture
‫‌‏‬-MRI brain

Could be cauda equina syndrome

65‫‌‏‬years man presents to your clinic and looks weak , dehydrated, pale , thin and emacitaed. he
complains of anal itching , discomfort from the pas few months. On examination, you find an anal
mass that is 2 cm away from the anal verge , cauliflower like and friable. What is your most likely
diagnosis??
‫‌‏‬A- Anal Cancer
‫‌‏‬B- Rectal Cancer
‫‌‏‬C- condyloma accuminatae
‫ ‏‬rucellosis management ?
‌B
‫‌‏‬Doxycycline and clindamycin
Doxycycline and (gentamicin or streptomycin or Rifampin) should be the best Ans

‫ ‏‬an who is working in a factory and he has ‫‏‬Solid lung nodule. How you will treated?
‌M
‫‌‏‬-FNA
‫‌‏‬-True cut
‫‌‏‬-observation

‌‫‏‬Sle patient who has signs of infection, how will you treat him ?
‫‌‏‬-start antibiotic withhold adalimumab and hydroxycoline
‫‌‏‬-continue same treatment and add antibiotic
‫‌‏‬-stop one of them and give antibiotic

Case about mild spinal stenosis


What’s the mx:
1-physiotherapy
2-epidural corticosteroids
3-surgery

5 month old boy with a history of flu like symptoms 3 days ago came to the ED with respiratory
distress, subcostal retractions..etc
His oxygen saturation is 80% despite giving 100% O2 on mask
What is the most important next step in management?
-IV steroids
-Intubation and mechanical ventilation
-Oral antibiotics
- I forgot

What causes false positive BNP?


Copd
Obesity

2 y/o pt post on chemo for ALL , contacted a chicken pox pt, what to do ?
A- Acyclovir 7 days
B- Give vaccine now
C- Stop chemo
D- Don’t remember
Answer should be give IVIG
Preg with placental abruption given 4L of blood what next very low platelets
Platelets
Cryoprecipitate

If there FFP its the answer

Female with behaviour and pain symptoms before menses


Pelvic congestion syndrome
PMS
No pmdd in options

Pt with jaundice uqp fever Cbd and extra hepatic duct are dilated mx?
⁃ MRCP
⁃ ERCP

How to measure Fetus length in 38 weeks gestation ?


Femur length
Abdominal circumference
CRL

Loin pain + hematuria + hard lump on examination how to diagnose ?


CT without contrast

26 y abd. Pain 2week(i think) hiiiiiigh LFT except Alp what will help u dx?
Hep A igG
Hep A igM
Hep B ab
Hep c ag

56y with Appendicitis treated with abx and resolved what next?
Colonoscopy after 6 weeks
Appendectomy after 12 weeks

Depend on age

SCD which inx will help you the most?


Liver bx
Bone marrow
Retic. Count

Electrophoresis is the best


Case of 10*15 biloma after cholecystectomy what to do ?
(imaging guided drainage)

Case with many signs of bowel obs. After cholecystectomy i think and they found “pneumobilia”
what is the cause?
Dx: Gallstone ileus

How to prevent neonatorum toxicum


- give vaccine to mother after 72 hour delivery
-give neonate human toxoid injection
-give infant anti toxoid tetanus
-Give mother during pregnancy

Clonzapine use for childhood with ;


Bipolar
Schizophrenia
Major depression

Case of ankylosing spondylitis , how to confirm diagnosis ?


HLA-B27
MRI Spine

First xray if negative do HLA B27 last is MRI

Child with tonsillitis , develop rash after antibiotic. Dx ?


Infectious mononucleosis

SLE malar rash, joint pain:


- HCL + AZP
- HCL + MTX
- HCL + CYC
-HCL + MMF

Businessman travelling, with oral candida, normal neutrophil high lymphocyte diagnosis?
- HIV
- Toxoplasmosis
- Brucellosis

63) Twin pregnancy, IDA, Placenta previa. What is the indication for IV IDA?
A- Constipation
B- Moderate to severe anemia
C - Placenta previa
D - Twin gestation.
butcher cut his finger by knife and comes after few days ( developed read streaks from the
wound) What's the causative organism ?
A pseudomonas
B streptococcus pyogen
C brucella
D klebsiella

Female had stress incontinence what you will find on P/E


Urethral hypermotility
Rectocele
Paraurethral defect something like that

HTN on hydrochlorothiazide has sun burn after apply sun protect, step after advising not to
exposed to sun and avoid midday sun:
1- Warm shower two times daily
2- Stop hyrochlorothiazidand monitor blood pressure

A patient with sudden severe epigastric pain diffuse guarding and sluggish bowel sound high
amylase next step?
A. Erect chest X-ray.
B. Abdominal X-ray.
C. CT abdomen.

Suspect perforation

42 YO mother presented with periumbilical mass, On exam you found the mass tender,
irreducible, and negative cough impulse. What is the appropriate next step ?
- Diagnostic Laparoscopy
- Abdominal Us
- Open repair
- Biopsy

After hysterectomy urine out of the vagina:


1- uretrovaginal fistula
2- urethrovaginal fistula
3- vesiclovaginal fistula

during = urethrovaginal fistula


Continuous leak = vesicovaginal fistula

Baby barking cough especially when agitated inspiratory wheeze


Tracheomalacia
Laryngomalacia
Bronchiolitis

- 6 days old infant , had sob , dyspnea poor feeding , his mother was diabetic . Dx?
A- ARD
B- bronchitis

DM with cardiac disease, your goal in management?


A-triglyceride less than 150
B-BP less than 140/90
C- LDL less than 70
D - HDL more than 70

Pt with crohn, which of the following increases risk of malignancy


a. Lesion located within 15 cm from the anal verge are
B.Primary sclerosing cholangitis
c. duration of 3 years

Highest risk factor for BPH


Age

Most considerable RF for MI


Smoke
HTN

Most important/significant is DM, if its not there choose smoking


Most common is HTN

Fetus 180 HR , early deceleration


And other information, and ask about which one indicate severity I think:
HR
early deceleration

207.Natural history stages when best to do screening ?


A. pre-disease stage
B. Preclinical stage
C. Pre-clinical disease stage
D. Pre-recovery, disability, death stage

30 something Female had her 1st menarche at 15 came with irregular cycle fsh lh were high dx
Ovarian insufficiency
Pcos

Pt with had black neck pigmentation with irregular cycle now hairstism cause of the pigmentation
Acanthosis nigricans

Pt post MVA abd destination and tender jvp was raised and hypotensive distant heart sound
Type of shock
Cardiogenic
Hypovolemic
Neurogenic
Septic
This seems like Obstructive shock (Cardiac Compressive shock)
Infected bedsore mx
Debridement with vac closure

Patient presented with Hematemsis medical free, no hx of alcohol use , no hx of medications :


A- Acute pancreatitis
B-Chronic pancreatitis
C-Esophageal varices
D- Pancreatic pseudocyst

Post thyroid surgery nurse notes spasm of the arm what to check
Ca

Postop 6 days with nausea vomiting and abdominal distention and cant pass stool
Labs show hypokalemia
What investigation you need to order
A) urine analysis
B)ECG
C)urine K
D) Stool k

Man whats to travel and seeking travelers diarrhea treatment labs were given showed abnormal
KFT what treatment to give ?
A. Fluco....
B. Bismuth
C. No need
D. Probiotics

Sle how to confirm renal involvement


Renal biopsy

Male patient had an oil dissolvent injury to his index finger. On P/E mild tenderness when moving
the hands and no other abnormalities, what is your next step?
A) Oral antibiotic
B) Oral antibiotics and steroid
C) Elevate the hand and ice packing
D) Surgical management

‫ جا سوال عن‬sepsis ‫ وكان‬lactate ‫ تقريبا‬٤ ‫فرق النورمال بشي بسيط كان‬


Sepsis
Septic shock
Sirs

Child presented with ulcers on mouth and gingival erythematous base and pale in the center. Dx?
A. Coxaci
B .EBV
C Herpes
Patient has high SAAG >1.1 what's the diagnosis :
A-Pancreatitis
B-TB
C-Cirrhosis
D-Nephrotic syndrome

4 hour prolonged ERCP resulted in surgical emphysema in abdomen, chest and neck where is the
source of injury ?
-Trachea
-duodenal
-esophagus
-if remember

After tibial fracture, the patient presented with signs and symptoms of compartment syndrome.
Which one of the following is the earliest alarming sign?

Pain on passive stretch


Paresthesia
Diminished pulse

24y/o male presented with *diarrhea and fatigue* After chemotherapy what electrolytes
abnormality would you suspect?
A- hypokalemia
B- hypocalcemia
C- hyponatremia
D- hypomagnesemia

Pediatrics patient with abnormal movement, tea coloured urine, edema around eyelid, HTN, low
C3, mother mentioned that child has preceding sling infection
1-PSGN
2-HUS
3-IgA

Q- Female pregnant beyond 20th week of gestation and complain about absent fetal kicks in her
abdomen. Ultrasound showed that the fetus died at approximately the 18th weeks of gestation, the
question was about the type of this condition:
A- Missed abortion
B- incomplete abortion
C- Intra uterine fetal demise

55 years old female had bilateral green discharge, Physical examination was normal, they did
mammogram ductal …… (I think ductal ectasia?)
mammogram finding: no suspicious malignancy, what’s the management?
-Breast MRI
-Biopsy
-follow up (I think 6 month)
-Galactogram
What will you screen for a newborn?
-hearing
-vision
-vision and hearing
- depend in clinical settings

1 year child has HIV, Low CD4 count, which vaccine is contraindicated?
- OPV
- Varicella
- MMR
- Dtap

HIV patient can get all type of vaccine if CD4 >200 except BCG and OPV

child have dry non purulent conjunctivitis, cracked red lips, erythema (I think trunk, sole, hand)
what’s the diagnosis?
- Kawasaki disease
- Rubella
- Measles

child had meningitis, and he contacted his brother & sister, what will give as prophylaxis ?
- penicillin + rifampicin 2 times
- penicillin
- Ciprofloxacin one dose
- Ceftriaxone 3 doses IM

Patient had thyroidectomy (or neck surgery not sure) after surgery he start to suffocate, he get
intubated, then examined Finding: midline laryngeal fold What’s the management?
- Nerve repair
- tracheostomy

Lady 32 years old, suspect lump in her breast, what’s the investigation?
Mammogram
Ultrasound

Less than 30 -> US


30-40 -> Mammogram
More than 40 -> US + complimentary Us

Man with mania symptoms (buy cars with his saving money & sleep less)
What’ll give him?
- Lithium
- Olanzapine
child on leukotriene only, had multiple asthma exacerbation after exercise, admit to hospital and
he was controlled by albuterol what will you give for long-term treatment?

- Oral corticosteroid + SABA


- LABA + SABA as needed
- leukotriene + SABA
- 2 daily ICS + SABA

3 hours baby had a seizure for seconds and resolve immediately


Labs finding:
RBC: 4 (low)
HB: 210 (high) (N: up to 190)
Glucose: 2.4(low) (N: start from 2.8)
All other labs were normal
How will you treat?
- IV diazepam
- Glucose bolus
- IV fluid

Newly married women come to gyne dr for screening and check..which of the following is highly
diagnostic?
A-vaginal inspection
B-general appearance
C-Vaginal US
D-pelvic digital examination

Paediatric pt overweight she have high blood pressure and family history of HTN
What is the cause of high blood pressure? ICQ
Primary hypertension
pheochromocytoma

HTN in young age -> think of secondary causes

33yo male with painless swelling in the posterior triangle of his neck.
On US thyroid is normal and the swelling has a cystic component. FNA shows complete follicular
cells.
What is the most likely diagnosis?
Apparent thyroid
Metastatic cancer
Ectopic thyroid
Thyroglossal cyst

20 something multiparous in the 2nd stage of labor History of herpes and asthma and mitral
stenosis Which of the following indicate the use of forceps? ICQ
- mitral stenosis
- prolonged labor
- i dont remember the rest

Both A and B

Mid axillary thoracostomy site?


5 IC
6 IC
between 6&7
Between 8&10

Pregnant lady heavy smoker 20 packs , not taking folic or any supplements present with abruption
, what is the factor?
Low folic diet
Low carotene diet
Smoking
No risk factor

A 35 year old man presents to the emergency room with a three day history
of fever, cough and dyspnea. He is awake and alert. Despite receiving appropriate antibiotic and a
bolus of 500 ML Normal saline solution, he remains hypertensive duck Central venous pressure
measured through on internal jugular catheter is 4 cm H2O (see image). H heart rate 126 per
minute, RR 24/min,, temperature 38.6 oxygen saturation 91% which of the following is the most
appropriate management?
A. Dopamine infusion
B. 250 ML 5% albumin
C. 500 ML NS
D. Norepinephrine infusion

Biggest risk factor for DM 2?


A. Central obesity
B. High body mass index
c. Pregnancy

a 50 year old female patient presented to the general surgery clinic to


evaluate a chronic abdominal swelling big nosed later as a hernia. She’s scheduled to have the
operation in the next two days. Her history is significant for atrial fibrillation treated with warfarin
and her INR is 3.5. Which of the following is true?
A. Vitamin K
B. FFP
C. Platelet transfusion
D. No intervention needed

Old patient smoker 10 packs Has respiratory symptoms didn’t mention weight loss has clubbing:
A-COPD
B-Lung cancer

Trauma patient with emphysema Injury level one (mentioned)


Bp stable
What to do next :
A-Embolization
B-CT

Depend if its extensive emphysema then its Hard sign -> Cervical exploration
If its subcutaneous then its soft sign -> CTA

male had MVA accident, unconscious in the ER with open wound in the anterior neck
Vitally unstable Whats the appropriate management ?
Oxygen mask
Cricothyroidotomy
Tracheostomy
Endotracheal intubation

~7 years child , systolic murmur heard in lower sternal area with no thrill, ( more noticeable when
supine)
ASD
VSD
AORTIC STENOSIS
STILL MURMUR

Case of child acute pancreatitis ( dx not mentioned) vitality stable , increased WBSc.
what’s the most appropriate next management:
A. Npo , fluid resuscitation, analgesia
B. Start septic work up.

Middle aged,Medically surgically free , came with epigastric pain and abd. Distended,
Examination: tympanic abdomen + tenderness.
Radiology : pneumobilia ( exactly)
Diagnosis?
A. Small bowel volvulus.
B. gallstone ileus.

Appendicular abscess ( 10x12 cm ) reaching flank ?


Percutaneous drainage
Open app.
Lap app.
Open drainage

What is the most appropriate alternative treatment for severe depression ?


Electroencephalography therapy
Electroconvulsive
TCA
psychotherapy

Primi 22wk GA cervix is 1.5 cm dilated fetal membranes herniating manegment fetal viable (i think
there was no contractions I forgot sorry )
Tocolytic
Progesterone
I forgot the rest

Less than 24w and cervix less than 2.5cm -> Cervical cerclage

Scenario of pt with PUD syx and urea breath test was +ve and started treatment
What is the most appropriate way to monitor response to the therapy ?
Endoscopy and biopsy
Urea breath test
Clinical symptoms
Blood serology

Infant with continuous Machinery murmur


Came with symptoms of heart failure chest x- ray shows wet lungs
What will you do first ?
A) give diuretics
B) prostaglandin inhibitors

‎‫واحد عنده انيوريا وش اول شيء تسويه؟‬


Iv fluid
Abg
Ct

‫ وحدة طاح ع رجلها شيء جات المستشفى وعندها ال‬capillary refill ‫نازله وش اول شيء تسويه‬
Cta
Duplex
Compartment measures

‫ حق‬pms ‫ ايام عندها‬٨ anxiety + mastalgia

‫ سؤال واحد بعد عملية جاته‬septic shock syndrome ‫وش ممكن يصير ؟‬
Hypokalemia
Res acidosis
Anaerobic cell division ‫او شيء كذا‬

30‫وجدة جاها اجهاض ب‬w ‫ وش اهم شيء تسويه‬١٨ ‫قبل وخايفة والحين باالسبوع‬
Pelvic exam
Cervical length measure

‫ وحدة عندها‬placenta previa ‫وش اول شيء تسويه ؟‬


Us
Bhcg
Pelvic exam

‫وحدة حامل وما تعرف باي شهر ؟؟‬


CRL
Bhcg quantitive

‎‫ واحد كأن عنده‬hemolysis ‫ ومو محدد ان عنده‬iron def or not ‫ وعنده‬spherocytosis ‫ وش افضل‬test ‫؟‬
Prehral blood film
Hb electrophoresis
Best is Osmotic Fragility in spherocytosis

Elderly patient post abdominal aortic aneurysm repair by 8 hr found that urine output 50 cc
What to do
A-urine catheter
B- hydration
C- dialysis
D- ‫ في ما معناه انك تحسب‬urine out put and electrolyte

They suspect hypovolemic AKI what you’ll see ?


A-BUN/Cr >20
B-FENA >2%

27 year old patient 3 days post sleeve gastrectomy, he is presented with mild RUQ pain,upon
examination there was mild tenderness with no signs of peritonitis vitals: BP:100/80 HR:133 what
is the cause?
A- gastric leakage
B- sepsis
C- decreased oral hydration
D- inadequate analgesia
Thawaba said D

a 68 year old patient was recently diagnosed with hepatocellular carcinoma .. What's the next
step?
A-colonoscopy
B- chest CT
C - CT liver
D - Surgery

Child get genatalia burn 3rd degree , 1% of body surface:


Next
A. Refer to burn unit
B. Sterile abx ointment
C. Dressing
D. Admission

‫جاني سوالين‬
‫ سناريو طويل وتعالج على انه‬croup ‫ ومكتوب بالحرف انه اخذ العالجات حقت ال‬croup ‫المناسبه وال تحسن وش تعطي‬
A- antibiotics
B- predenesoln ‫مع دوز‬
C- dexamtheasome ‫مع دوز‬
D- racemic epinephrine

‫والسوال الي بعده نفس السناريو قال وش تطلب له‬


A- lateral neck x-ray
B- chest inspiration and expiration x ray
C- visualize by laryngoscopy
D- chest ct

A female presented with back pain and fever, she was given NSAIDs and advised for bed rest at
home, then she presented with inability to move her lower limbs, O/E there was tenderness on the
midback.
Spinal MRI: Diffuse densities on T6,
What is the most appropriate next step?
A- Reassurance
B- Specimen from T6 vertebra
C- Brucella titer
D- Bone marrow aspiration for culture

Elderly, had chronic productive cough, whitish that end with blood streaks:
Diagnosis?
-copd
- goodpasture syndrome
-hereditary telangiectasia
16 yo male CO 4 years altered bowel habit he has 3-4x a day diarrhea that is foul smelling
containing undigested food, no blood or mucus He has abdominal distention and after meals(no
mention of the type of the food ). No secondary sexual characteristics
What investigation?
A. Stool fat test
B. Barium
C. Hydrogen breath test
D. Anti- something test
Most likely Celiac ?

20- most common cause of IE organism:


A. Enterococcus species
B. Streptococcus species
C. S. Aurus

Prophylaxis for Pt with prosthetic valve want to do dental procedure


A. Ampicillin

2 Cases about ttt of h. Pylori


Triple by PPi + amoxacillin or metronidazole + Clarithromycin
Quadrupe PPi + bismuth + tetracycline + metronidazole

Reasses after 2w if take ppi only , and after 4w if taken abx

526-Child got pertussis infection, has two siblings 3 and 5 yrs, both up to date in vaccination
A- Give them both booster vaccine
B- Chemoprophylaxis by Marcolide
C- Give booster only if high risk
D- Observation for signs and symptoms of infection ‫مكرر‬

•High diagnostic value for Ovulation:


-FSH
-Progesterone at day 21

Case of a patient “presented with several episodes of hematemesis, he has no history of previous
episodes”. He is not alcoholic, not on any medication, and has no stigmata of liver disease. What
is the diagnosis?
A- PUD
B- Mallory weiss
C- Esophageal varices
Not Clear scenario, if Q as is then A because PUD is the MC cause of UGIB

•A case of lung cancer -> Lung nodule 8 mm, revealed to be squamous cell carcinoma, what is the
most important to do while preparing for surgery:
-pulmonary function test
-Radiotherapy
-Chemotherapy

A young boy case came with jaundice, he’s mediterranean It makes you think it’s mediterranean
fever but I think it was G6PD because he had high reticulocytes What drug not to give
1- amoxicillin
2- aspirin

Obese wt 120 ht (not sure) around 174 he tried diet and exercise but didn’t work
1-Bariatric sux
2-Wt loss medication

Most Common organism in IE in Prosthetic valve less than 60 days


1-Staph epidermidis
2-staph aureus
3-strep viridans

Pancreatic pseudocyst with early satiety and fullness after observation for 3 weeks increases in
size , what is the next step?
- observation
- percutaneous drainage
- Internal drainage
- excision

Child with hypertelorism and other manifestation, what you will ordr?
a. TORCH screening
b. Genetic screening
c. Metabolic screening

Pt 25y asthma , chronic sinusitis not controlled


What to add ? ICQ
Montelukast
Budesonide

Man with tibial fracture i think managed in the ER with cast, then he developed swelling in the
toes. (Clear scenario about compartment syndrome)
What is the EARLIEST symptom?
- pain
- paresthesia
- cold extremities
- couldn't remember

43 yo female hx intermittent bleeding for 6 months , come for contraception what the most
appropriate investigation?

-TSH
- trans vaginal US
- FSH

Pt on antidepressant then developed constipation, what class of antidepressants?


-Ssri
-TCA
-SNRI
-MAOI

Elderly, 89-90 yo lives in (‫ )اتوقع دار رعاية‬has a abdominal pain for 24 hrs, on examination there’s
diffuse tenderness and he’s vitally stable
What’s the appropriate next step
-abdominal US
-sigmoidoscopy
-colonoscopy
-last choice I can’t remember if it was laparotomy or something else

The percentage by which smoking increases the risk of MI?


A. 2
B. 3
C. 4
D. 5

Q7; eldery smoker k/c of poorly controlled DM comes with ulcers on tip of three of his toes,
diminished dorsalis pedis bilaterally, however, intact popliteal pulse, what’s the initial
management;
Amputation
Long term anticoagulation
Immediate surgical intervention
Diet modification and lifestyle changes

Q6;eldery female (60s) with rectal bleeding, diagnosed with grade 3 hemorrhoids at 9 and 3
o’clock, what’s next;
Rubber band
Hemorrhoidectomy
Colonoscopy

How to confirm Dx of PROM;


Speculum
US
Chemical

Young male previously healthy presented with fatigue, body ache, cough with whitish sputum and
sometimes associated with blood. Dx?
A- Bronchitis
B- pulmonary edema
C- hereditary hemorrhagic telangiectasia
D- ?

19 years old patient complains of headache, menstrual pain Starting with period and lasts for 2-3
day. It’s progressive over the years. What’s the Diagnosis?
A- primary dysmenorrhea
B-secondary dysmenorrhea
C-premenstrual syndrome

After ileal resection, which element to be delivered parenterally?


A. Iron
B. Zinc
C. Bile salts
D. Vit B12

A child presented with acute exacerbation of asthma, he was given SABA, Inhaled corticosteroids
and magnesium sulphate, he was given 100 ٪‫؜‬oxygen but the the oxygen saturation still 80. What
is the most important next step ?

1- add ipratropium
2-add aminophylline
3- add antibiotic ( not sure)
4- intubation and mechanical ventilation

a Pregnant at 38 weeks gestation presented with 24 hours spontaneous rupture of membrane.


She has no pain and on examination the uterus is soft and no evidence of contractions. CTG was
reassuring. What is the best next step?
1- C/S
2- observation
3- induction of labour
4- corticosteroids

Female patient has menorrhagia . On pelvis US it showed a Mass. where is the lesion( exactly like
this short question )
A- intramural
B- subserosal
C- submucosal
E- cervical

Old patient known case of A fib presented with mild diffuse abdominal pain, hemodynamically
stable. What is the best next?
A-US
B-CT ( like this)
C- diagnostic laparoscopy
D- explorAtory laparoscopy

No colonoscopy in answers

Infant with inguinal hernia, not obstructed, strangulated or irreducible, the testes at the ipsilateral
side was slightly elevated

Herniotomy
Hernia repair with mesh
Wait until 6 years
Women with a history of salpingostomy for ectopic. Came for (something I forgot). On PE there is
a protruding mass from the cervix.
Her hcg serial measurement “was decreasing in first two days then start to increase from the third
day to the seventh” ICQ

What to do NEXT:
-Immediate MTX
-Bx
-Work-up for metastasis without bx

55 year female in ICU post hartmann procedure on the 2ed day post op, has 20ml/d~ urine output
Central line pressure 10mm. Mx?
A. re explore abdomen
B. CT with contrast abdomen
C. Duplex us renal
D. MRI abdomen or us abdomen not sure

32 post CS 16 days , presenting with fever not responding to antipyretic and Abx, previous
multiple Cs with significant adhesion. Ct shows small bowel fistula l, which is appropriate next?
A.NPO and start pancreatic and gastric secretion inhibitors
B. resect and anastomose
C.restrict to soft diet
D.colostomy

Hx of sleeve bypass present with Severe pain and mild hypotension


Next step?
A.Endoscopy
B.Us
C.Laparotomy
D.Diagnostic laparoscopy

Patient on oxytocin, epidural and MgS04, preeclampsia. Her CTG: absence variability (or
non-reactive). What's the cause?
A. MgS04 toxicity
B. Oxytocin
C. Epiduralanalgesia

Preeclampsia: what is expected to significantly decrease ?


1- Urea
2- Creatinine
3- Plasma volume
4- Platelets

30, Female, have hemorrhage pulmonary, glomerulonephritis, sinusitis+…… (i forget)?


⁃ Giant Cell Arteritis
⁃ polyartrites nodosa
⁃?
⁃?
Ans Should be Granulmoatosis with polyangitis ‘ wegner granulomatosis ‘

Pregnant, Smoker, planning to quit, which will affect the baby?


⁃ Low birth weight
⁃ TTN
⁃ Neonatal hyponatremia
⁃?

Young women, k/c of SLE, asymptomatic, came for follow up, vaccine?
⁃ influenza
⁃ BCG
⁃ varicella
⁃ OPV

Elderly was completely healthy except for elevated BP for the first time
What's your next step?
A- Ambulatory BP measurement
B- Start anti HTN
C- Measure the BP two times later on in the clinic
D- Measure BP two time in home

Pt presented with inguinal lump since 3 years , positive cough impulse + positive inguinal
occlusion he is asymptomatic what to do :
-herniotomy
-observation
-herniorrhaphy
-mesh repair

Anal examination shows 3 and 9 oclock fistula, what next


colonoscopy
reasunce

38 w baby delivery on non complicated c/s complain of tachypnea and grunting, Dx


meconium aspirated
bronchopulmonary dysplasia
transient tachypnea of newborn

Neck swelling showed enlarged lymph nodes with follicular thyroid cells, thyroid was normal.
What to do
reassures
thyroid surgery

Child 3 years, with signs of Cushing syndrome What is the highest predictive value for diagnosis
.A. Renal US
.B. Height-weight ratio
C. ACTH stimulation test

Child with flu-like symptoms developed vesicular rash all over the body confirm diagnosis
A. Herpes simplex virus-1 IgM
B. Herpes simplex virus-2 IgM
C. Mumps IgM
D. Varicella-zoster virus IgM

DM case with recurrent infections, left upper cavitation on CXR, what is your precaution
A. Airborne
B. droplet

Post MI admitted to ICU 2 days after BP 88/60 in shock first MMx


A. Dobutamine
B. IV fluids

A Pregnant 28 weeks GA known case of hyperthyroidism on 15mg carbimazole came with Sx of


hyperthyroidism, labs: low TSH high t4, what's the most appropriate management
A.switch to PTU
B.increase carbi dose to 20mg
C.referral for thyroidectomy
D. RAI

Pregnant everything normal she did urine test : positive e.coli and she is sensitive to nitro and
cipro and Trimethoprim / Sulfamethoxazole Management
oral nitro
oral cipro
oral Trimethoprim / Sulfamethoxazole
no need treatment

10 days baby with respiratory distress ,lethargy Absent femoral reflux Next management
surgical repair
balloon dilation (something like that)
prostaglandin

33years old women with heavy menstruation During examination noticed firm fundal mass Next in
management
CT
US
MRI
biopsy
Treatment of brucellosis with neurological symptoms
A. 3 weeks
B. 6 weeks
C. 4 months
D. 6 months

Cough & fever after antibiotics he became better but symptoms never gone What’s
the most likely diagnosis
A. PE
B. Parapneumonic effusion
C. HF
D. TB

Patient with HTN not responding for three anti hypertensive drugs on Ultrasound the one kidney
is bigger what is diagnosis
A Adrenal hyperplasia
B Renal artery stenosis
C Pheochromocytoma

SLE pt pregnant on treatment azathioprine and immunosuppressive drug newborn vaccinations


a- Give all
b- Delay after 6 months
c- Delay after 12 months
d- All live vaccinations are contraindicated

Depend in which trimester did she started to take the medications , if second then delay live only for 6m ,
if third then delay live only for 12m

Patient with hx of pancreatitis months ago and now referred to surgery clinic for mild
abdominalpain and , what initial inves? I think it was pseudocyst
A-U/S
B- CT
C- endoscopy
Initial is US but confirmatory is CT

Question about Meningitis, a 7-year-old patient infected with a gram-positive diplococci.


Treatment is
A.Ceftriaxone
B.Penicillin + gentamicin
C.Gentamicin
D.Gentamicin plus vancomycin

Adult patient with inguinal hernia underwent hernia repair with mesh, came now complaining of
pain extending to the thigh associated with numbness and paresthesia, what to do
A. Remove the mesh
B. Remove mesh with neurectomy
C. Nerve block
D. Anti-inflammatory
5 years old boy with mid-shaft femur fracture, angulation 25 degree. What is the Best
management
A. Closed reduction & hip spica
B. Open reduction & plate
C. Open reduction & IMN

Case of seatbelt injury in RTA which part is affected


A. Gastric laceration
B. Duodenal perforation
C. Jejunal
If no Chance fx then C
If with Chance fx then B

A post-menopausal woman comes to you with chronic urge incontinence symptoms (~6 months
of urinating before entering the bathroom. What is the “most appropriate next best step
a. Urethral pressure profile
b. Urine culture
c. Urethrocystoscopy
d. Renal Ultrasound

:What is the cause of poor wound healing in controlled DM


A) Peripheral neuropathy
B) Poor blood supply
C) infection
D) Callus

knee trauma on the lateral side and swelling on medial side. McMurray and Lachman test are both
negative. which ligament is injured
A. medial meniscus
B. lateral meniscus
C. medial collateral
D. lateral collateral

A child was brought with history of jelly-like stool. The patient was stable and not obstructed,
next management?
A- NGT and keep NPO
B- Warm enema under US
C-laparotomy excision
D-laparotomy reduction

What age nocturnal enuresis concerns you


A. 5
B. 6
C. 7
D. 8
50 years man with chest pain radiating to the neck with exercise. Ecg is normal. Best modality to
rule out ischemia
Cardiac CT
Echo
Creatine kinase enzyme
Stress Echo is the best but if Echo Vs Cardiac CT angiography then choose CTA ?

Case of TIA (left sided body weakness and left face weakness) what is the next step in
management
Aspirin
Head CT
Brain MRI

Female 47 years old she has iron def anemia w 3rd degree hemorrhoid what would u do
Colonoscopy
Hemorrhoidectomy
CT scan

A pregnant lady complains of vaginal bleeding, she's a confirmed case of low lying placenta, no
active bleeding now and no pain. Next step in management
Ultrasound
Biophysical profile

Confirmed case so it should be CTG

Scenario of Endometriosis with persistent symptoms even with NSAIDs, best next step in
management?
Ocp
laser ablation

What is the most effective reversible contraception


hormonal vaginal ring
male condom
female condom

Dysmenorrhea, how to best diagnose endometriosis?


laparoscopically
US
MRI

Pregnant lady 39W her routine BP throughout the pregnance was 120/80 (normal) then suddenly
became 150/90 what is the diagnosis
eclampsia
Gestational hypertension
Chronic hypertension
Superimposed hypertension
30 years old post partum woman admitted for right leg DVT and was started on enoxparin 80mg
BID. Then she developed sudden onset dyspnea and right pleuritic chest pain, on PE; She was
dyspneic and apprehensive, heart sound showed loud P2 and lungs were clear on auscultation,
vital signs normal ABG: normal HCO3 and Po2, decreased PCO2 and high pH CT showed
thrombus in right lower pulmonary artery
Which of the following is most appropriate step in management
A. Switch Enoxparin to sodium heparin
B. Thrombolytic therapy
C. Same management
D. Thromboectomy

48 y.o very obese lady, BMI 41 had pelvic organ prolapse many times and then had
surgery, what do you except she’s having now
Enterocele
Rectocele
Uterine prolapse
Cystocele
Vesicocele

57 y/o female complaining of abnormal uterine bleeding she has an endometrial


polyp, on US endometrial lining was 19mm, what will you offer to this patient at this
stage
A) open hysterectomy
B) laparoscopic hysterectomy
C) hysteroscopy with polypectomy

pregnant suddenly at her mid second trimester came to ER complaining of gush of


fluid + tissue passed out
complete aburtion
Cervical incompetence
Incomplete aburtion
Threatened aburtion

Woman schedules for CS, she reports having a “difficult CS” in her last pregnancy
How to prevent adhesions
close the peritonium
Add a layer the incision site
Do CS preterm
A barrier consisting of oxidized regenerated cellulose

46 years old female smoker for 18 years concern about cancer , appropriate
screening for her
Mammogram
Colonoscopy
HRCT
All of the above

Patient post op complaining for 1 week intermittent fever on examination abdomen


and chest normal and the wound is clean
What's the most appropriate step
A) reassure
B) ct abdominal
C) x ray
D) exploration

Smoking cessation can decrease the risk of MI after


A. 3 years
B. 1 year
C. 2 years
D. 18 months

Old pt with Retroperitoneal hemorrhage with history hodgkin lymphoma what’s the dx
A. Lymphosarcoma
B. Liposarcoma
C. Sarcoma

40 years Pt with hx of bloody discharge from breast ? P/E normal Initial invest
A. US
B. Mammogram
C. MRI

49 year old man heavy smoker , otherwise he’s healthy came for lung cancer screen what do you
do for him
low dose CT
sputum cytology
chest x ray
maybe something biopsy

years old female presented to gynecology clinic complaining of dysuria and urinary 54
incontinence, she stated that the leak is not related to specific activities and occasionally
associated with cough Upon investigation urinalysis and cultures were insignificant, However,
urodynamic study showed: contracting bladder even with small amount of dripping Which of the
following is the most appropriate management
A. Kegel exercise
B. Anterior Colporrhaphy
C. Anticholinergic agent
D. Bladder Suspension

baby born on 35th weeks of gestation When to give him vaccine


A. At discharge
B. 2 months after the estimated date of delivery

Appendicular abscess reaching right flank


Percutaneous drainage
.Open app
.Lap app

Same bilateral cataract case in common. With positive urine substance. Which of the following
has the HIGHEST DIAGNOSTIC VALUE
A- HIDA scan
B- Abdominal US
C- Alpha 1 anti trypsin
D- Urine reducing substance

15years old case of meningitis LP to be done what to do


A. Take consent form parents and assent from patient
B. take consent from parents and donʼt assent patient
C. no need for consent
D. consent patient only

A patient coming from an endemic area for malaria. On Quinine(or


Quinidine?!). Later on, he was admitted to the hospital with Heart
Failure. Doctor did not ask the patient about travel or medication hx. Thus, he
.ordered the nurse to give digoxin. The patient went into deterioration
How could it be prevented
.A. Computerized drug prescription system
.B. Nurse failed to write drug formularies
.C. Pharmacist who failed to check prescription
D. Patient who did not bring his medicine pack/ report his medicines

Ectopic pregnancy to with salpingostomy how to follow up


weekly HCG

If salpingECTOMY then only once

Pregnant with suspicious of cervical lesion what to do next


A- Cone biopsy
B- Cur Rita get
C- Colposcopy
D- Pap smear

Female 27 year old, did pap 3 years ago was normal what next
Repeat pap w cytology
No need

Breast hard nodule with tirated skin Dx ICQ


Ductal ectasia

45 years old male or female, has a mass 5 cm in the right upper limb, (MRI shows
a mass from triceps) what to do next
A- Excisional biopsy
B- Core needle biopsy
C- Imaging

Basal skull fracture in jugular foramen what is affected


A- Masticashion
B- Ipsilateral vocal cord

Male with lung nodule not change 1,2,3 years what next
Follow up 1 year
Follow up 6
No further follow up needed

Patient with migraine headaches what medication is used for acute attacks
A- Triptans
B- NSAIDs
C- SSRIs

Child with respiratory symptoms BP 160/99, HR 160, retic: 12 dx


A. Sickle cell anemia
B. Polycythemia

Female patient with unilateral nipple dryness, crust and oozing discharge
bilateral breast ultrasound and mammography are normal. What is the next step
A- Nipple Biopsy
B- ABX

Patient in ER, after you finished primary survey, his BP is 90/50


A- IV fluids
B- Call surgeon in duty

CHILD WITH SLE symptoms which one is in 4 of 11 the criteria of american


A- Erythema marginatum
B- Positive rheumatoid factor
C- Hemolytic anemia and reticulocytosis

Pregnant with SLE medication steroid hydroxy and cyclosporine


Stop hydrox
Stop steroid
Stop cyclosporine and give azathioprine

A patient presents to the emergency department with paralysis of all four extremities
after being involved in an automobile accident. The patients' extremities are warm
and pink. Which of the following is the most likely diagnosis
A. Hemorrhagic
B. Irreversible
C. Neurogenic
D. Cardiogenic

Female with right cervical lump Upon examination she has enlarged cervical lymph node with
normal thyroid gland. Percutaneous biopsy taken from the lump Histopathology report: follicular
thyroid cells, which of the following is the most likely name of cervical lump
.A-metastasis of thyroid gland
.B-Follicular thyroid carcinoma
.C-Papillary thyroid carcinoma
D-aberrant thyroid tissue

a female patient came complaining of venous spider nevi, no other engorged veins, she wanted to
treated cosmetically, what investigation should be done next
A) CT venography
B) vein plethysmography
C) venous duplex

Other smiliar recalls

308-Woman complaining of spider veins on her hips for cosmetic reasons, no symptoms and no
other varicoses, How is the ddx established?

A- Compression ultrasound
B- Venogram
C- Clinical diagnosis no need more investigation
Answer: A

Female 30s asking for cosmetic treatment of varicosity in her thigh, asymptomatic, no other
varicosity, how would u investigate?
A. Duplex
B. CT venography
C. No need for further investigation
Answer is A

UTD⎯ The diagnosis of chronic venous disease is suggested by the presence of typical symptoms
(leg pain, fatigue, heaviness) and physical examination findings. Venous duplex ultrasound
examination confirms the diagnosis demonstrating the presence of venous reflux.
The majority of symptomatic patients should undergo venous duplex ultrasonography to evaluate
the nature and extent of venous reflux, which impacts the choice of treatment..

Child complain from fever and sore throat, had contact with child has flu-like illness , in
examination only hyperaemia of throat , Rapid antigen is negative and swab culture was taken.
What you will do
A- supportive treatment
)B- antibiotics pending culture results ( written like this exactly
.C- Single dose IM ceftriaxone
D- Start ampicillin 10 days regardless

This patient centor criteria is :


Child -> 1
Fever -> 1
Hyperemeic throat -> 1
So 3

The role of antihypertensive in pre-eclampsia?


A- To prevent maternal complication as stroke
B- prevent IUGR
C- prevent fetus demise

Pt have Rt testes swelling for 6 months , in the last 4 months it increase in size , in examination it
was separable from the testes and doesn’t reach the inguinal
A - spermatocele
B- epididymal cyst
C - indirect hernia

Risk factor for preeclampsia


A) Maternal age
B) Gestational age
C) Multiple pregnancy

Female had rapid glomerulonephritis + pulmonary hemorrhage + recurrent sinusitis + numbness?


-Giant cell arteritis.
-polymyalgia rheumatica.
-Granulomatosis with polyangiitis (Wegners)

Patient has a rash on buttoks and lower limbs. What is the commonest joint to be involved?
A- Iliosacral
B- Hip
C- Knee and ankle
D- Small joints of the foot

Patient known case of CKD for the past 3 years presented with history of numbness in both hands
and in examination sensation was absent below knee level and absent ankle reflex, lab findings
suggestive of iron deficiency anemia and high renal function test and mild hyperkalemia, What the
next appropriate next step in management?
A- Erythropoietin
B- Vitamin B complex
C- Dialysis

55 year old female Patient diagnosed with MI and was treated at the hospital, upon her discharge
she asked you what is the best Way prevent reinfarction?
A- Use of aspirin for short duration.
B- Use of Calcium channel blocker indefinitely.
C- Use of ACEI if she develops heart failure to prevent cardiac changes
D- Use of spironolactone for long duration

Diabetic type 2, obese, done lifestyle modifications for 3 months (metformin was not used), Hb
A1C is 8.5. What is the medication of choice?
A- DPP Inhibitor
B- Biguanide (Metformin)
C- Sulphonylurea
D- SGLT2 Inhibitor

What’s the best management for (clear scenario of SBP) PT confused and unstable
A-Diuretic and IV Metronidazole
B-Lactulose Enema and IV cephalosporin

16- Patient came with right-sided pleural effusion and a positive history of hemoptysis. There was
no shift of the trachea nor the cardiac apex. What is the diagnosis?
A. Rupture of the esophagus
B. Heart failure
C. Tuberculosis
D. Cancer obstructing the ipsilateral bronchus

17- Girls ingest 50 tab of paracetamol with 500 mg each tab Come in (6)hrs in ER What will you do:
A- Observe
B- Antidote
C- Antidote with charcoal
18- World health organization (WHO) to determine the health of the population in KSA What
should KSA share to WHO:
A- Health determinant
B- Health indicator
C- Risk factors
D- Something variable

Post 7 cycle chemotherapy fever with high neutrophils 70% next


A- Give him NSAID and antibiotic
B- Wait for culture sensitivity
C- Give empiric Abx

Patient had RTA, presented with bruises on his chest, distended neck veins, low blood pressure,
clear lungs, What is the diagnosis?
A- Cardiac Tamponade
B- Cardiac Contusion.

14year old Female seen in clinic with fever, weight loss, fatigue, and headache, you notice
heliotrope rash What is your diagnosis?
A-Juvenile dermatomyositis
B- Rheumatoid arthritis
C- Scleroderma
D- SLE
>> Came without picture but it was mentioned in the Q (lesion on eyelid)

22- 20 years old female, medically free presented with epigastric tenderness long duration , not
related to anything, some time it associated with vomiting Her mother have chronic non specific
abdominal pain Which of the following most appropriate for diagnosis?
A- Endoscopy
B- Barium swallow
C- stool analysis

23- Child with recurrent Sinopulmonary infection and Gi infection His brother died 6 month due to
sepsis. What is the most likely diagnosis?
A- Iga deficiency
B- X-linked chronic granulomatous disease

25- Patient came with a DVT picture and diagnosis was mentioned (+ i think imaging showed
proximal dvt), he was stable Also had previous pulmonary embolism many years back. What is
your next step in management?
A- Aspirin
B- Anticoagulation for 2 months
C- Long term anticoagulation
D- Warfarin

26- A 21 years old known case of DM on insulin Labs including creatinine are normal Urinalysis ++
protein What is most appropriate management?
A- Ace inhibitors
B- Beta blocker
C- Calcium channel blocker

27- Patient presented with neck swelling, she is completely asymptomatic, neck US was done
reveal a solid mass of 7 - mm with regular margins Which of the followirg is the most appropriate
rext step?
A- FNA
B- Thyroid scirtigraphy
C- Check TSH

28- Which of the following medications, when used alone as maintenance Therapy in persistent
asthma, is associated with an increased risk of asthma-related mortality?
A- Inhaled fluticasone
B- Inhaled salmeterol
C- Oral zafirlukast
D- Oral prednisone

32- Elderly had cough with white sputum that turned yellowish with diffuse infiltrates bilaterally
and more dense in right lung . Dx ?
A. Atypical pneumonia
B. pneumococcal pneumonia
C. Viral pneumonia

33- 33 y/o was drowsy and confused . Hemoglobin 8 - MCV low - Reticulocyte 5 - Nothing about
iron. Next best management?
A- blood transfusions
B- IV iron
C-IM iron
D-oral iron

36- 50 years old man with chest pain radiating to the neck with exercise. ECG is normal. Best
modality to rule out ischemia:
A. Cardiac CT
B. ECHO
C. Creatine Kinase Enzyme
D. Stress ECHO.

37- A patient presented with symptoms of MI 2 hours ago and it’s anterolateral on ecg and PCI is
NOT available management?
A- aspirin,streptokinase, heparin and beta blockers
B- aspirin, streptokinase, nitroglycerin and beta blockers

38- P.t C/O Dysphagia of solid more than liquid what is the best initial test:
A- Barium
B- Endoscopy
C- Manometry
D- 24h pH impedance testing
40- Elderly pt with CHF on ACE, spironolactone, furosemide,admitted for some reason I forgot,
upon admission she was on fluids (?) Labs show she’s hypotensive with increased urea and
creatinine and FENa 0.6 What to do?
A. Increase furosemide.
B. Increase spironolactone
C. Increase fluids carefully

41- male patient, known case of HTN, CAD, and HF is on aspirin, beta blocker thiazide diuretic,
and statin. He is stable and controlled. What will you add to their medications?
A. ACEI
B. ARB
C. Clopidogrel
D. Aldosterone antagonist

42- Risk factor of stroke


A- HTN
B- DM.
C- hyperlipidemia.
D- obesity

Pregnant woman Rh - fetus Rh+


She gave anti D before …?
What to give now
A-Anti d 100 microgram
B-1000 microgram
C-1500 microgram
D-No need to take

10 ‫ مايكرو مو‬1000 ‫مو متاكده من الصياغه ممكن تنزلوها وتشوفوا ريكول افضل له الني متاكده مو من االسئله المكرره ومتاكده من وجود‬
‫االف‬

Patient on unfractionated heparin developed low platelets and other symptoms


heparin induced thrombocytopenia). What to do(
A-Continue heparin
B-Stop heparin and use warfarin
C-Stop heparin and use non heparin based anti coagulation

Patient with malar rash, reynaud phenomenon, joint pain of MCP joints, healed ulcer in top of 2nd
finger, proximal muscle weakness pt can’t climb the stairs, what is diagnostic AB
anti ccp
anti smith
Anti ds dna
anti RNP

Mixed connective tissue disease anti-U1 RNP


Gun shoot they remove pancreas and spleen and part of gut his vital : 90/67 come after 4-6 days
and have fever what is next step
A- CT
B- laparotomy exp
C-US
D-laparoscopic diagnosis

Pt in ER post MVA typical for pneumothorax asked what’s Next? (I dont remember his vitals but i
think they were borderline stable bp 90/sth
A. CXR
B. Needle decompression
Other options didnt seem “next” to do

Pregnant (i think 3rd trim) came due to not feeling fetal movement for 1 day. NST done and was
normal also BPD and sth else all were normal. What to do
A. Reassure and BPD in 1 week
B. Contraction stress test
C. Admit for observation (i think
D. Something about delivering

Child patient presented with respiratory symptoms, with a previous history of 5-6 respiratory
infections, 3-4 gastroenteritis with giardiasis, also there was greasy foul smelling stool, upon
examination nothing abnormal except for non-palpable lymph nodes, what is the most likely
diagnosis?
X-linked agammaglobulinemia
Severe combined immunodeficiency (SCID)
IgA deficiency
Could be B or C
LN goes with SCID and Resp + Gastro (Giardia ) go with Selective IgA Deficiency

COPD patient, with typical signs and symptoms on therapy, doing well. What is the prognostic
factor to his disease?
FEV1
FEV/FVC
Acute exacerbation per year
CT scan

Middle age male patient, complaining of Shortness of breath in night, and cough (I think), his past
history insignificant except for his working in car painting shop, also he states that the symptoms
disappear when he's out of work, and come back during work, on examination he was completely
normal, what is the best advice (I'm sure) for him?
Start corticosteroid therapy
Change his job
CT scan
X-ray
Test during working days or methacholine challenge test ?

Patient with DM, HTN, on multiple drugs, no mention of atrial fibrillation, smoker, presented to E.R
with severe left leg pain, and he has previous history of leg pain started after walking 200m, on
examination, left leg, absent of posterior tibialis, popliteal and femoral pulses, on the right sided,
intact femoral, popliteal pulse, but absence of posterior tibial pulse, what is most appropriate step
(not best, not next step)?

CT angiography
Conventional angiography
MRI angiography
Arterial duplex

It should be ASAP heparin then check neurovascular stability if stable do CTA , if not do surgery

Young patient with 3-year??? history of fatigue, abdominal bloating and mucus in the stool, (for
me? It's celiac symptoms), asking about most diagnostic tests for him?
Anti-endomysial antibody IgA
lactose breath test
esophagogram
I'm sure 100% no IgA tTG or biopsy in the choices
Confirmatory is Bx
Screening is Either anti-endomysial antibody or TTG (better)

Patient with stable angina, symptoms appears on exertion and disappear when he set on his sofa,
he is free of pain now, they asking about most diagnostic test for ischemia (I'm sure 100%):

Echo
Stress ECG
CT scan

Old patient I think, with cardiac signs and symptoms, on examination there was pansystolic
murmur crescendo-decrescendo radiated to the carotid artery (100% sure), what is the most likely
diagnosis?

Tricuspid???
Mitral regurgitation
Aortic stenosis
Mitral stenosis

Patient with non-STEMI angina, they started him on, aspirin, statin, nitrates, I think (ACE was not
started 100% sure, but I forgot about beta blockers), so what should we add?

ARBs drugs end with sartan


Beta blocker
Warfarin
Forgot it

Patient with typical signs and symptoms of COPD, PEF less than 0.7, and negative bronchodilator
test (I'm not sure about the test), also have decreased DLCO, what is the most likely diagnosis?

Bronchial asthma
Chronic bronchitis
Emphysema
Interstitial lung diseases (not 100% sure about it

Patient with interstitial pneumonitis (mentioned), is asking about what is common with this type of
inflammation or the common cause of this finding?

Lobar pneumonia
Bronchoalveolar pneumonia
Viral pneumonia
Secondary (T.B)

Patient 64 years old, with DM and HTN, no other medical history mentioned, calculate his CHADS2

1
2
3
4

Patient not more than 64, he has DM and HTN, with atrial fibrillation, what should start him on?
Clopidogrel and Aspirin
Warfarin
Heparin
Aspirin alone (not sure)

Patient 64-years old (not sure), he has DM, HTN and persistent atrial fibrillation, and heart failure
sign & symptoms (S4??), previous stroke 3 year ago with hemiparesis, what should start?
Clopidogrel
Warfarin
Heparin
Aspirin

Patient presented with Sjogren disease (mentioned), with hypokalemia, asking about type of renal
tubular acidosis?

Type 1
Type 2
Type 3
Type 4

Old female patient with typical symptoms of depression, sadness, loss of interest, feeling
worthless and suicidal thought, which one of following mediators responsible for her symptoms?

Serotonin
GABA
Glutamate (not sure)

Patient underwent ultrasound, she is postmenopausal on tamoxifen, U/S showed 2x3 cm uterine
fibroid, after 5 years she come for follow up, and new ultrasound showed, previous fibroid
increase to 4x6 cm, and also there was endometrial hyperplasia 5 mm (100% sure), so what is the
most likely diagnosis?

Leiomyoma
Leiomyosarcoma or lyeosarcoma
Endometrial adenocarcinoma (100% sure)
Secondary ovarian cancer

Obese female patient, with DM2, complaining of heavy menstrual bleeding and I think irregular
menses, she did ultrasound and showed, endometrial thickening of 18 mm (sure 100%), so what is
appropriate next step for her?
Endometrial scratching or sampling in the clinic
Hysterectomy
Cystoscopy
Endometrial adenocarcinoma (100% sure)
Secondary ovarian cancer

Child can walk, no mention of telling a story but can tell you 6-7 words (sure 100%), what is his
age?

24
19
17
12

Female patient with heavy menstrual bleeding, no bleeding between periods, and she has
abdominal pain started 2-3 days before her menstrual cycle, the pain radiated to the upper thigs
and back, pain ends after period, physical examination was completely normal, no palpable
masses or anything else. What is the most likely diagnosis?

Primary amenorrhea
Secondary amenorrhea
Endometriosis
Unrelated choice

She has premenstrual syndrome. By exclusion C

Patient with typical signs and symptoms of PCOS, the doctor order blood tests and ultrasound
but still pending, what is the most appropriate next step?

Start OCP
Start metformin
Start hormonal therapy
Lifestyle modifications, and induce exercise

Female patient, completely normal no signs or symptoms coming for regular follow up, no
compression symptoms, on examination there was thyroid enlargement only, she underwent
ultrasound examination, and it showed, diffuse (I think) enlarged thyroid, solid nodule (they didn't
mention which one), the first one was in right 0.6x0.9 mm, second one left thyroid nodule 2x3 cm,
what is the next step in her management?

Thyroid scintigraphy
FNA of both nodules
FNA of the large nodule
Follow up after 6 months (not sure)
No TSH in choices (100% sure)

Patient with typical risk factors and signs of abdominal aortic aneurism, presented with severe
tearing pain radiated to the back, on examination, there was pulsatile abdominal mass, no
previous history of AAA, his vital sign 80/60 (sure), what is the most appropriate step?
Exploratory laparotomy
Abdominal ultrasound
CT scan

Female patient coming for regular follow up, no complain, no palpable breast masses, she
underwent mammogram, radiologist report was: BIRADS 0, due to diffuse hyperintensities breast
fat (I think?) in both left and right upper quadrant?, so what is the next appropriate step?
Consider other modality to diagnose
Follow up with mammogram after 6 months
Excision of both right and left upper quadrant lesions
Unrelated choice

Child patient presented with a lot of sign and symptoms of heart disease, the cardiologist did
echocardiogram, it showed left to right shifting of blood from atrium, also there was atrial septal
defect with bulging of something??? What is most likely diagnosis?
Atrial septal defect
Patent ductus arteriosus
Atrial ventricular septal defect
Forgot it

Patient post roux-n-y surgery, developed fever, chills and abdominal pain radiates to left shoulder,
he was hypotensive 90/50 (sure 100%), what is the appropriate management for him?
Abdominal ultrasound
CT with contrast
Exploratory laparotomy
Exploratory laparoscopy

Female middle age coming for pap smear I think, complaining also of vaginal bleeding, her doctor
did colposcopy and found multiple cervical lesion or masses (with term I forgot it), what is the
appropriate step for her?
Take biopsy from the lesion
Hysterectomy
Excision of the lesion
I think there was no pap in the choices

3 years male child doesn't improve with albuterol, so what is the next step in his management?
Add salbutamol
Add montelukast
Add inhaled budesonide
Add theophylline

4 years old child present with history of cough and shortness of breath, he doesn't improve on
salbutamol many times, and his mother concerned about him, on examination, there was
decrease air entry in the right lung, nothing else, so what is the most appropriate next step?
Bronchoscopy
Fluoroscopy
Anterior posterior and lateral chest x-ray
CT scan

Child with fever, sore throat, and sand-paper rash, asking about most common organism
responsible for his symptoms?
Staphylococcus aureus
Streptococcus pyogenes

Child presented to the hospital with history of multiple recurrent chest infection, on examination
there was eczema on scalp and face and spread to the body involving the extensor surfaces of
both elbows, except the diaper line, CBC showed platelets count 70,000 and Hb very low (5?),
what is the most likely diagnosis?
Atopic dermatitis
Wiskott-Aldrich syndrome

Female patient 55 years old, post-menopause I think, complaining of vaginal pruritis, and clear
odorless vaginal discharge, on examination there was dryness, excoriation, what is most likely
diagnosis?
Candida
Trichomoniasis
Atrophic vaginitis

Child smiles to others, when lying on his stomach, he can elevate his body, what this child can do
also (the didn't mention setting without support)?

Can hold his head independently


Can roll over from side to side
Other things away from his age

Female patient with left breast lump, what is the best type of biopsy to be done?
True-cut biopsy
Incisional biopsy
Excision biopsy
FNA (not sure)

Male middle aged patient, complaining of flanks pain, dysuria for days I think, (they didn't mention
tenderness, fever, chills or hematuria), what is the most appropriate investigation to be done?
Ultrasound
CT without contrast
CT with contrast
MCUG

Female pregnant patient, diagnosed with UTI, which of the following is contraindicated (or
indicated, I'm not sure) for her?
Ciprofloxacin
Nitrofurantoin
Amoxicillin
Ceftriaxone (not sure)

Indication for oxygen therapy on COPD patient


2 blood reading of pO2 less than 7.2?? pKa
2 blood reading of pO2 less than 6 pKa
1 blood reading of pO2 less than 7.2?? pKa
1 blood reading of pO2 less than 6 pKa

Patient presented to the ER after MVA, severe maxillofacial trauma, mandible trauma, oral full of
blood and secretion, what is the appropriate step in his management?
Nasotracheal intubation
Orotracheal intubation
Laryngeal mask
Cricothyroidotomy

Patient after MVA, 3 attempt of intubation failed (they didn’t mention type of intubation), what is
the appropriate management for him?
Nasotracheal intubation
Orotracheal intubation
Cricothyroidotomy
Tracheostomy

Patient with previous attacks of pancreatitis admitted to the ICU previously, now he’s complaining
of abdominal pain, fever, and he’s WBCs 17,000 on ultrasound (not CT) there was 12x15 cm well
defined lesion in the peritoneum (I think there was a hint indicates infected cyst, but not sure),
which is the most appropriate step in management?
Endoscopy
Percutaneous
Surgically

Patient developed hernia, repaired by laparoscopy, after the surgery, he developed swelling in the
site of procedure, no fever, no other complain, on examination, there is no redness or hotness,
but there was tenderness, no CBC in the scenario, which of the following is the right diagnosis?
Seroma
Hematoma
Recurrent

A Q about an assessment tool that used in the developmental screening that can be done by the
parents only without the help from the doctor
A. The Ages & Stages Questionnaires
B. Battelle Developmental Inventory Screening Tool,
C. Bayley Infant Neurodevelopmental Screen
D. Child Development Inventory

24 y/o male presented with diarrhea and fatigue what electrolytes abnormality would you suspect
A- hypokalemia
B- hypocalcemia
C- hyponatremia
D- hypomagnesemia

Color of discharge in intraductal papilloma ?


A. red
B. blue
C. green

duct ectasia, color of discharge


A. red
B. blue
C. green

A woman with ulcerated nipple with oozing, you’ve done bilateral mammo (or US) you found
nothing, what is next?
A. Nipple biopsy
B. Refer to dermatology
C. Follow up in 6 months
D. I think something topical

A boy was playing football and felt sudden pain in his groin while kicking the ball or aiming to the
goal smth like that, there is a swelling at the groin but no cough impulse and not reducible, vitals
showed fever and wbcs were high, what's your management?
A) aspiration to rule out hematoma
B) give analgesics and check after 8 hrs
C) surgical exploration for hernia
D) force reduction manually or smth like that

man got high temperature , and SOB after receiving blood


- allergic reaction
-non hemolytic febrile reaction
-TRALI
-hemolytic reaction

Patient with discharge with heavy menestrual bleeding 10 pads per a day with adnexial motion
tenderness (i think in palpation there is nodularity) ?
PID
Endometriosis
Adenomyosis

SLE with active arthritis... On HCQ asking about what to add


Azathioprine
Methotrexate

RA on steroid and Methotrexate since 6 m still symptomatic


HCQ
Adalimumab
Cyclo

SLE with deteriorated renal functions and urine contain erythrocyte and leukocytes asking about
cause of renal failure..

On x ray... Erosive arthritis...

Case of systemic Sclerosis with BP180/90 and renal failure asking about ttt
Lisinopril
bb

Pt with early diastolic murmur increasing with leaning foreward


Aortic regurgitation

Target HA1c in diabetic pt Less than


4
5
6
7

40 yrs Pt with acute onset of palpitation... Everything normal in history... No BP.. Spo2
92%...asking about ttt...
Cardioversion... rhythm + unstable
Amiodarone... rhythm + stable
Bb… rate

Pt with absent distal pulses, palpable popliteal pulses, diminished sensation, altered motor
response, diagnosed with afib
A. Upper knee amputation
B. Thrombolytic therapy
C. Femoral artery embolectomy
D. Heparinization and observe

2 years old female complaining of sore throat and fever for 3 days, with bilateral knee joint pain.
(Case is complete)high Esr .
Your diagnosis ?
A- JIA
B- Osgood
C- SLE
D- Acute RF
Women 40 years old women asking about colon cancer screening , there is no family history or
past medical history ..
1-you said for here not needed in her case
2- started from 50 years
3- started (i think from now) then every 5 years

Most cancer screen in population :


1-Lung cancer
2-Colon cancer
3-Oral cancer

burn injury in the hand and foot with edema.


What is the cause
1-prostaglandin I2
2-Streptokinase
3-Serotonin
4-Bradikinin

What is the tocolytic drugs that cause patent ductus arteriosus ?


1-Indomethacin
2-Nifedipine

Women presented to ER with abdominal pain ,periumbilical protruding irreducible mass , no


pulsation with cough ?
1-Strangulated hernia
2-Obturator hernia

Pregnant at 8 week of GA had painful bleeding. Cervix was closed


US showed crown rump length of 7 weeks
Dx?
Threatened abortion
Incomplete abortion
Complete abortion
Invitable abortion

Pregnant lady presented to ER at 7 weeks gestational age with abdominal pain, Uterine mass 2*3
cm with BHCG level (18000 i think but it was more than 5000), whats make the medical therapy fail
?
Mass size
Level of BHCG
Clinical symptoms
Gestational age

lung nodule that grown 3mm since 9 m + then this month it was 10mm .. no any symptoms ,
normal physical examination , no LN enlargement
- blind aspiration
- Refer to thoracic surgery
- nothing to do
-follow up after 6 month
Pt had stroke and admitted to the ICU, developed new weakness. On examination there is
papilledema, CT done showing *hemorrhagic transformation* . What is the definitive treatment??
A/ decompressive craniotomy
B/ Mannitol

• Hypothyroidism clear Hx and labs with low Na (it was 129) normal 134
What to do next ?
1. Levothyroxine
2. treat the hyponatremia with hyper Na fluid

• Case of Hypothyroidism labs with Ab ( anti thyroglobulin Ab and other one + ), ESR normal Dx ?
1. Hashimoto
2. Sub acute thyroiditis
3. Gravies

• Postpartum 2weeks , asymptomatic , investigation showed that she had UTI ( ecoli sensitive to
all drugs mentioned in the Answers ) next ?
1. No need to treat
2. Cipro
3. Trimeth
4. Other drug

Asymptotic bacteruria only treated in pregnancy or patient undergoing urological surgery

• Pt DM i think also HTN and he had retinopathy, nothing related to renal , would you add ACEI or
no need or other medication can not recall

• Pt with Rh - and her husband Rh+ , presented for follow up , stable no bleeding , what to do ?
1. Repeat Ab
2. Assessment of fetus via US

Answer is Give anti rh

Female patient came with a history of sudden loss of vision in the left eye that resolved
spontaneously after 20 minutes. What is the most likely diagnosis?
A) multiple sclerosis
B) retinal detachment
c) TIA

Newly married young woman came for a routine check up gyne, highest diagnostic value?
A. General appearance
B. Vaginal inspection
C. Abdominal exam
D. Pelvic digital exam
25 yo female, worried about cervical cancer, she took her first dose of HPV vaccine 3 months ago.
What is the best thing to do on this visit today?
A. Schedule appointment after 3 months
B. No need to do anything at this visit
C. Give 2nd dose at this visit
D. Repeat 1st dose

Pt of stab wound 2 cm penetrating injury with omentum passing out


Pt is stable
Ct report negative findings
Next step management
1 observation
2 close the wound
3 leave the wound open
4 laparotomy

A male patient has paraplegia as a result of MVA 1 yr ago. He came now with a picture of PE + leg
edema
On ultrasound: DVT in thigh.
Patient was hypotensive:
1 IVC
2 alteplase
3 enoxaparin

Typical MI scenario they give you the ECG findings: ST elevation on lead I,L,V5-6: what type of MI?
1 inferior MI
2 anterior
3 lateral

X-ray pic of RDS “ground glass appearance “asking about the management:
1 Intubation and ventilation
2 chest tube
3 Surfactant

Neck penetrating injury on zone 1 with subcutaneous emphysema:


1 Neck exploration
2 CTA
3 Angio embolization

woman with spotting scanty blood after intercourse. She had a history of warts on vulva with
cryotherapy done 2 yrs back. What is the site of the bleeding?
A. vulva
B. vagina
C. Uterine cervix
D. Uterine body

Patient with obstructive jaundice turned to septic cholangitis. What are you expecting to see?
1 high cardiac output
2 increase systemic vascular resistance
3 hypoglycemia
4 systemic vasoconstriction

18 years old female asking about pap testing what’s true


1 at 21 years
2 after being sexually active
3 she doesn’t need the test

😅
Pt came from sudan and diagnosed as malaria
Options is drugs not sure about them

Pt with hernia reducible came to er with pain and tenderness over the scrotum
Incarcerated
Obstructed
Strangulated

35 female with Green discharge from nipple


Mammo shows dilated pre areola ducts briad2
1 biopsy
2 wide excision
3 follow up

Adult have trauma before 1 h of arriving the hospital, he was febrile and conscious asking about
water then he loss of consciousness
Bp 90/60 , HR 139
What type of hemorrhagic shock
A- 1
B- 2
C-3
D-4

burn + green discharge what is type of organisms?


E. Coli
Staph
Pseudomonas

Old patient came to the clinic what vaccine to give?


A- pneumoccoal
B- meningiocooca
C- bcg

Old age takes influenza, varicella and pneumococcal vaccine

Entamoeba histolytica(liver)
Initial management:
-Metronidazole
- another drug
-Percutaneous drainage
-Percutaneous aspiration

Placenta attached???
- Placenta Increta
- Placenta Penceta
- Placenta Accreta

10 weeks pregnant complaining of biliary colic for the past 5 weeks.


What is the most appropriate management?
A - Laparoscopic cholecystectomy now
B - Laparoscopic cholecystectomy after delivery
C - Laparoscopic cholecystectomy in second trimester
D - Laparoscopic cholecystectomy in third trimester

• pt with hematuria after running a marathon Dx other Q what is best ttt for him > Rhabdomyolysis ,
hydration

• Old male forgot his son I think , no cardiovascular hx , what is the comments cause ?
1. Alzheimer
2. Multiple micro infarction
3. Parkinson

Baby delivered at 38 weeks (or 39 werks) of gestation


Mother asked about the vaccination of HBV
1-At birth
2-After 1 week
3-after 1 month

1-Child with fever, sore throat, and sand-paper rash, asking about most common organism
responsible for his symptoms?
o Staphylococcus aureus
o Streptococcus pyogenes

2-Child presented to the hospital with history of multiple recurrent chest infection, on examination
there was eczema on scalp and face and spread to the body involving the extensor surfaces of
both elbows, except the diaper line, CBC showed platelets count 70,000 and Hb very low (5?),
what is the most likely diagnosis?
o Atopic dermatitis
o Wiskott-Aldrich syndrome
3-Child smiles to others, when lying on his stomach, he can elevate his body, what this child can
do also (the didn't mention setting without support)?
o Can hold his head independently
o Can roll over from side to side
o Other things away from his age

Answer is B or A (role over is 6 months)

4-Patient with pallor, fatigue, and symptoms of anemia, on examination there was hepatomegaly 2
cm below the right costal margin, labs showed, Hb 5, direct and indirect coomb's test was
positive, peripheral blood smear showed (exactly picture of spherocytosis), asking about most
likely diagnosis?
o Spherocytosis
o Autoimmune hemolytic anemia
o Fanconi anemia
o G6PD

6-Case of cystic fibrosis (giver), ask about the way of inheritance


A. Autosomal recessive
B. Autosomal dominant
C. X-linked

7-Obese patient did a Roux surgery for GERD asking about the percentage of recurrence:
A. 5%
B. 10%
C. 15%
D. 30

8-Patient on ACE for her HTN wants to conceive. What is your advice for her medication

A. Continue it as there is no contraindication for it


B. Reduce the dose
C. The same dose
D. Change to other drug class (not sure if the drug is written or just mentioned the class)

9-Mom came in antenatal period wants your advice in breastfeeding:


A. It binds the baby to her mom
B. Protects him against infections
C. Protects him against cerebral diseases
D. Can't remember but I think it is unrelated

30- 68 Y.o k/c of dm and htn , presented to the hospital with chest pain for past
week,this pain retrosternal and last for 15 min,increasing ,ecg show st segment
depression in lead V1-V4 , The patient received statin ,aspirin ,B blocker , heparin
,nitrate , captopril .what add ?
A. Candesartan
B. Clopidogrel ( any pt with ACS should receive dual antiplatelet )
C. Amlodipine
D. Atelaps ( STEMI, PE , Strock in 1st 4.5 hr , PAD )

31- 49 yo male k/c asthma and moderate MR presented to ER with sob,o/e he has Grade lll
pansystolic murmur and clear lung fields.best diagnostic test ?

A. TEE
B. TTE
C. SPIROMETRY
D. CHEST X RAY

32- 47 FEMALE CAME WITH FATIGUE FOR 2 MONTHS ASSOCIATED WITH WEIGHT
GAIN,SHE ALSO COMPLAINING OF AMENORRHEA FOR 6 MONTHS AND BILATERAL
MILKY BREAST DISCHARGE . WHAT IS THE MOST APPROPRIATE NEXT STEP?
A. TSH
B. SERUM PROLACTIN
C. MAMMOGRAM
D. PITUITARY MRI

33- 67 FEMALE K/C CASE POORLY CONTROLLED DM2 AND HYPERTENSION.SHE IS


ON ASPIRIN, ATENOLOL ,METFORMIN,INSULIN,MULTIVITAMINS. SHE COMPLAINING
OF DIZZINESS WHEN SHE STAND ,ON EXAMINATION THERE IS DECREASE IN HER BP
WHEN STAND BUT HR DOESN'T CHANGE. MOST LIKELY EXPLANATION ?
A. SIDE EFFECT MED ( postural hypotension with tachycardia )
B. AUTOIMMUNE NEUROPATHY( postural hypotension without tachycardia )
C. ADDISON Disease
D. DEHYDRATION

Baby delivered at 38 weeks (or 39 weeks) of gestation


Mother asked about the vaccination of HBV
1-At birth
2-After 1 week
3-after 1 month

Pregnant lady smokes 20 cig/day, she has hypertension. What is the most common risk factor for
getting a baby with malformation:
A. HTN
B. Age
C. Smoking

Pt presented with inguinal lump since 3 years , positive cough impulse + positive inguinal
occlusion he is asymptomatic what to do :
-herniotomy if child
-observation
-hernioraphy
-mesh repair if adult
pregnant with SOB tachycardia , BP normal, doppler normal, no DVT, what is the next
investigation:
A-D dimer
B- V/Q scan
C- CT

40something pt come for employee check up found to have cardiomegaly in x ray Asymptomatic
ecg normal ،Echo left ef 40%. What to do next?
A- ACEI
B- Digoxin
C- Niacin
D- Echo after 3 or 6 months not sure

Elderly asymptotic came for check up found o\e found murmur, valve gradient 40, what’s your
next step?
A⁃ F/u
B⁃ Surgery valve replacement
C⁃ Valvuloplasty

pt with early diastolic murmur increasing with leaning forward > AR

35 years old female known case of SLE came with left and right ankle severe pain. A diagnosis of
SLE arthritis is made. She is already on lupus medications. What drug to add ?
A. methotrexate
B. hydroxyurea
C. sulfasalazine
D. prednisolone

Chest gun wound entry was lateral to left nipple and exit point below left scapula, patient
suddenly became worse w/ raised JVP and muffled heart sound, your action?
A- Pericardiocentesis
B- Immediate thoracotomy
C- CT

36 year old male after road traffic accident presented with chest ecchymosis.
HR normal BP normal RR normal ECG shows Arrhythmia “written like that no image”
Chest x-ray Sternal fracture What is the most likely cause?
A-Aortic disruption
B-Cardiac contusion

2.SLE pt with deterioration of renal profile ..raised JVP …LOWER LIMBS EDEMA …C3 LOW WHAT
COMPLICATION PT HAS:
CONSTRICTIVE PERICARDITIS
RENAL FAILURE
HEPATIC FAILURE

3. Pt with systemic sclerosis ..comes with fatigue and dizziness


Bp188/90 Renal impairment
What medications to add
Lisinopril
Amlodipine
BB

4.Child presents with LL edema and puffy face … hx of skin infection 2 weeks ago
High BP.. Inx … high cr …C3 low
Diagnosis ?
PSGN

8.40 yrs pt with acute presentation of palpitation …ECG irregular HR 170 SPO2 92% cardiac ex
normal ..ECHO normal
What to give
Cardioversion
Amidaron
Digoxin
Metoprolol

9.Pt asymptomatic ECHO ef 40 % dilated cardiomyopathy what ttt to give


BB
Lisinopril
Amiodipine

11.Pt with inferior MI (ECG ATTACGED ) given GTN..ASA …LMWH suddenly deteriorated BO 75/50
JVP RAISED chest clear normal heart sound no murmur .. what the possible cause
Papillary muscles rupture
RV failure

No murmur -> RV failure


Murmur-> papillary muscle rupture

12.Pt with symptoms of asthmatic symptoms in working place only …becomes asymptomatic in
weekend…now she is asymptomatic …normal ex and inx … how to manage her?
Inhaler ventoline
Reassess during working days

18.Pt with anaphylaxis secondary to last vaccine come for vaccination what to do
Allergic test

19.Child with hx with polyuria and polydipsia recurrent admission with severe dehydration …with
family hx of same symptoms among boys
Diagnosis
DM
DI

20.Newborn with hx of brother died due to sepsis..asking about vaccines now?


Give BCG
DON’T GIVE BCG
Give vaccine acc to Saudi arabia protocol

21.Preterm …wt 1500..gaining 0.5 g daily since admission to nursery …asking about time of
vaccine
At birth
1 week after discharge
1m after discharge

If there is at discharge its the answer if not then B ,

23.2 Q about pt 60 yr come to annual check up what to give


Pneumococcal vaccine
Hepatitis vaccine

24.In case of limited supply of pcv …who will given the priority to be vaccinated
Pregnant lady
School child
HTN patient

25.Pregnant 31 weeks with uterine contractions ..os closed …what the most important medication
to give
Nifedipine
Mg sulphate

26.PROM … What initial step to do


Sterile speculum exam

27.PROM ..the most significant method to confirm


Sterile Speculum exam
Chemical test of fluid

28.Pregnant in 38 weeks with headach and visual symptoms


BP 170/90
Plt 50
URINE +++ PROTEIN
The most importatnt ttt
Emergency delivery

29.Drugs that prevent preclampsia


ASA
30.Pt with vaginal bleeding ..HCG 350000..u/s snowstorm appearance
Diagnosis : molar pregnancy

31.Pt with ectopic pregnancy vitally stable 4 cm BCG 3000 best ttt
MTX
SURGERY

32.Risk of ectopic pregnancy recurrence


10%
30%
50%

35. Postpartum lady with difficulty to lactate her baby …fatigue ...Cold intolerance
…coarse facial features.. low T3
What initial step
Request TSH
To give thyroxine
To give estrogen

13.The most significant differentiate between UC AND crohns


Non.caseating granuloma

14.Pt with dry cough..joints pain ..skin rash ..in CT bilateral hilar lymphadenopathy (sarcoidosis)
Ttt
Conservative
Prednisolone

10 days baby with respiratory distress ,lethargy


Absent femoral reflux
) Next management(
surgical repair
balloon dilation (something like that)
prostaglandin

63 years old male medically free complaining of right knee swelling And pain Examination
revealed right knee pain erythema , on examination there is right knee effusion and redness , on
x-ray (left) knee shows osteophytes and narrowing space, no labs , what's your diagnosis ?
- gout
- septic arthritis RT knee
- osteoarthritis LT knee
child diagnosed with pertussis and admitted to hospital to start treatment , he has 2 siblings at
home one ate age of 3 y and other at 5 y , that is the best regarding asymptomatic sibling
prophylaxis ?
A– booster vaccine now
B- booster vaccine in they are high risk children
C- start macrolides as prophylaxis
D- close observation

Patient 28 y had MVA with no obvious external bleeding presented with decreased air entry in left
side. And hypotensive tachycardia tachypnea most appropriate management ?
A-Thoracotomy
B-Needle thoracostomy
C-Thoracic tube insertion
D-Pericardiocentesis

women (forgotten age) have a history of genital warts and now complaining of bright red scanty
bleeding also moderate itching, what is the most common site of this bleeding ?
A- vaginal
B- uterus
C- cervical
D- vulvar

Q about eldrly with Afib came with severe abdominal pain, what to order?
A-CT
B-MRI
C-US

Bilateral tubo ovarian abscess management ?


A. Laparotomy
B. laparoscopic drainage
C. US guided drainage
D. IV abx

9- Elderly asymptotic came for check up found o\e found murmur, valve gradient 40,
What's your next step?
A⁃ F/u
B⁃ Surgery valve replacement
C⁃ Valvuloplasty
(another recall)

Elderly with ejection systolic murmur, asymptomatic. His pressure gradient is 40


mmhg. What’s the appropriate management?
A. balloon valvuloplasty
B. Valve replacement
C. Evaluate by echo or exercise test
D. follow-up
Is it the scenario of cholecystitis ? Pt presented with severe abdominal pain , after 3 days
everything returned to normal . Us done and showed multiple stones what you will do ? ICQ
- interval cholecystectomy
- cholecystectomy before discharge
- cholecystectomy 6-8 weeks

13- 70 Y.O male complain of lower abdominal pain and desire for urination and has a history of
progressive urinary tract obstruction . High psa. most likely diagnosis
1- prostatic cancer
2- BPH
3- UTI

Answer is A , Also have Back pain

-64 pt admitted to hospital because of pneumonia, she K/C of end stage renal disease , “ nothing
about surgery “ vital stable , what prophylaxis will you give pt ?
A- enzoparin
B-UFH
C- founoparix

treated breast cancer when she can get pregnant after treatment?
A- 3M
B- 9m
C- 2 year
D- 5year

Poor prognosis of schizophrenia:


a. Onset in Adolescent years
b. Family history
c. Acute
d. Anxiety with flares

According to psych-smle group

GERD case endoscopy was normal what is the most appropriate next
step ?
A- PPI
B- Abx
C- 24hr ph monitoring
D- Manometry
If esophagitis then do Manometry
If only gastritis then C

3 years old boy barky cough no inspiratory stridor, positive monophasic wheeze in auscultation,
dx?
A- bronchiolitis
B- Tracheomalacia
C- Laryngomalacia
D- Bronchial asthma

Pediatric pt fell 2 days ago ( Fx of radius and ulna ) her parents said she doesn’t move her hand
after that but they think it was normal they didn’t see any bruises, Ask about most identifying
child abuse in this case ?
A- Delay presentation 2 days later
B- same parent story
C- type of the Fx radius and ulna

pt with femur fx , before reduction most imp is ?


A- Give analgesic
B- give sedation

Check pulse is the answer if its between the choices

Pregnant in 34 week of GA came with labor pain admitted, she had PROM 4 weeks ago , O/E cervix
is 3 cm, there is clear liquor, Ask about next step?
A- Emergency CS
B- call Anastasia and deliver pt
C- Wait until 37 weeks
D- give her corticosteroids

Case painful soft palate and posterior pharynx and sparing of gingiva?
-Congenital something
-Aphthous ulcer
-Gingival cyst
-Herpangina

56.Lady post c/s by 1 week comes with SOB ..SPO2:92%.. (no BP )\confirmed PE what
to give?
1- LMWH
2- THROMBOLYSIS
3- IVC filter
If Stable A
If Unstable B

37.child diagnosed with pertussis and admitted to hospital to start treatment , he has
2 siblings at home one ate age of 3 y and other at 5 y , that is the best regarding
Asymptomatic sibling prophylaxis ?
a)booster vaccine now
b)booster vaccine in they are high risk children
c)start macrolides
d)close observation

44.3 Y.O. male patient K/C of UC presented with 7-8 bouts of bloody diarrhea
per day. Associated with fever and joint pain. O~E patient looks ill and in pain.
What is the best next step in management?
a)Oral budesonide
b)IV methylprednisolone
c)Oral 5-ASA
d)Azathioprine

68.what is the earliest sign of fracture that make you concern about it
a)pain
b)pulseless
c)paresthesia
d)cold
e)hotness or redness

Elderly woman with cerebellum hemorrhage. Can’t take food orally with absent gag reflex, what is
the appropriate type of feeding? (Ps: no duration was mentioned)
A- duodeno something.
B - jejunostomy tube
C - parenteral nutrition
D - TPN

22 y/o female Co of bloody diarrhea since 3 mo p/e is unremarkable. VS are normal. Upon PR there
were clots of blood only. What's the dx?
A - Uc
B - infective colitis
C - celiac disease
D - internal hemorrhoids

Elderly kc of htn well controlled c/o confusion and irritability. He was fine except when he injured
his left thigh 5 days ago and he has been bed bound ever since. VS shows hypotension and o2 of
88% Also shows tachycardia and he has arrhythmia on ecg
A - PE
B - cerebral infarction
C - brain hemorrhage
D - arrhythmia

A patient vomiting severely and presents with dehydration which of these is most diagnostic of
“COMPENSATED” shock?
A)Anuria
2)hypotension
3)confusion
4)can’t remember but wrong option

Pale Extremities should be the Ans if not in the answers then I’ll go with A
Morbidly obese patients came with irregular menses, 3-4 between each menses. Now she
presents with menses for 10 days heavy and with clots, what could be the cause for the bleed?
A)fibroid
B)chronic anovulation
C)coagulation disorder

24 years old came in for PAP test and showed abnormal cell findings
A- Colposcopy
B- Repeat Pap

34 week gestation with features of severe preeclampsia ttt?


-Stabilize, mg sulphate, deliver
-stabilize, steroid, deliver

Odorless watery discharge organism?

Case of brucellosis first line ttt?


A.Doxycycline+clarithromycin
B. Doxycycline streptomycin

Doxycycline + rifmapicin is the first line

762. Most prognostic liver function test in patient with high AST AND ALT A-ALT B-PT
A-ALT
B-PT
C-ALBUMIN
D-BILIRUBIN

Parents brought their child to the hospital for inability to move his arm x ray showed fracture at
both distal ulna and radial bone. The child looks healthy, dressed well, and has good hygiene.
They recall that 2 days ago the mom dropped a plate at the child's arm and noticed he couldn’t
move his arm and after the 2nd day the child couldn’t move his arm. They didn’t notice any
bruising so they thought it’s just a strain. Which of the following is indicative of child abuse.

1)Type of fracture
2) Delayed presentation
3) Consistent story of the parents
4) the appearance of the baby

Parents brought their child to the hospital for inability to move his arm x-ray showed fracture at
both distal ulna and radial bone. The child looks healthy and dressed well, and has good hygiene.
They recall that 2 days ago the mom dropped a plate at the child's arm and noticed he couldn’t
move his arm and after the 2nd day the child couldn’t move his arm. They didn’t notice any
bruising so they thought it’s just a strain. What should you do?

A-tell police about child abuse


B-call the social services
C-treat and believe family about their story

A pt want to quit smoking and he said will do it next year what this is called
A-preparation
B-precontemplation
C-contemplation

Child complained of abdominal pain and heartburn, on EX: mild abdominal tenderness, his mother
also has a history of mild abdominal pain for 10 years, what’s the appropriate management ?
1/ omeprazole
2/Azithromycin
3/clarithromycin
4/ hypo-allergic diet

A 34-year-old woman patient coded in the Female Medical Ward. The code team was trying to
intubate her with a size 9.0 ETT tube. 40 seconds has passed while trying to insert the tube. The
vocal cords are visible, the ETT is lubricated, the stylet is inserted and prepared, but tube
insertion was not possible.
Weight 60 Kg Height 150 cm What should be done now?

A-Ask the nurse to give size 8.0 ETT while making sure not to lose the view
B-Remove the laryngoscope، oxygenate the patient، then retry to intubate the patient with another tube
that has the same size
C-Remove the laryngoscope، oxygenate the patient، then retry to intubate the patient
with another tube that has size 7.5
D-Keep trying to intubate the patient while providing a flow-by oxygen with oxygen tubing

Pregnancy previous history of DVT what to give?


A- aspirin
B- warfarin
C- enoxaparin
D- no need
Elderly female with Hx of COPD came to you complaining of SOB DYSPNEA She is using LABA,
LAMA, SABA, ICS and i think other medications , she did chest physiotherapy Vitals are stable
ABG normal O2%90-93% PH normal Co2 normal As much as i remember everything was normal
What is the next step?
A- repeat physiotherapy
B- add oral steroids
C- long term O2 therapy
D- Mechanical ventilation

There is messing info , she has cor pulmonale, o2 was 88%

Management for mMRC 2 (Modified Medical Research Council)


A. Combined Salmetrol + fluticasone
B. Phosphodiesterase-4 (PDE₄) inhibitors + Salmetrol + fluticasone
C. Albutarol as needed + Salmetrol+ pulmonary rehabitilization
D. Albutarol as needed + fluticasone

Female patient in the OR, she told the nurses that she didn’t like to receive
blood transfusion during the surgery she needs to receive blood, She is drowsy. What will you do.
A- take consent from the husband
B- Take consent from eithical committee
C- Take consent from the patient before she loses conciseness
D- give without consent.

A 15 y/o boy is very concerned that he may be developing the same


disease as his father who died aged 37 y/o of colon cancer. The boy has noticed
he has developed unexpected episodes of diarrhea and lower abdominal pain.
These patients also develop other comorbidities. Which of the following would be
a typical example?
a. Astrocytoma
b. Osteosarcoma
c. Retinoblastoma
d. Duodenal carcinoma

young female with chest pain started after she arrived by flight journey what is
management?
A-Thoracostomy tube
B- Thoracotomy
C-Observation

3 years old child, his mother noticed blood in the diaper for 3 days, and previously before 2
months, he is otherwise asymptomatic
on physical examination he had pale conjunctiva
lab results were consistent with IDA. Diagnosis?
A-Meckel’s diverticulum
B-Juvenile polyposis
‘ there is a pic woth Q ‘ the case was about a post menopausal women who didn’t complain of any
thing except a minimal bleeding when she touches the urethra (the radish spot), what is your Dx:

A- urethral caruncle
B- urethral prolapse
C- vaginal atrophy
D- furuncle

Patient on enalapril and other medications and DM and maybe HTN? had dvt So heparin was
given. On day 5 she had high Potassium High PT High aPTT Other labs were normal. CBC not
given in the Q. What drug should you stop?
A- enalapril
B- heparin

58-Elderly had cough with white sputum that turned yellowish with diffuse infiltrates bilaterally
and more dense in right lung . Dx ?
A. Atypical pneumonia
B. pneumococcal pneumonia
C. Viral pneumonia

59-33 y/o was drowsy and confused . Hemoglobin 8 - MCV low - Reticulocyte 5 - Nothing about
iron. Next best management?
A- blood transfusions
B- IV iron
C-IM iron
D-oral iron

60-Neck swelling showed enlarged lymph nodes with follicular thyroid cells, thyroid was normal.
What to do

A. reassures
B. thyroid surgery

62-Anal examination shows 3 and 9 oclock fistula, what next

A. colonscopy
B, reasunce

70-A patient known case of polymyalgia rheumatic. Came with B symptoms. What is the
diagnosis?

a. CML
b. CLL
c. Giant cell arteritis
d. AML

A test for liver cirrhosis picked 200 cirrhotic out of 800 cirrhotic patients and 100 cirrhotic out of
800 healthy individuals. What is the sensitivity of this test?
A-25%
B-33%
C-67%

Patient post MVA underwent a CXR showing multiple left sided rib fractures from 3 to 7. After
some time another CXR was done showing left sided basilar atelectasis and effusion. Which of the
following is the most likely diagnosis?

A) Flail Chest
B) Pulmonary Contusion
C) Thoracic Aortic Rupture
D) Pericardial Effusion

A patient in her 40's with abdominal pain took a biopsy and it showed vasculitis. what is the
diagnosis
A- Takayasu arteritis
B- Polyarteritis nodosa
C- Wegner
D- giant cell arteritis

Child 15 months gastroenteritis on exam sunken eyes, depressed fontanel,decreased skin skin
turgor degree of dehydration :
A- 5
B-10
C-15
D-20

‫ في سؤال انو ال‬endometriosis degenerate ‫بيتكون من؟‬


-fatty
-red
Granular
Hylaine

Female 30-year-old her pap smear, result showed squamous cell ca (SCC), what to do next?
A. Colposcopy directed biopsy
B. Repeat pap smear
C. Total hysterectomy
D. Neoadjuvant chemotherapy

Ctg of Prolonged deceleration+low maternal bp


-oxytocin
-mgso4
-epidural

‫ وجاتني صورة ال‬condymalta ‫بس بطريقة ثانية انو ايش العالج ليها‬
-anti bx
Anti viral
-steroid
-nothing

‫ وجاتني صورة‬inerior lead st segment depresstion


‫وال دفرنشيال كان‬
Anterior st segment depression
Stable with st segment depression. ‫ متاكدة انها ما كانت‬elevation***

Patient presenting with moderate bleeding in 9th week of pregnancy, Os is open, doctor saw some
tissue on the cervix. The US showed 3x4 fetus.
What is your management?
A. Expectant management
B. Evacuation of the contents
C. Oxytocin

A patient with Crohn’s disease treated with azathioprine and other medication presented
complaining of pus out from anal. Vital signs are stable also no fever. what is the next step?
A. Pelvic MRI
B. Broad IV AntiBiotic
C. Swab and culture
No Percutaneous drainage in the choices

Patient RTA, became paraplegic and imaging showed T4 fracture. He is breathing comfortably. His
BP was 70/45.
A. immediate intubation
B. give intravenous fluids

‫شهور عندو‬٨ ‫ في انو بيبي عمرو‬inguinal hernia not complicated


Ttt
-herniotomy
-hernioplasty
-no need for repair

A 45-year-old male patient complaining of back pain worsen in the night, not improved by rest,
and improved with exercise and in the morning, and the most likely diagnosis is Ankylosing
spondylitis, which of the following is the most appropriate test in this case?

A HLA b27
B MRI sacroiliac
C CT sacroiliac
D Anti CCP
What is the earliest symptom that indicates dementia?
A- Becoming irritable/agitated
B- Difficulty finding words
C- Forgetting where he put things
D- Forgotten future appointment

young female with pain for 2-3 days during menstruation, this causing her
problem at work, you give her nsaid after nsaid she gets better and normal daily activity but she
think she need more sedation what to give
-diazepam
- educate her about her self care

Pt is with non-acth dependent cushing for rr adrenalectomy ..what is the


Postoperative management ?
A. postoperative fludrocortisone
B. postoperative metatone ?
C. preoperative hydrocortisone
D. pre operative (drug for pheochromocytoma forget it’s name )

Mother asking about the benefits of breastfeeding, Best answer?


-Decreases hemorrhagic disease of newborn
-Strengthens the bond between mom and baby
-Decreases intracranial infections
-Decreases primary immunodeficiency

PSYCHIATRY
ADHD = MORE HX FROM MOTHER & TEACHER

Old dm in ICU multiple ABX and suddenly collapsed labs


show very high liver enzymes?? Alp1800 Alt1000 ]
- A- Viral hepatitis
- B- drug induced hepatitis
- C- ischemic hepatitis

Pt, k/c of SLE, she is on prednisolone, hydroxychloroquine, MMF, she wants to be pregnant, what
to do?

A. Stop MMF start MTX


B. Stop MMF start azathioprine

Elderly, recently transferred to a nursing home for 3 months, lost interest activity, cries
every day, decrease appetite, along with short memory impairment, what is the
diagnosis?
A- Depression
B- Alzheimer disease
C- Vascular dementia
when you detect pregnant women infected with HBV ? Ask about type of prevention ;
-primary
-primordial
-secondary -
tertiary

Which of the following indicate life threatening asthma:


A. Respiratory rate > 28
B. Peak flow rate <30% predicted

Patient has diarrhea for 10 months and unintentional weight loss, labs show iron deficiency
anemia:
A. Loperamide
B. Albendazole
C. Metronidazole

Mother has her first child and he turned out to have sickle cell disease. She has since remarried
and what’s to go for premarital testing.

What is the first test you should do?


Paternal hemoglobin electrophoresis
Paternal chromosomal analysis
Maternal hemoglobin electrophoresis
Mixed biomarkers or something vague like it

31 wk preterm labour most important drug :


steroid

Long scenario of elderly female that has back and she is stooping and bending her back while she
walks to relieve the pain. What is the diagnosis?
A- Lumbar spine stenosis
B- Degenerative Lumbar spine

Patient with history of pancreatitis 6 weeks ago presents with fever and epigastric pain. US
showed 15cm well circumscribed cyst in the lesser curvature (no description of what is inside the
cyst). How to drain it?
A. Endoscopic
B. Laparoscopic
C. Percutaneous
D. Open
Pediatric patient came with her mom to the clinic. She said “hi” to the doctor. Asked her mom for
milk “me”, and was feeding her doll, then she pointed to her mouth and said “mouth”. She is
mimicking her mother, how old is she:
A-12 m
B- 15m
C- 18m
B-24 m

Case of milk discharge in non breastfeeding women with high prolactin level what radiological
test?
A. Sella turcica MRI
B. Pelvic for ovary
C. Adrenal

Child with fever and rash start around the scratch then spread to trunk and limbs ttt?
A.Atipyretic
B.Antiviral
C.Antibiotic

Pt present with convulsion k/c of epilepsy on valproic acid iv ttt?


A.Diazepam
B.phynetoin
C.borbenetol

Case of face laceration needs suture.., Side effect of lidocaine with standard dose ?
A.Dizziness
B. Palpitation
C.

Pregnant with warts in the vulva ttt?


A. Cryotherapy
B.electropherisis

At what age baby talk 6-10 word and recognize two body part
-17 month
-19month
-24 month
12month

35 years old pregnant female at 41 weeks gestational age. No contraction, no cervical dilation.
Fetal Ultrasound is reassuring, CTG showed acceleration with good variability (reassuring). Whats
the best management
A- induction of labor
B- Reassess after 1 week
C- immediate CS
D- Labor augmentation
Patient with active hepatitis c with lab data of liver function...what is suggested drug for ttt
A- ribavirin
B-Ledipasvir-sofosbuvir
C-Sofosbuvir-velpatasvir
D-preg? Interferon

Elderly with sob and productive cough , cxray normal ,minimal response after glucocorticoid trial
?
-copd
-lung fibrosis

16 year old female came with menses every 6 to 8 weak what is dx


Normal physiological variance
Ovarian cystadenoma

An 82 years old male with very painful micturition and week urination. What is the most
appropriate
management?
A. ABx for UTI
B. Foley catheter and ABx
C. Cystoscopy and TURP

Mother with child born premature 23 weeks and needs intubation and resuscitation but she
refused. What to do?
A. respect her wish
B resuscitate

48 y.o very obese lady, BMI 41 had hx of pelvic organ prolapse had no
surgery, what do you except she’s having now:
Enterocele
Rectocele
Uterine prolapse
Cystocele
Vesicocele

67 years old male complaining of painless hematuria , whx is the diagnostic test?
A- Cystoscopy
B- CT abdomen
C- Iv pyelogram
D- Us

Pregnant lady with dm in active labor ctg show abnormal fetal heart pattern which of the following
is the most appropriate next step?
- Change maternal position
- put an o2 mask
- stop oxytocin
- give tocolytic

Pt diabetic on metformin. came with abnormal liver


function test LFTS : ALT: 165 AST: 158 Other were all
high. Dx?

— A-hemochromatosis
— B- viral hepatitis
— C- autoimmune liver disease
— D- non -alcoholic fatty liver disease

Pt with normal resp examination increased jvp and s4 normal vital sign
What to give to improve symptoms?
-diuretics
-warfarin
-asprin\

patient in ICU admitted for pneumonia and intubated then extubatated then developed elevated
liver enzymes. What is the cause?
A-Ischemic hepatitis
B-Viral hepatitis
C- Medication induced

Which of the following medication affect work of warfarin:


A-SSRI
B- digoxin
C- sulfamethoxazole
D- steroid

24 year old man presented with 2-months of bloody diarrhea .He passed 3-4 loose months/ day
associated with blood ,and mucus.He gave history of left lower quadrant pain .No fever or weight
loos . He underwent sigmoidoscopy.which showed loss of normal vascular pattern with erosions
up to splenic flexure (see lab results).
Which is the following is the best treatment option ?
Test Result

RBS 5 1012/L)
HB. 10.9
Platelets count 488
WBC. 6
ESR 32
Consistency LIQUID
Color. Yellow.
Mucus and blood. ++.
Leukocytes. +++.

A) Mesalamine enema.
B) Mesalamine suppository.
C) Mesalamine oral and enema.
D) Mesalamine oral and suppository

Pediatric with seizure and other signs, labs showed metabolic acidosis, urine analysis:
aminoaciduria and glycosuria, what’s the diagnosis?
A. Galactosemia
B. Homocystinuria
C. Methylmalonic Acidemia
D. Maple syrup urine syndrome

Pediatric with fever 38 degree mom worried about vaccination


-defer for two week
-give him now
-give him analgesia and discharge

Swimmer ear treatment :


-ampicillin
-topical neomycin

Child with ear pain and foul smell discharge And he likes to swim
What is the organism:
-staph aureus
-pseudomonas aeruginosa
-staph epi

Ectopic pregnancy managed with salpingostomy. Bhcg postoperative was 3500 . how to follow up
the B HCG?
A. No need follow up
B. Pelvic Ultrasound
C. Weekly measurements of HCG until undetectable

Pt pregnant with severe epigastric pain .took antacid but relieved slightly what you will do to
reach dx
-upper gi endoscopy
-ph 24 hour
-manometry
-barium swallow

Pt with dysphagia to fluid only what you will do to reach dx :


-upper gi endoscopy
-manometry
-ph 24 hour
-barium swallow

75-Female coming to the fertility clinic to conceive ovulation is normal and did semen analysis for
the husband and it’s normal , what is most appropriate?
A- tubal patency test.
B- the other three are hormones.

76-Patient with a typical clinical presentation of pneumonia and asked , What’s the Next
Diagnostic step?
A- Blood Culture.
B- Chest X-ray.
C- Sputum Culture.

Which drug can aggravate the gout in Dm pt?


Lasix
Plavix
Aspirin
Metformin

Doctor work at hospital and prescribe drug that approved by saudi drug and food administration
,he is share hold with the company what is that ethical issue called
-unrespect for patient privacy
-lack of confidentiality
-Conflict of interest

58- What is the optimal time for anticoagulants for an initial PE episode?
A. 3 months
B. 6 months
C. 12 months
D. 6 weeks

Dm patient on metformin come with increased creatinine albumin ratio ..


microalbuminuria, for the first time, What is the management ?
A. Repeat the ratio
B. 24 h urine microalbuminuria
C. Renal ultrasound

62- Old patient of HF with dilated cardiomyopathy and Afib .. what is the most
appropriate management to control his heart rate?
A- Digoxin
63- When to say that a fever is fever of unknown origin?
A- 7 days
B- 14 days
C- 21 days
D- 28 days

In pediatrics 8 or 14days
In adult 21days

Recurrent UTI elderly , on exam had bulging( in the urethra or vagina I don’t remember)
post voiding test
urodynamic test
video urodynamic

Highest risk for endometral ca >


DM
progestrone
Late menerach
Early menopause
If Estrogen instead of progesterone then Best Ans if not then maybe A

Infantile colic case:


change formula
behavioral adaptation

Dehydration baby i think 10% , what will be the level of sodium


normal
low
High
unchanged

Child with knee arthritis and URTI infection 3 weeks ago now had murmur ( I suspect RF)
Asking about the highest diagnostic test ?
ECG
antistreptolysin O
ESR

Patient with bloody diarrhea on colonoscopy there was lesion in the ileum or blood
from ileum
Crohns

Female in middle 30s with mass. A biopsy showed hyperplasia with atypia , what makes you think
it’s malignancy?
Her age
Hyperplasia
Atypia
Surfactant ‫ صورة‬RDS
‎‫ وفي‬RDS ‫ ثانية نفس الصورة وكان الجواب‬nitrous oxide

Polyp with bleeding, low hgb


correct anemia
myomectomy

Pregnant in 40 weeks GA, presented with ruptured membrane for 24 hours with no abdominal pain
or contractions. Fetal CTG was reassuring. What is the next step?
- CS
- Induction of labor
- Give Dexamethasone
- Give Indomethacin

2 Qs about about intestinal ascariasis

Boy presented with 2 weeks bloody diarrhea abdominal distension, bloating weight loss and
fatigue
What’s the Dx? ascariasis
How to Investigate for this Dx? (Confirmatory)
Stool sample

peds with projectile vomiting what electrolyte disturbance will he have?


HypoK
HypoCa
HypoPO4

112-36 old male at ER C/O Right abdominal Pain , O/E : fever, anorexia , weight loss , tenderness
in RQ and Lower intercostal margins also patient is toxic
Temp. 37.9 ( I think but it was elevated ) wbc high, bilirubin high

US : cystic lesion without septates


CT : homogenous (not sure) and “THICK WALL with Peripheral enhancement

- what’s most appropriate Mx :


A. Ceftriaxone
B. Metronidazole
C. Surgical drainage
D. Percutaneous drainage

114-Scenario with features of bowel obstruction (nausea, vomiting, abdominal distension,


andcentral abdominal pain). Examination shows abdominal distension with diffuse tenderness
and exaggerated bowel sounds.
What is the next best initial step?
A - Barium enema
B - Colonoscopy
C - Examine the groin
D - Laparotomy

115-Smoking cessation can decrease the risk of MI after


A. 3 years
B. 1 year
C. 2 years
D. 18 months

116-Old pt with Retroperitoneal hemorrhage with history hodgkin lymphoma what’s the dx
A. Lymphosarcoma
B. Liposarcoma
C. Sarcoma

124-Which of the following is the earliest plain radiographic finding of rheumatoid arthritis
A- Juxta-articular osteopenia
B- No abnormality
C- Soft tissue swelling
D- Subchondral erosions
E- Symmetric joint space loss

125-Patient came to the ER with signs and symptoms of myocardial infarction he was going for
PCI, when the cardiologist was assessing his condition he notice the patient was depressed with
low
mood, the patient refused the PCI he demonstrate good understanding of his problem to the .
doctor what to do?
A. Treat the patient regardless of the consent
B. Refer the patient for psychiatric assessment then take the consent
C. Respect the patient choice after discussing the reasons of refusing
D. Take the consent from the patient relative

Child brought by his mother, she reported that he was crying for the last 24 hours, denied
vomiting on examination the tympanic membrane (bulging and immobility of the tympanic
membrane )
What’s the treatment
- observation
- amoxicillin

Depend on age
Patient diagnosed as Atypical complex hyperplasia
What is management?
Progesterone
Tamoxifen
Others choices i forgot

Depend on age and completion of family

55-year-old female comes with bilateral breast pain bilateral green discharge from multiple ducts.
Imaging: Multiple dilated ducts, not suspicious. What to do?
A - Core needle biopsy
B - Interval follow up imaging
C - Galactogram
D - Surgery

Vaccine for 9 m?
Mcv + measles

Pt did splenectomy and admit in ICU then he developed sepsis , from where he get it :
A-foley catheter
B-URTI
C-colon

46-year-old male presents to ER with RUQ pain, fever, chills and rigors and SOB. He is three
weeks post laparotomy for perforated duodenal ulcer.
Pain radiation to the tip of shoulder
On examination, he is febrile and tender in the right hypochondrium.
Chest X-Ray showed right lower lobe atelectasis with some pleural effusion. What is the most
appropriate next step?
A - CT chest
B - Abdominal Ultrasound
C. Antibiotics

If there is Abdominal CT its better

32 year old pregnant lady in 3rd trimester was complaining of palpitations and sweating and heat
intolerance along being restless she have notice a large goiter but no dyspnea or difficult
swallowing and change in both orbits that became more prominent. ECG - sinus tachycardia. CBC
-LFT - RFT within normal. TSH 0.1 ( low ) , T4 10 ( high ). Thyroid Scan showed diffuse
homogenous uptake. Anti-Thyroid Stimulating Antibody is pending. what is your next step for
management ?*
Thyroidectomy
Radio-active Iodine Therapy
Methimazole
Propylthiouracil.

Child 9 or 10 years diagnosed as rheumatic fever. What’s the most common organism?
GAS
GBS
Saph aureus
Staph epidermis

43 yrs female heavy smoker , came for screening , which screening test indicated for her ?
A. Ct lung
B. Colposcopy
C. Mammography and ct lung(not sure)
D. Mammo+colposco+ct lung
Mammogram alone

Pregnant 8 weaks gestation diagnosed with missed abortion, what you will do next?
A-Misoprostol
B-Oxytocin
C-Hystrectomy
D-Methotrixate

By exclusion A
Less than 13w -> conservative unless unstable or heavy bleeding or sepsis

Boy K/c of DM1 had ketacidosis but treated , what is your advice or ur management regarding his
insulin regimen?
1- Nph insulin twice a day
2-Just before male and cancel long acting
3- basal insulin with long acting glargine
4- basal insulin without long acting

If choice 3 was BOLUS not basal it would be the answer


It should be 2 short acting + long acting OR 3 short acting and one long acting

Patient with left inflamed red hemi scrotum, on examination there was red firm irreducible painful
swelling with high riding which is extended to to left inguinal region. Left testis couldn't be
palpated. What is the diagnosis?
A. Torsion appendix testis.
B. testicular torsion
C. epididymo orchitis.
D. incarcerated inguinal hernia

Child known case of asthma present to ER have an asthma attack , his father is heavy smoker,
you told him many time to stop smoking but he is likely careless about his son, what you will do?
A-Call child protection team
B-Consult the ethical committe
C-Explain to father importance of immunization
D-Ignore father refusal and focus to treat the child

Child has recurrent uti and undescended testis and aniridia


What else is expected to be found
Nephroblastoma
>> wagr syndrome

Bilateral hydronephrosis in baby what is the best diagnostic test


MCUG
DMSA

Child with sandpaper like rash what organism


Strept pyogens
Staph aurues

Child has fatigue and splenomegaly


Hb low
RBC low
MCV low
Retic normal
Iron normal
What to replace in this case
A Iron
B. B12
C. Folate
D. Erythrocytes

By exclusion

5- Pt with recurrent epigastric pain for weeks associated with nausea sometimes and she had
vomiting ...... mostly no other symptoms of upper GI Bleeding ... in previous hx she was using
analgesia!! For long duration for her joint pain ?
A. Acute gastritis
B. Duodenal perforation

11- Most affected nerve in PDA surgery;


A. Phrenic N
B. Vagus N
C. Lt Recurrent laryngeal N
D. Lt other nerve

13- Patient in coma on M/V after MVA for 5 days with mainly head injury on . What's the best way
for feeding at an early stage?
A-NGT
B-Gastrostomy tube

NJT is better

14- Pt with stab wound in anterior triangle of the neck ..... pt only mild hypotension, tachycardia
Radiological study showed : a vascular injury .........
A. Vascular Ligation
B. Intravascular repair
C. Open repair

17- A pediatric with weight in the 6th percentile, height on 0.4 percentile, wrist widening. What
deficiency does the patient have?
A. B1
B. E
C. D
D. A

18-Male patient came from India RUQ pain .. on and off fever for 3 weeks . raised LFT , high WBC
(Neurtrophol 70% Lymphocytes 20%) Dx :
A. hydatid cyst
B. Tuberculosis
C. Amebic abscess

Newly married female came with symptoms of uti with urgency and frequency positive nitrate
afebrile what to do?
A.Empirical abx
B.Tell her its normal and discharge her
C.Urine culture

Pt with dysphagia to fluid only what you will do to reach dx :


A-upper gi endoscopy
B-manometry
C-ph 24 hour
D-barium swallow

60- Pt with severe epigastric pain espacially after feeding ,,started ppi but he complain of burning
after lying down after dinner what you will do for mngt:
A-life style modifications
3 Other options I couldn’t remember but there was no incease of dose of

79- pt fall from 4m hight and state that he cant feel his lower extremities, on PE he had warm
extrimries. he was given crystalloid fluid which improves his BP, a CVP was placed to continue
resuscitation and it showed CVP of 2mmgh BP 76/43 (not sure)
(other vitals were given but can’t recall) which of the following is the best initial managment?
A- IV fluid
B- blood transfusion
C- IV mannitol
D- IV steroid

81- What is raised in iron deficiency anemia?


A.Total iron binding capacity
B.Serum ferritin

82- case of female auditory hallucination And delusional For 1 month after that she recover
without treatment

Might be brief psychotic disorder

83- patient RTA After doing recitation , He was fine but deteriorated bp 50/70 ?
A- FAST
B- laparotomy

13 year female came with bloody diarrhoea and weight loss


A- chrons
B- peptic ulcer
C- pancreatitis

Pt was diagnosed with abruption placenta and fetal death, cervix 5cm dilated with mild
contraction :
A.Augment labor
B.CS
C.Forceps Or ventouse

If stable A , unstable B

Asymptomatic grade 3 posterior vaginal prolapse management?


A.Conservative
B.Mesh repair something
C.Posterior vaginal repair
D.Hysterectomy

12 y boy c/o lower abd swelling after heavy work , O/e lower abd swelling with no cough impulse :
A.ventral hernia
B. periperitoneal lipoma
C.rectus sheath hematoma

12 y old boy previous hx of gromeic insertion 1 y back he likes to swim has hx of ear discharge
what organism?
A.staph
B. strept
C. pseudomonas

Postpartum depression 3 months on psychotherapy, which of the following will optimize treatment:
A- continue breastfeeding
B- include the family in the treatment

A patient came from India, hypo-echoic lesion on the right side of the liver:
A- Hydatid cyst
B- Ameba

Clear brucellosis case scenario, asking about the treatment:


A-Doxycycline + Streptomycin
No Doxycycline + Rifampin in the choices

Neck mass showed lymph nodes with follicular cells. Normal thyroid gland on examination, what to do:
A- Refer to surgery
B- Follow up in 3 months

Case of endometriosis, which of the following is an acceptable method to diagnose?


A- laparoscopy
B- US
C- MRI

39 y/o male, complaining of dyspnea, increasing with activity, and he denied chest pain or other cardiac
symptoms. Dx?
A- Angina
B- Angina equivalent
C- Muscular pain

Male present with chest pain increasing with movement but it progresses for the last 3 week, becomes at
rest and come at night:
A- Prinzmetal angina
B- Unstable angina

Q: neonate with machinery murmur, what is your next step?


A- NSAIDs
Q: Wheezing and rash post IV contrast, what is your immediate action?
A- Intubation
B- Epinephrine

Q: Neonate a few hours after delivery, body rash, and isolated thrombocytopenia (platelets 5).
A- Alloimmune thrombocytopenia
B- Congenital rubella
C- Congenital CMV

Crying infant 2 months old, what is the usual age of this condition?
A- 2 months
B- 3 months
C- 6 months

Regarding the same crying infant, what’ll do?


A- Behavioral adaptation
B- Change formula

Q: Post operative patient, Urine output of 7ml/h (exactly like this), what to give:
A- NS
B- Diuretics

Patient with SLE not controlled with one hydroxychloroquine, what to add?

Q: Patient came from RTA, no external bleeding, decreased breath on the affected side, severe
hypotension, no mention of percussion findings.
A- Chest tube
B- Needle decompression

Another same question was with external bleeding -> Hemothorax , here its mostly Pneumothorax

Q- Neonate w/ signs if Right sided heart failure, and pansystolic murmur:


A- VSD
B- ASD

Q- Most common cause of post menopausal bleeding:


Atrophic vaginitis

‫ ‏‬other with an ADHD kid, complaining of his behavior, she was told to do positive reinforcement of good
M
things and ignoring bad things? Which of the following would help her achieve the advice?
‫‏‬A-Mother training program
‫‏‬B-Behavioral managment
‫‏‬C- timeout to positive reinforcement
‫‏‬D- family therapy

Asthma with moderate presestant symptoms? On beta agonist what to add?


Ics
Ics and laba

Patient go for umrah and s&s of meningitis treatment ?


Steroids ceftrixon azetheromicin
Ceftrixon azetheromycin

infant
Hbg low
Mcv low mch low retics normal dx?
Normal
Scd
Thalassemia trait
Lead poisoning

If less than 3m age then its normal physiological anemia

IBS pt advice :
A. Increase honey intake
B. Increase fruit intake
C. Avoid lentis

48 y.o very obese lady, BMI 41 had hx of pelvic organ prolapse had no surgery, but she had + family
history of prolapse repair what do you except she’s having now:
A.Enterocele
B. Rectocele
C. Uterine prolapse
D. Cystocele

67 years old male complaining of painless hematuria , whx is the diagnostic test?
A- Cystoscopy
B- CT abdomen
C- Iv pyelogram
D- Us

Pregegnant lady with dm in active labor ctg show abnormal fetal heart pattern which of the following is the
most appropriate next step?
A- Change maternal position
B- put an o2 mask
C- stop oxytocin
D- give tocolytic

Clozapine is used in pediatric age group to treat


A- schizophrenia
B- bipolar disorders
C- major depressive disorders

Q about screening for breast cancer in age group 50-74


A-Yearly
B-Every 2 years

40-50 -> anually


50-75 -> every 2 years

A 35 year old car driver crashed into a concrete block without a safety belt on. Thirty minutes
after and on the way by ambulance to the hospital he begins to become breathless. On
administration of 100% oxygen there is not much improvement in this condition. On arrival at
the Emergency Department he has lost consciousness and appears cyanosed with markedly
distended jugular veins.
Blood pressure 80/40
Heart rate 120 /min
Respiratory rate 34 /min
Temperature 36.6 c
Oxygen saturation 60% on room air
What immediate action should be taken?
A. Intubation and 100% oxygen
B. Rapid infusion of crystalloid
C. Needle decompression
D. IV 0.2 mg adrenaline

• 20s female, medically free presented with epigastric tenderness long duration , not related to any thing,
some time it associated with vomiting. Her mother have chronic non specific abdominal pain Patient had
+ blood and mucous in stool asking what to give her
A- omeprazole
B- Tetracycline

PUD caused by h.pylori?

At what age baby talk 6-10 word and recognize two body part with no pincer grip ‫متاكد‬
-17 month
-19month
-24 month
12month

Written Dx ( Aspergillosis ) ttt ?


Voriconazole

Zone 1 Neck injury ? ICQ

If unstable or with hard signs then neck exploration


If stable with no hard signs then CTA

3- Zone 3 neck injury? ICQ


Embolization or endovascular repair

4- women diagnosed i think HF 9 month ago , now came c/o non productive cough she is on ACE , statin
and other drug . EF 40% what to do ?
A- discontinue ACE

7- lactating mother 5 days fever rt breast red swelling tender :


Abx
I&D

8- 4th day post op stomach resection now came with picture of STEMI what to do ?
Ballon angioplasty
Thrombolytic
Warfarin

9- Dysuria Dyspareunia Dribbling :


Urge
Overflow
Stress
Urethral diverticulum

10- 9 y o with meningitis Blood culture gram +ve cocci . CSF will be ? ICQ
Bacterial picture of high nutrophiles , low glucose , high protein

Case of 17 or 19 year 30 week she was managed by corticosteroid i think for preterm labor , Ask about
the rationale behind corticosteroids administration:
- hypoglycemia
- RDS

60 female Most common source of


bleeding
Uterus
Lower genitalia
Fallopian tube
Ovary

How to Stop bleeding after delivery if you wish to preserve fertility .. which to ligate
Internal iliac artery embolization
External iliac artery
Internal iliac vein
Some uetro.. ligament

Q. Female post CS 16 days ago present with fever and pain that is not improving with antipyretics and
ABX. She has history of 4 previous CS. CT showed fistula
What is the appropriate management?
A. NPO with pancreatic and gastric inhibitors
B. Resection and anastomosis
C. Stool softners
D. (Forget it)
Q. 62 years old F present to the ED with acute pain started 2 hours ago. The pain progress with time and
became more severe. US done showed Solitery with cystic component complex left ovarian cyst measure
7x8 cm.
(Scenario didn't mention any GI or Genitourinary symptoms)
What is the appropriate management?
A. Laparotomy
B. Tumor markers
C. MRI
D. CT scan

According to dr.wafaa : indication of surgery in ovarian cyst is 1- acute abdomen 2- complex cyst 3- more
than 10cm
Not sure but patient has acute abdomen and looks unstable so A, if stable B

Q. Patient post hartman procedure for diverticulitis has urine output of 50 ml over 8 hours. Central venous
pressure was 10 with lower limb edema. Other examinations are unremarkable. Vitals: Normal.
(No labs provided)
What to do?
A. Explore
B. US
C. CT abdomen
D. "Forgot it, sorry"

Q. Female patient present with PE.


Which of the following will indicate thrombophilia work-up?
A. -ve family history
B. Use of oral contraceptivd pills
C. Age more that 50
D. Connective tissue disease

Female known case of hep C came with multiple liver masses, what is further managment
A us
B biopsy
C CT

patient smoke 20 cigarettes and vital signs show BP 140/92 Which of the following is the most
considerable risk factor for MI ?

A) HTN
B) Smoking
C) Age

It should be DM

Women with previous history of miscarriage at 30 weeks. Now she is pregnant at 18 weeks and afraid of
miscarriage again.
What is most important to do?
Pelvic exam
Cervical lengths measure

An object fell on this pt’s foot and the capillary refill was delayed what are you going to do =
A-Cta
B-Duplex
C-Compartment measurement

pt has hemorrhoid at 3 , 7 .. had injection sclerotherapy and resolved... what is the type of hemorrhoid?
A-external
B-internal
C-thrombosed

15 m boy examination showed a right inguinal reductible hernia and high riding right testicle and the left
side is not swollen. How to manage?
- Wait 3 years
- left orchiplexy
- laparoscopic exploration

‫ في سؤال عن‬pregnant k/c of asthma and there is 100000 colony bacteria what is your management during
labor and delivery?
Corticosteroids
Antibiotics

Post cholecystectomy patient 8 days came with pain in angle of mouth with fever 39 stable everything
else normal ?
Ct
X ray
Antibiotics
Paracetamol

20 year old women , with no known medical illness ,having increased jaundice ,nasuea , vomiting and
RUQ PAIN
Labs show : High AST,ALT , Bilirubin,ALP
Which of the following has the greatest prognostic value ?
A-Bilirubin
B-AST
C-Pt
D-ALT

HTN nephropathy type ICQ

Htn induced nephrosclerosis

Sacriilliac joint tender with hepatosplenomegaly in veteranian?


Toxo
Syphilis
Brucellosis
TB
T1dm uncontrolled
Mother concerned
What test modality to use in annual follow up or screen “‫"نسيت الصيغة‬
A- Serum creatinine
B- Microalbuminurea
C- Urine analysis
D- HgbA1C

17 YO female has DM type 1 for 5 years with 3 times DKA. Mum is asking the prognosis of the disease,
Which can be used for annual screening fir nephropathy?
A-Urine analysis
B-Microalbumin in urine
C-Creatinin
D-HgA1c

Albumin / creatinine ratio is the best , by exclusion B

‫ جاني و كان عنده يومين‬vomiting and diarrhea


Dehydration baby i think 10% , what will be the level of sodium >
A-normal
B, low
C, high

29 GA pregnant has grade 2 pelvic organ prolapse,what’s the managment?


Begin with conservative if failed then surgery

Case of female pregnant i think 30 w 3 days ago she had PROM and it was managed then she present
today with fever , what the diagnosis ?
- chorioamnionitis

Loin pain + hematouria + hard lump on examination what is most diagnostic ?


A-CT
B-MRI
C-Scintigraphy ‫مدري وشو‬
D-Ultrasound

27 years old female complaining of green yellow discharge that started after
her mensuration, Upon examination: red lesions “like bruises”
Whats the diagnosis?

A. atrophic vaginitis
B. Bacterial vaginosis
C. candidiasis
D. trichomoniasis
A female patient presented complaining of vaginal discharge and itching. she also complains of urinary
symptoms. on examination the discharge was cottage cheese like. what is the most appropriate
management?
A. Topical antifungal
B. Topical antiboitic
C. Oral anti-fungal
D. Oral antiboitoic

If pregnant then A

Child had x-ray showing fracture of both radius and ulna with volar displacement, on examination there
was 1 cm wound, what is tha most appropriate management:
A- Closed reduction with above elbow cast
B- Closed reduction with below elbow cast
C- Irrigation of the wound and depridment

Q10) pedia with open fracture radius and ulnar best next ?
1) cast below elbow
2) cast above elbow
3) debridement with open fixation by k wire
4) debridement with open fixation by IM nail

122-2 days old with low oral intake and low activity and fever, labs urine Ecoli Aciduria and glycosuria?
A. Galactosemia
B. Maple syrup urine diseases
C. Homocysteinemia
D.DKA

Pediatric with recurrent UTI and undescended testes and mass in loin
-Nephroblastoma

70 year old female came for check up asymptomatic found to have AS he is diabetic HTN. Echo done
showrr normal EF with concentric LV hypertrophy and severely stenosed Valve ( didnt mention how much
cm) how would u managr?
Acei
Follow up and observe
aortic valve replacement
Diuretics

‫ جا زي دا بس كان عندو‬Symptoms of heart failure


And the EF was 40%
AVR

Patient with CKD has hyperkalemia what should the pt eat with same amount everyday:
A- Green leaves
B- Grapes
C-tomatoes
Other similar recall

Pt diagnosed with malignancy,


involved in his care is a multidisciplinary team.*

What approach is most beneficial for pt?


- Compassion
- Provide a favorable outcome
- Ability to Make dissisuions

Child with abdominal pain, mucus and blood with stool what is the cause?
Ascaris
Taenia solium
Amebiasis
Salt bile diet

15 year old complaining of abd pain, tenesmus, diarrhea and bloody stool for 2 months What next?
-Reassurance
- Further investigation
- Refer to pediatric surgen

Pt with abdominal pain and on exam per umbilical mass, irreducible, Vitals and lab are all normal
What to order next?
- Cl level
- Ca level

Pt presented with fatigue N/v and RUQ pain for 7 days prognostic factor
A-bilirubin
B-INR
C-AST
D-prothrombin time
27 years old female medically free presented to ER with something They need to do CT what test must
the ER doc do before CT
A-Pregnancy test
B-renal function test
C-CBC
D- INR

Pregnant 35 weeks came for decreased fetal movement CTG shoewd fetal heart rate of 130 ,
accelerations , poor variability, no deceleration . after 1h of monitoring and hydration pt CTG imroved with
good varibility what is the most appropiate
A-discharge home with fetal kick chart
B-observation
C-CS
D-IOL

Characters of major depressive disorder


A-Late awakening in the morning
B-Decreased eye contact during conversation
C-Hypomania
D-Hallucination

60 male medically free came for check up CXR showd opacity <3 cm on upper lobe LT or RT not sure,
next step
A-Review old x rays
B-Ct with IV contrast
C-Ct guided needle biposy
D-Surgical resiction

CNS brucellosis duration of tx


A-4m
B-6m
C-8w
D-6w

5 y o vomiting and diarrhea for 4 days O/E pt looks dehydrated, hypocative, sunken eyes , doughy skin ,
dry mecus membranes , cap refill is 4 seconds Bp 90/70 and tachycardic rest of vitals are normal
What is his Na level
A-Low
B-High
C-No corrolation
D-normal

50 years old had a miscarriage asking if her age was a risk for miscarriage what should you tell her
A- she has 3% risk of miscarriage
B- 10-50 %
C-80%
D-Age is not related
Which one of the following urine collection methods is the most sensitive and reliable in establishing the
dx of UTI
A-Mid stream urine
B-suprapubic aspiration
C-Folly’s cath
D-Urine bag to genitalia

Femoral shaft fracture, on exam vitals are stable , deformed LL,no other injuries
What is most important priority in management
A-Pain control
B-Maintaining joint function
C-Controlling Bleeding
D-Decrease soft tissue trauma

Case of 50 y/o grade 3 POP with mild to moderate pelvic discomfort which was managed conservatively
for 6 month. presented today with no improvement or worsening of her clinical symptoms. Which one of
the following is appropriate for her?
A) reassure and continue conservative
B) council her regarding surgical options
C) follow up 6 month
D)

35 y/o ? (no comment regarding preserving fertility) presented with menorrhagia. Dx with lager fibroid
10cm. Doctor decided to do hysterectomy but the pt refused. Which one of the following is ( appropriate
alternative )for this intervention?
A) POP
B) NSAID
C) Uterine artery embolization
D) forger but not OCP

Pt with dysmenorrhea only Sever that she doesn’t go to work. Her boss is angry that she skips work
always. Which one of the following is used to (decrease her pain) and help her get back to work?
A) paracetamol
B) OCP
forget but no NSIADS

This question about BCG vaccine , newborn his mother tell you that somebody in her family die from
immune diefincy disease what to do ?
-give BCG now
-all vaccine according to schedule
delay

Female at 39w of gestation, Examination shows polyhydramnios, what case should you discuss its
possibility with her?
A-Trisomy 21
B-Renal agenesis

How to confirm spinal inj in MVA with the end of second survey?
A-ct
B- pelvic binder
C- spinal binder

Female pregnant present with RUQ sever pain Her Bp 160/100


Labs 3+ protein Plt low Cr high Urea high What’s the diagnosis
A hepatic rupture
B hepatic capsule distention
C cholycystitis
D pyleonephritis

Young women after doing total thyroidectomy and then went under deterioration of shortness of breath
and pain after extubation they did laryngoscopy and then revealed that vocal cord is midway between
opening and closing position ( something like that i cant remember exactly ) what u should do immediately
:
1-tracheostomy
2-cricothyroidectomy
3- repair of nerve injury
4 -reintubation again

Health organization “good health for all” the year 2030 will be the end of preventable death for children
under 5 y/o by ?
School health
Health education
Immunization
-one more option I forgot

3 years old child mile stone


- play football by alternative foot
-climb the stairs
-Trace a triangle
-knows week days

Picture of the egg on the string asking about the diagnose?


-tga
-Pulmonary stenosis
-Tof

12 yrs old girl diagnosed with beckwith-wiedemann syndrome came with her family and they were
concerned that maybe her disease associated with some liver carcinoma what screening you should do ?
1- alpha fetoprotein
2- vma in urine and HVA
3- abdomen u/s i think i forgot

Case about shok i forgot cenario but about shock they give alot of parameter but mainly he have 2.5 l
cardiac output and high pwcp (20)
And 90/60 what typw of shock ?
1- cardiogenic
Hypovolemic
Septic
Q about a mother worried about her child if jhe will remain short her
height 155 her husband 187 “not quite sure about the numbers “
So the heightof the child will be : ICQ

887-Beckwith–Wiedemann syndrome, Associated Cancer?


Wilms tumor (nephroblastoma)

- Boy presented with 2 weeks bloody diarrhea abdominal distension, bloating weight loss and fatigue
What’s the Dx?
-intestinal ascariasis

2- How to Investigate for this Dx? (Confirmatory)


-Stool microscopy

3- peds with projectile vomiting what electrolyte disturbance will he have?


A- HypoK
B- HypoCa
C- HypoPO4

4- Electrolyte abnormality in TPN?


Hypophosphatemia and hypomagnesaemia + Hypokalaemia

5- Patient on parenteral feeding developed weakness and convulsions What cause these symptoms?
A- Hypokalaemia
B- Hypomagnesemia
C- Hyperkalaemia
D- Hypermagnesemia

43 yrs female heavy smoker, came for screening ,which screening test indicated :
A- Ct lung
B- Colposcopy
C- Mammography and ct lung(not sure)
D- Mammo+colposco+ct lung
Mammogram alone

98- World health organization (WHO) to determine the health of the population in KSA What should KSA
share to WHO:
A. Health determinant
B- Health indicator
C- Risk factors
D- Something variable

25 female pt, headache, malaise, fever


on examination: bilateral painful anogenital lesions, what dx :
A- genetal warts
B- genetial herpes
C- chlamydia infection

6 y girl with reddish urine, she had constipation for 2 weeks, then became inconteinent. On examination
there is abdominal tenderness no organomegaly, labs : normal wbc, low rbc.
Urine analysis: leukocytosis, erythrocytosis, +++ blood , what the dx ?
A- UTI
B- HSP
C- AIHA
D- PSGN

Pregnant 30GA, complaining of breast mass, on examination lumps at areola, soft, no tenderness, what
the dx ?
A- laceferious duct
B- mondor’s disease
C- montogmery disease

montogomery glands not disease

Pt in 30s, with productive cough, chest pain, sob, fever for 4 days. on examination there is dullness noted
on the lower half of right lung, CXR, lobar pneumonia with mild effusion, what the treatment?
A- azithromycin plus ceftriaxone
B- azithromycine plus ceftriaxone and do thoracocentesis

Typical case of pneumonia, ask about the treatment:


A- Moxifloxacin 400mg, IV, OD
B- piptazo
C- macrolide

Pregnant, 30GA, present with contractions. No vaginal bleeding or leaking. Normal CTG, with 4
contractions every 10 min, what is the management?
A- perform US to asses fetal weight
B- perform digital pelvic exam to asses cervical dilation
Female in her 30s, present with fatigue for one week, on examination splenomegaly, high liver enzymes,
high LDG, low haptoglobine, normal platelets, what is the management?
A- parentral iron
B- hydroxyurea
C- corticosteroids

Case unclear but low Haptoglobin means hemolysis.

32 male patient, develop sever hypovolumic shock due to traumatic splenic injury, managed by
splenectomy. In the operation he received 8 unit of blood, then transferred to the ICU. On the 3rd day he
became febrile. Blood culture : positive for gram negative bacilli
What is the most probable source of infection ?
A- contaminated blood
B- respiratory tract infection
C- urinary tract infection
D- from intestinal source

Female pt with malaise and headache, 2 days after she developed skin rash. She was in jeddah 10 days
ago, what is the management?
A- anitbiotics
B- antiviral
C- supportive

22 male, present with days hx of chest pain and dyspnea, happend in the airplane, on examination:
absent breath sound in right side. Cxr: right hemothorax.
What is the management?
A- tube thoracostomy
B- thoracentesis
C- conservative

Pr with SVT, initially was conscious, then start to be less responsive, and BP 80/46
What is immediate action or management?
A- electrical cautery
B- cardioversion

Pt present complaining of sob, productive cough, yellow sputum, chest pain, and fever for 4 days. On
examination crepitation on right lung. What is the diagnostic test ?
A- chest x-ray
B- blood culture
C- sputum culture

55 years old male patient, known case of pulmonary hypertention, with chronic thrombophilia. Which of
the following has the highest diagnostic value ?
A- chest X-ray
B- spiral CT chest with contract
C- ventilation perfusion scan of lung
D- Echocardiography

Male patient in 20s, present with sever epigastric pain for 6 hours, start suddenly.
On examination there is diffuse tenderness, sluggish bowel sound. Labs: high WBCs, Amylase 300
Which of the following best initial diagnostic step ?
A- Erect chest X-ray
B- abdominal X-ray
C- abdominal CT
D- abdominal US

Pt with prosthetic valve, present with infective endocarditis. Culture showed (MRSA). what is the proper
management:
A- vancomycin and rifampin
B- vancomycin and gentamicin
C- vancomycin and ceftriaxone
D- vancomycin and rifampin and gentamicin

35 year old pregnant came with bilateral breast tenderness and mass what investigation you’ll do?
A. US
B. mammography
C. reassess after delivery

breast mass behind nipple, on US there is hypoecoich lesion, what next


A. FNA
B. core biopsy
C. exicional biopsy
D. reassess after

41 came twice to the clinic for CTG , they did CTG this time and showed deceleration , what you will do ?
-delivery
- daily CTG
- us for BPP
- us for amniotic fluid

Child maybe less than 5 years old, presented with inflamed hemiscrotum, the cord is thick and
edematous, the testes in examination is viable what’s the Dx
A. Testicular torsio
B. Appendage torsio
C. Epydidmorchitis

Which of the following indicates no response to IVIG in Kawasaki


A. Neutropenia
B. temperature 38
C. duration of fever
D. High fever
What indicate adequate systemic perfusion?
A. Cardiac index
B. Mixed venous O2 saturation
C. Central venous pressure
D. pulmonary capillary wedge pressure

Pt vaginal discharge (no description of smell or character) with severe itching. Husband has mild urethral
discharge. Mx
A. Metronidazole
B. Fluconazole
C. Sth azole
D. Sulfonamide

Milestone of a child that can jump on both legs makes blocks of 6, says 2 names and 2 wordsi think
A. 1 year
B. 2 year
C. 3 year
D. 4 year

A boy was playing football and felt sudden pain in his groin while kicking the ball or aiming to the goal
smth like that, there is a swelling at the groin but no cough impulse and not reducible, vitals showed fever
and wbcs were high, whats ur management?
A) aspiration to rule out hematoma
B) give analgesics and check after 8 hrs
C) surgical exploration for hernia
D) force reduction manually or smth like that

Case of impaired memory On ct it shows periventricular hyperintensities (no image shown) what is
diagnosis?
A- lewy body dementia
B-Vascular dementia
C- Normal pressure hydrocephalus

32 male patient, develop sever hypovolumic shock due to traumatic splenic injury, managed by
splenectomy. In the operation he received 8 unit of blood, then transferred to the ICU. On the 3rd day he
became febrile. Blood culture : positive for gram negative bacilli
What is the most probable source of infection ?
A- contaminated blood
B- respiratory tract infection
C- urinary tract infection
D- from intestinal source

Child came in the early morning complaining of barking cough and inspiratory stridor he lapeled as case
of croup and receive mx but no improvement what is the important investigation you should do ?
1-lateral neck x-ray
2- direct visulaziation by laryngoscope
3 cxr
4. Pinpoint toxicity what will you give?
A.Naloxone

5. 64 pt admitted to hospital because of pneumonia, she K/C of end stage renal disease what prophylaxis
will you give pt ?
A- enzoparin
B-UFH
C- founoparix

6. pt diagnosed as trachiomalecia ask about what is the best diagnostic tool ?


A-bronchoscopy
B-ct chest
C-x ray

9. Victim of RTA came with open book pelvic fracture, there’s blood in the meatus and scrotal echymosis,
next step?
A- Folley catheter
B- Ureteroscope urethrogram
C- Pelvic CT

10. Female presented with premature rupture of membrane with fever and vaginal discharge. The uterus
is tender to touch. What is the possible diagnosis?
A. chorioamnionitis.
B. UTI.

13. Child has recurrent uti and undescended testis and aniridia What else is expected to be found
Nephroblastoma

D tap vaccination and a child with previous febrile seizures “not sure if it was due to the last dose of dtap
or not “ what will be your action?
-Tel the mother it will not affect the baby
-Delay
-according to the saudi guidelines it’s okay to give the dtap in case of febrile seizures

Patient presented with painless vaginal bleeding and placenta covering the internal os , Which of the
following is considerd a risk factor for her presentation ?
A- HTN
b- multigestation
C- smoking
D- Dm

12 year old boy injured with supracondylar fracture and distal radial pulse absent , which of the following
is the appropriate next step in management?
1- K wire
2- intramedullary nail
3- surgical exploration
Closed reduction
A mother bringed her 2 week old infant consered of multiple collection on his skin . 3 collections 2 of them
where superficial red flat spot on the lower limp and the third where in the right of the eyelid , which of the
following is the appropriate step in mamangment?
A- refer to ophthalmologist
B- refer to pediatric surgeon for excision
C- rassure the mother that it will dissapear in 5 years
D- antibiotic for 1 week

A child had recurrent otits media and chronic diarrhea which of the following is the appropriate diagnositic
method ?
A- C1 level
B- immunoglobulin level
C- small bowel biopsy

43 year old lady developed paraumbilical ventral hernia that is irreducible and no cough impulse with
vomitng and air fluid level, the skin is red upon examination, which of the following is the appropriate
differential diagnosis?
A- incarcerated hernia
B- strangulated
C- obstructed
D- irreducible

Patient had 4 mm ureteric colic what is the appropriate managment?


A- external shock wave lithotripsy
B- conservative treatment
C- ureterscopy

Which of the following antidepressants causes constipation?


A- SSRI
B- Snri
C- Tca
D- Moa

43 G4p3 36 weeks came for follow up , fundal height 36 cm with cephalic presentation 6 cm dilated and
fetal heart rate 145bm. After 5 minutes she had rupture membrane and suddenly the fetus developed
bradycardia. Which of the following is the most important to exclude as differential diagnosis?
A- cord prolapse
B- abruptio placenta
C- uterun rupture
.
.
Which of the following has the heighest diagnostic value for renal colic patient ?
A- kub x ray
B- Ct abdomin

Full term delivered healthy baby


Vacation should give before discharge
HBV & BCG
HBV& DTAP
MMR & BCG
DTAP & BCG

Pt. Appindicitis
When they open it was normal what should do
Remove it and close

If its open then u have to remove the appendix even if its normal.
If its laparoscopic search for other causes if there positive other abnormality then leave the appendix and
treat the abnormality , if no other abnormality remove the appendix

20 days after normal vaginal delivery pt came to obegyne for check up and there was pink cervix and on
microscopic there was epithelial cells, no itching no symptoms
Management:
A- Metronidazole
B- Reassure

COPD pt how can we now improving


‫ هو بيشنت من سنين عنده‬copd ‫ ويستخدم عالجات ناسيه كان‬poor ‫تقريبًا والسؤال كان كيف نعرف انه امبروف من الخيارات اذكر‬
FEV
FEV/FVC
‫تقريبًا كان كذا ما اذكر حيل‬

Pediatric with recurrent sinopulmonary infection and recurrent gastroenteritis which is confirmed to be
with giardiasis. His brother died of sepsis?
A) X-linked agamm
B) selective Iga

Female patient with with long standing incisional hernia came to ER with fecalith discharge and ulcer all
vital are normal, what will u do ?
A- dressing
B- laparoscopy
C- MRI
D-lapratomy abdomen

Q about research and at the end What is the Most appropriate way to review the proposal? -review the
proposal by Your colleague
-let your colleague to review the research result
There should be ethical approval before the research

Infant with nausea vomiting and sever abdominal distention, malnourished


X ray showed huge stomach whats Diagnosis?
1-Pyloric stenosis
2-duodenal atresia

Management of acites with hypervolimic hyponatremia


Restriction of fluid + spironolactone or furesmoide
Management of grade 2 ascites

Patient is known nephrotic and he just finished steroid course and want MMR vaccine he take 2
mk/kg steroid for 6 weeks - What you will do ?
A-No vaccines
B-Give the vaccine
C-Ask her to come back after 4 weeks
D-Ask her to come back after 7 weeks

Middle age male patient, complaining of Shortness of breath in night, and cough (I think), his past history
insignificant except for his working in car painting shop, also he states that the symptoms disappear when
he's out of work, and come back during work, on examination he was completely normal, what is the best
advice (I'm sure) for him?
A- Start corticosteroid therapy
B- Change his job
C- CT scan
D- X-ray

RTA patient with lower back pain and spinal injury has not ruled out yet you are waiting for the consultant
what to do to prevent further injury while waiting How to confirm or to know that
2ndry survey is complete???
1- pelvic binders
2- movement restriction
3- CT
4- spinal binder

Infected central line organism :


Staph aureus
Epidermidis
24 years old with recurrent on and off PR bleeding after defecation. She has spontaneous reduction of
hemorroids. Hb was low. On examination: 3 hemorroids 2,7,10 O’clock how would u manage?
Observe
Band ligation
Hemorroidectomy
Conservative

Pathophysiology regarding patient with appendicitis complicated by Appendicular mass (Abscess), patient
was Feverish (38.7)?
A. Peripheral Vasoconstriction
B. Decreased Cardiac Index
C. Redistribution of Blood
D. Bradycardia

41. COPD pa,ent presenting with bilateral lower limb edema and pulmonary
hypertension. PO2 8.6, PCO2 7.5, pH within normal range Which of the following
the appropriate management?
A) start furosemide
B) Start oxygen therapy
C) Prednisolone therapy

Child came with colicky abdominal pain, vomiting, bloody stool. US showed doughnut sign. What is the
most important step in management of this case ?
• A- urgent surgery referral
• B- NGT decompression
• C- IV Fluid resuscitation
• D- Barium Enema

Patient will be dehydrated so begin stabilization first

Case of Cushing sign what to do regard to vaccination?


If the patient taking systemic steroid for more than 2w or more than 2mg/kg -> must delay live vaccine for
1m

child with flu- like symptoms fever and cough on physical exam show Crackles and bronchial breath
sounds . On CXR diffuse infiltration more dens in middle lobe . (Pic CXR given)
Heart rate , respiratory rate blood pressure, O2
All within acceptable range
Most appropriate Mx?

A- Inhaled steroid with SABA


B- Admission and IV ABX
C-Discharge on Amoxicillin for 7 days
Patient complain of diarrhoea, culture show C. Difficile infection , Most appropriate management ?
Iv metronidazole
Oral vancomycin
IV cephalosporin (

65 y , Male , smoker came for check up, Most appropriate screening test ?
A-Colon cancer
B-Abdominal aortic aneurysm
C - osteoporosis ( not sure )

Old Female patient , When should you screen for Osteoporosis ?


A- 65 - 69
B- 60 - 64
C- 55 - 59

Child with recurrent arthritis in wrist and knee + morning stiffness improved throughout day ( no labs and
no mention of rheumatoid factor in scenario) .. Dx ?
A-Juvenile idiopathic Arthritis
B- Reactive arthritis

24 years women sexually active complaing of recurrent UTI , Most appropriate recommendation ?
A- Antibiotic if symptomatic
B-Antibiotic for 2 weeks
C- Antibiotic for 6 month
D- Antibiotic postcoital

Thyroid mass with high Calcitonin


A- Medullary carcinoma
B-Papillary

Female in pregnant 9 week of gestation came with flank pain vaginal bleeding , on pelvic examination OS
closed and see blood through it
A- Reassurae , and revaluation after 1 week
B- Admitted , stabilize and observation for possible termination of pregnancy

Time of screening for infant with high risk of Iron deficiency anemia ?
A- At birth
B- 4 month
C- 10 month

Patient diagnosed with croup and was given initial management including racemic epinephrine but still no
improvement ?
A- Repeat epinephrine
B- IV Corticosteroid

Child after viral URTI complain of dry cough and was given SABA but no improvement.. appropriate
management?
A-Montelukast
B- Inhaled corticosteroids
C- ABX

Rheumatoid Patient came with fever and Shortness of breath . he was on Methotrexate and Adalimumab
. Most appropriate action ?
A- Hold Methotrexate and give Antibiotic
B- Hold Adalimumab and give Antibiotic
C- Hold current therapy and give Antibiotic
D- Hold current therapy without Antibiotic

Female pt with malaise and headache, 2 days after she developed skin rash. She was in jeddah 10 days
ago, what is the management?
Labs show : Leukopenia , Thrombocytopenia
A- anitbiotics
B- antiviral
C- supportive

40s old female, with heavy bleeding came to ER, what to do?
Anonymous Poll
A- IUD
B- D/C
C- Hysterectomy
D- Mefenamic acid

Breast feeding mother have mastitis and received ABx ,then developed discharge, doctor suspect Breast
abscess ,Most appropriate step ?
A- Change Antibiotic
B- Encourage breast feeding
C- Avoid breast feeding
D -U.S with something .

no I&D or needl aspiration in the options

https://t.me/smlemay/8122
SMLE - B, [Oct 22, 2021 at 10:01 PM]
A patient complaining of burning chest pain and unpleasant taste when lifting heavy objects?
A. Gastritis.
B. Esophagitis.
C. Perforated peptic ulcer.

Same question but ask about Management :


A- refer to antireflux surgery
B- PPI

As far as I remember theres no Endoscopy in options


24 years old male complain of Dysphagia to liquid more than solid , what investigation will help you to
reach the diagnosis ?
A- Manometry
B- 24 Ph
C-Barium swallow
D-Endoscopy

Dysphagia to solid Components, most appropriate step?


A-Endoscopy
B-Manometry
C-24 Ph monitoring
D-Barium

S/S of small Bowel obstruction in paediatric initial investigation?


A- U.S
B-CT
C-Abdominal X-Ray
D-Barium

Patient with Schizophrenia on medication come to ER with distention and other signs .
Imaging show hugely distend large bowel megacolon “Ogilvie's syndrome” , vitally stable no peritonitis ,
as far as I remember no indication for surgery
Appropriate management?
A- colectomy with colostomy
B- rectal Decompression

Both LL burn wt is 70 calculate fluid


A 5L in the first 6 bourse and 5L in the next 16 hours
B 2.5 L in the first 6 hours and 2.5 in the next 16 hours
C. Normal saline 1L frist 24h than 200(maybe)mL/h
D. Mantainence fluid

Milestone, baby pulls himself to stand, crawls without difficulty, which age is he?
A. 8 months
B. 10 months
C. 12 months
D. 14 months

Milestones
Baby rolls and sits unsoppurted but cannot pincer grab
A. 4 months
B. 6 months
C. 9 months

Patient with lower GI bleeding, failed to identify source of bleeding after colonoscopy, NGT inserted and
turned with greenish material or fluid what would you do next ?

- upper GI endoscopy
- Technetium
1500 gram neonate with negative HBV mother, when to vaccine :
-at birth
-1 month based on choronological age
-2 months based on corrected age
-within 1 week of hospital discharge

If there is at discharge its the answer

Milestone:
Tripod position, can flip supine to prone , immature grasp
4 month
6 month

A child brought the clinic complaining of chronic proteinuria renal disease and hypertension. The parents
are concerned about progression to end stage renal disease. Which of the following drug is useful in
preventing the progression:
-Enalpril
-Clonidine
-Propanolol
-Amlodipine

22 years old male mediclly free had unprotected sex presented with dysuria and urethral discharge
cluture showed chlamydia trachomatis
Abx:
Gentamycin
Azethromycin
Ceftrixone
Nitro

8 yo boy complaing of feeling palptations and chest pain no past medical hx and -ve family history -ve
medication history normal resp and cardio exam with no added sounds
Vitals
Hr:265
Bp:95/76
Rr:normal
Temp:norma
Most appropriate next step :
12 lead ecg
Chest x ray
Cardic enzymes
Echo

65 elderly required intubation to trafer into icu collapsed in ward 2 days later did surgery for empyema
Vitals
Bb110/70
Hr 110
Rr 23
Temp 37.8
Dx
-sirs
-bactermia
-spesis
- severe sepsis

40 yo female presented with 2x2 firm breast lump for 2 years in the inner ruq of the breast no skin
changes normal axilla
Biopsy : cystosarcoma phlloyde
Mx:
Chemo
Radiation
Simple mastectomy
Wide local excsion

23 yo female pregnant 28wks with fetal hydrops peak systolic flow 1.8mom on mca doppler cause ?
-anuplide
- diaphragmatic hernia
- anti-kell antibody
- cervical teratoma

26 yo female menarche at 15 lmp 9 month ago


Labs:
Fsh:50
Lh:37
Bhcg2
Diagnosis:
-pcos
- premature ovarian insufficiency
-ovarian tumor
- asherman syndrome

Female pregnant 32 wks with veginal bleeding700cc 7cm dilated no contractions


- intrapartum
-antepartum

Patient presented with abdominal distention yellow sclera vague epigastrc abdominal pain paracentcies
was done showed
Serum albumin 30
Acetic albumin 14
Acetic please protein 10
Nephrotic
Congestive heart failure
Acute pancreatitis
Liver cirrhosis
Patient with hx of rhemutaic heart dissease presented with sob and pnd and orthopnea basal crackels
holosystolic murmur improved on carvidlol ,linsopril, fursumaide , spriolictone
Echo shwoed :
Sever mitral regurgitation lv hypertrophy ef 45%
Follow up 6 month
Mitral valve replacment
Arb

Pt with left chest painful eruption, papules, vesicles and crusted what is the most appropriate test?
A Biopsy
B Blood culture
C Swab and culture
D No further investigation

56 year old male with long standing heartburn, did endoscopy and showed barrette esophagus with low
dysplasia, what is the initial
A Esophageal resection
B Bantoprazole
C Renitadine
D Sucralfate

What indicate cholesterol embolism (no hyaline cast in options)


A WBC cast
B RBC cast
C Granular cast
D Eosinophiluria

Treatment of uncomplicated cystitis for 5 year old male


A Amoxicillin
B Ciprofluxacin
C No need for ttt

Ciprofloxacin usually not given for less than 18y

Lactating mother, her breast is red and swollen


Ask about the organism:
1- Staph aureus
2- Hemophlus influnza

Mother came complaining that her child who's 8 y.o boy has an attention defect and always plays and
loses his toys, after talking to him there's no abnormality in his history or a visible defect in his attention,
what do you do??
A. Give him amphetamine(forgot dose)
B. Reassure the mother that it's normal in his age
C. Ask futher questions from parents and teachers

399-Child had Type1 diabetes and was consulting the family regarding celiac screening, which of the
following is true:
A- Screen at diagnosis then every 5 years
B- Screen at diagnosis then every 2 years if asymptotic
C- Screen annually

smoking cessation for pt with chest pain on exertion and relived by rest on examination he’s anxious with
nicotine staining on fingers
A- buspirone
B- varenicline

Anticoagulation in newly diagnosed pt with Afib DM HTN no contraindication to any anticoagulant


Warfarin
Clopidogril
Aspirin
Clopidogril and aspirin

11 months old infant ,Crying and irrtibale dark foul smell stool mixed with blood and mucus , what will u do
investi-gation :
• A. technetium scan .
• B. Ultrasound
• C-Stool analysis
• D-Stool clutire

Q: Elderly on amitryptiline (30mg) at night for insomnia C/o drowsiness, management?


A. Switch to SSRI
B. Change to taking at morning
C. Change to taking TID
D. Decrease to 10mg

Which among the following is NOT a side effect of bupropion?


A. Diarrhoea
B. Blurred vision
C. incresed sweating
D. dry mouth

A 33yo drug addict wants to quit. She says she is ready to stop the drug abuse. She is supported
by her friends and family. What drug tx would you give her?
a. Benzodiazepines
b. Diazipoxide
c. Lithium
d. Methadone
e. Disulfiram

B, E ‫ممكن يساعدوا في تخفيف االعراض االنسحابية خاصة لو كحول‬


‫ و‬D ‫ لو‬opioids
A ‫ بشكل عام يساعد على تخفيف اعراض ال‬irritability & anxiety with withdrawal symptoms
And as she is having good family support to control other symptoms I think Benzodiazepines is the best
answer by exclusion.
C is used for controlling mood swing but not the first choice.

‫لو في صيغة ثانية للسؤال ممكن يكون اوضح الجواب‬

I will go with A by exclusion

According to psych-smle group

Patient known case of Alzheimer's came with severe agitation. What is the treatment
A-Olanzapine
B-Haloperidol
C-Buspurone
D-Thioridazine

Olanzapine and haloperidol can treat agitation however we start with atypical antipsychotics (Olanzapine,
risperdone , quetapine) because of less side effects, if not improving then we choose typical
antipsychotics ( haloperidol)

According to psych-smle group

Pt I think he known case of Alzheimer came to ER with severe agitation the doctor give him a big dose of
Haloperidol and he develops side effects I don’t remember what r they exactly, what u should give him
now? They mean what is the antidote of haloperidol?
A-Naloxone
B-bromocriptine
C-Glycogen

Nicotine Effects?
A- lung cancer
B- Addiction
C- HTN

Post partum depression What to do beside psychotherapy:


A-Small dose of antidepressants
B-Mother breast feed the baby
C- involve family in therapy

Answer C. although B is also correct to increase the bond of mother and her baby yet involving the family
is more important for support at this stage.

According to psych-smle group

Patient came to the clinics , he speaks rapidly and jumping between topics , which of the following


describing his condition?
A- Flight of Ideas
B- loss of associations ❓
C- can not remember
D- Forget it

Pt came to the clinic, he is talkative jumping from topic to topic without completing each one. What is this
called?
A. Flight of ideas
B. Thought insertion
C. Broadcasting
D. Thought withdrawal

‎‫ هذا شي مهم في ال‬mental status examination



‫وصف‬ ‫ قاعدين نتكلم عن‬thought process
Jumping between ideas, rapid speech but coherent described flight of ideas

Rapid speech frequently changing the topic but incoherent described as loosening of association


‫السؤالين‬ ‫ فالجواب في‬flight of ideas ‫دام ما حدد هل الكالم مفهوم او ال‬

According to psych-smle group

Poor prognosis of schezophrenoia:


a. Onset in Adolecant years
b. Family history
c. Acute
d. Anxiety with flares

According to psych-smle group

According to psych-smle group


Bipolar disease feature in hypomania episode with weight gain & previous mania episode what to give :
A. Lithium
B. Olanzapine
C. Mitrazapine

According to psych-smle group

15 year-old, his friend died in a car accident and he told his mom he's thinking of
suicide but not going to act on it, what to do?
A⁃ Asking him directly about his suicidality is gonna increase the risk of him acting on it
B⁃ Ask him in details about his suicidal thoughts and feelings
C- reassurance and never to mention suicide thoughts to patient

Suicide thoughts shouldn’t not be ignored in any Hx.


According to psych-smle group

23 years old, he came complainig of the first time hearing sounds (auditory hallucinations) What is your
management
a. Olanzapine
b. Flouxitine
c. Antipsychotic & CBT
d. CBT

Brief psychotic episode : Antipsychotics important to balance the dopamine irregularity caused
hallucination and CBT is a technical way to improve quality of life as it can be traumatic to some patients

According to psych-smle group

COPD presented to the ER severe respiratory distress and hypoxia, not improved by bronchodilator, best
next?
A- corticosteroid
B- bipap
C-?
D-?

Patient with s/s of trichomonos and receive treatment, what about her husband?
A- no treatment
B- metronidazole
C- follow up after 2 weeks
D- other medication

Prosthetic valve prophylactic: ICQ


1- clindamycin
2- Ampicillin
‫جاء دواء دواء مو مجموعة كاملة‬

10 years old boy had tonsillectomy and after 5 days he came with bleeding
What’s the cause
- sepsis
- incomplete removal
- coagulopathy
- foreign body

Prophylaxis for militarian who works in southern saudi arabia


-Atovaquone
-Mefloquine

Pt pedi 12 y c/c pain epi with burning sensation


Not related to Anything mother have also non specific clinical Abd pain
Lab - low plt
Anaemia
What Mx ?
Omeprazole
Azathro
Tetra cycle
Hypo Allergeic diet

40 years old female, multiparous, her last delivery was 2 years ago and it was complicated and used
forceps
complaining of mass bulging from the vagina with long standing or walking and when coughing
what is the first thing to do in the examination?
a) Bonney test
b) Speculum examination (pelvic organ prolapse)
c) Cystourethrogram
d) urodynamic

A 45 - year old patient complains of perianal swelling, fresh bleeding per rectum and weight loss over the
last 3 months on examination, there is a mass 1 cm from the anal verge. She has no obstructive
symptoms (see report), Blood pressure Heart rate Respiratory rate Temperature Biopsy:
Adenocarcinoma.
MRI abdomen: Localized lesion with craniocaudal extension of 3 cm with associated lymphadenopathy.
110/70 mmHg 96 / min 18 / min 36.6 ° C CT scan chest: No evidence of metastasis. Which of the
following is the most appropriate treatment?
A. Diversion colostomy
B. Low anterior resection
C. Concurrent chemoradiation
D. Abdominoperineal resection

61 years old man complain of indigestion for 2 months, he never had similar attach in his life and he eats
and drinks well. He says he had weight loss over this period. There is no change on bowel habitats and
no abdominal pains, abdominal examination is normal and blood tests including CBC, RFT, LFT,
C-reactive protein are normal..
A) urgent endoscopy
B) routine abdominal US
C) abdominal and erect x ray today
D) referral to Gastroenterology

55 yrs male complains of productive cough for 3 months he has similar attack of cough for the previous 2
years,
A) bronchiectasis
B) bronchial asthma
C) pulmonary fibrosis
D) chronic bronchitis

38 years old women complains of fever and headache for 2 days. On examination, there is scattered
petechiae all over her body.
PBS showing the above photo
Fever:- 38.6
Plt: 32
LDH: 690
High indirect and total bilirubin.
Reticulcyts: 5.6 (high)

A) platelet exchange
B) intravenous Imunnoglobulins and prednisolone
C) arbgarban ((( I forgot the name ))
D)

*elderly 70 yrs old post esophageal dilatation came back to ED 8 hrs with severe chest pain Possible
underlying cause ?
1/ MI
2/Perforation
3/ Bleeding

*8 weeks with s/s of RHF and fever his last PE showed pansystolic murmur
He was febrile with tachycardia and dyspnea; he was previously fine 2 weeks back what is next best
management ?
1/ Refer for surgical intervention
2/ give medical diuretic

*CD pt with abdominal pain and diarrhea for days now experiencing generalized muscle weakness ECG
shows flattering of T wave
What are possible electrolyte imbalance ?
1/ hypomaganesium
2/ hyponatremia
3/Hypokalemia
4/Hypocalcemia

*RTA with collar for neck his airway is patent and secured now what is next ?
Vital sign show hypotension
1/ CT scan
2/vascular access

28. Patient has dysphagia to only liquid what is the highest yield diagnostic method?
A. manometry
B. upper endoscopy

Treatment of pt with ovarine tumor and ascites


- surgery
- surgery chemotherapy
- surgery radiotherapy

Thyroid goiter with hypothyroidism lab show high TSH , elevated ESR , Anti-TPO antibody was Less than
notrmal value ( < 9 IU/mL )
A-Subacute thyroiditis
B-Hashimotos

Female in 20’s with multiple fibroids , best way to preserve her fertility?
A- Myomectomy
B- OCP
C-progesterone

Preganat 38 weeks with typical preeclampsia with HELP syndrome Prepphral blood smear show halment
cells
- Plasma exchange
- urgent delivery

Patient with TTp


Management
Plasma exchange

Yung female complain of nausea and vomiting when she is on flight plane for more than one hour
What is the tx :
- ginger
-odenosertan something like that
-gliposertan somthing like it

Previous recall:
Female pt complain of vomiting after 1 hour of flight what's the best drug to use:
A. Ginger
B. Ondasteron
C. Phenylephrine
Answer: Scopolamine or antihistamine (diphenhydramine or meclizine)
Diphenhydramine came in the exam so it's the right answer

Which of the following indicate uterine bleeding


A. Perfused bleeding
B. Postcoital bleeding

Symptoms of candidiasis infection and then they asked -> this condition comes with ?
A. DM
B- not remember

Child with repetitive eye movement, all other things was normal,Dx?
1- Tic disorder
2- Tourette syndrome

37 year old female primigravida at 34 GA, know to have multiple fibroids, presented now with abdominal
pain, on examination: cervix is closed and no evidence of contractions.
What is the most appropriate management?
A- Observation
B- Myomectomy
C- CS
D- Induction of labor

Cause of death in patient in dialysis?


Sepsis
Ischamic heart disease
Hypekalemia

Female 28 years G: 7 P:6, at 38 GA went into labor for 7 hours then delivered a 3000 g baby, after few
hours she bled profusely, which of the following is the most likely cause of her bleeding?
A- Grand multiparity
B- Fetal weight
C- Prolonged labor
D- Precipitous labor

Sudden abdominal pain after eating, severe score 10/10, tense abdomen with high lactate?
- laparotomy
- colonoscopy
- conservative

Trauma Retripertonial bladder injury ?


- Foley catheter
- laparoscopy
- repair (only like this)
-cystoscope

Chest pain, sweating, stable ecg old MI already on medications, labs pending ?
- thrombolysis
- wait for labs result
- pci

https://t.me/smlemay/8208
SMLE - B, [Oct 24, 2021 at 8:49 PM]
• child presented to you with history of coin ingestion 6 hours ago, child is stable, on X-ray, its in the
stomach, Mx?
• A-Ultrasound
• B Immediate Endoscopy
• C Wait till it passes by
• D-CT abdomen

794-A child swallowed a coin. And on imaging it was on stomach. What to do (I had the same scenario
but with drooling and fatigue)
A) observe
B) remove by endoscopy
C) go home and wait till it comes out in stool
D) don’t remember

Child coming with his family from Africa with signs and symptoms of neck stiffness, fever, his CSF
findings are showing features of viral meningitis. Which of the following is the most likely causative virus?
1. Ebola
2. Ebstein-barr virus
3. CMV
4. Forgot the last virus

18 month old infant suddenly developed knee swelling severe enough that made him not moving it, on
examination he was resisting passive flexion. What is the single most reliable test for diagnosis?
1. Knee x-ray
2. Joint aspiration
3. Blood culture

Case girl with 2 week hx of abdominal pain after falling from her bicycle on examination there was bruises
on her abdomen what to do next ? All the choices were imagining
A. CT
B. ABDOMINAL US
C. MRI

Child present to the hospital with Hx of conjunctivitis fever then maculopapular rash on the face then
spreed to the rest of the body
- EBV
- vaccinia
- rubella
- coxsackie

Long senario of an MVA patient presenting with neck pain, weakness in the lower limbs, fracture in the
right humerus, ulna, and radius, vitals are unstable (I remember hypotension), asking about the most
likely underlying manifestation:
1. Hypovolemia
2. Neurogenic shock
3. Fat embolism

Burn case black soot over nostrils and mouth 40% carboxyhemoglobin.
(Carbon monoxide toxicity):
A. Hyperbaric oxygen
B. Intubation and ventilation with room air.
C. Intubation and ventilation with 100% o2.
D. Carbonic anhydrase inhibitors.

* Best bariatic for obesse with GERD?


- Gastric ring
- Sleeve
- Ballon
- Gasrtric bypass

Patient no hypotension and lactet level 3.0 :


A- Sepsis
B- Septicaemia
C- Septic shock
D-SIRS
Case also says he is on vasopressor to maintain BP

Patient came with new onset of LBBB ( given dx ) was given aspirin, heparin
what to the most appropriate next step ?
⁃ Give carvidolol
⁃ Give warfarin
⁃ Do pci
⁃ Do echo

Neonate came with cyanosis and machinery murmur what you do?
A. Oxygen mask
B. NSAIDs
C. iv fluids
Prostaglandin

7 y boy came with his mother becouse he wet his bed every night for duration of time he was not like this
before and mother telling his sister was nt like this in same age Urine analysis showed positive nitrate
What is the diagnosis

A nocturnal enuresis
B uti

Ederly man presented to the ER with rigid distended abdomen prepared for laparotomy with hypotension
and fever (38)
X ray showed free air under diaphragm
What is the best initial resuscitation option ?
- intubation and ionotropes
-Broad spectrum Abx
-Colloid via central line
-Crystalloid fluid via peripheral line

CMale patient with recurrent hematemesis, no other complain, Vitaly stable, labs show anemia, LFT
normal.
( not mention any other symptoms)
Whats your diagnosis :

👍
- esophageal varicess
- peptic ulcer disease
- erosive gastritis
- mallory weiss tear

https://t.me/smlemay/8223
SMLE - B, [Oct 25, 2021 at 4:33 PM]

👍
Pt come with sign of DVT , what is the initial and most appropriate next step ?
A-dopler us
B-spiral Ct
https://t.me/smlemay/8224
SMLE - B, [Oct 25, 2021 at 4:35 PM]
Long scenario (Pt 45 y male come for screening colon cancer.. who screening 50 .. ‫اتوقع ان قريب له عمل بعمر‬

👍
a/ At 50 years and all 5years
b/ at 50 years and all 10 years
c/ now

https://t.me/smlemay/8225
SMLE - B, [Oct 25, 2021 at 4:35 PM]
- Asthma patient came to er due to exacerbation, but patient is vitally stable

Most appropriate treatment:

👍
1. Iv steroids
2. Inhaled b blockers
3. Montelukast

https://t.me/smlemay/8226
SMLE - B, [Oct 25, 2021 at 4:35 PM]
Pregnant with genitalia chlamydia infection

👍
The baby will come with infection in
-eye
Lung
Liver
-…

https://t.me/smlemay/8227
SMLE - B, [Oct 25, 2021 at 5:50 PM]
irregular cycle + absent period for 6 weeks With pregnancy test positive since 2 week only before it was
negative How to determine the pregnancy age
A- Quantitative hcg

👍
B- Calculate from date of pregnancy test
C- Wait 2-4 WEEKs to do ultrasound
D- US know

https://t.me/smlemay/8228
SMLE - B, [Oct 25, 2021 at 5:52 PM]
Pt old heavy alcohol drinker came with epigastric pain with tenderness
He have multi smilar attack ( normal amylase )

👍
-biliary colic
- chronic pancreatitis

https://t.me/smlemay/8229
SMLE - B, [Oct 25, 2021 at 5:52 PM]
28 year old lady diagnosed with ectopic
Ectopic pregnancy most likely due to
A. Ovulation induction
B. PID

👍
C. IUD
D. Previous hx of ectopic

https://t.me/smlemay/8230
SMLE - B, [Oct 25, 2021 at 5:52 PM]
Trauma to the chest
Normal equal air entry lung
Distended JVP
Hypotensive
O2 sat 85%

A. Lung contusion
B. Flail

👍
C. Pneumothorax
D. Temponade

https://t.me/smlemay/8231
SMLE - B, [Oct 25, 2021 at 5:53 PM]

👍
34 weeks? Vaginal bleeding denies abdominal pain no contraction
A. US
B. Digital pelvic exam
C. Admit to ward?

https://t.me/smlemay/8232
SMLE - B, [Oct 25, 2021 at 5:53 PM]
Pt with recurrent episodes small bowel obstruction managed conservatively now presenting with another
episode for 4 days no improvement
BP: 92/65

👍
Whats your management?
A. Ex lap
B. Managed conservatively with IV NPO NGT

https://t.me/smlemay/8233
SMLE - B, [Oct 25, 2021 at 5:54 PM]

👍
US picture of twins and say it's monochorionic diamniotic ,cleavage at which day ?
4-8

https://t.me/smlemay/8234
SMLE - B, [Oct 25, 2021 at 5:54 PM]
Old patient with hypertension and DM presenting now with DVT Heparine was started
Lab shows hperkalemia ,which of the following should be stopped at this stage ?
Insulin

👍
Heparine
Lisinipril

https://t.me/smlemay/8235
SMLE - B, [Oct 25, 2021 at 5:54 PM]
19 Y.O girl newly diagnosed with type 1 DM , taking aspart and glargine , complaining of both fasting and
postPrandial hypoglycemia , A1c and fasting are with in normal range ?

👍
A- Stop both
B- Decrease the dose of both
C- Continue the same
D- Stop glargine and change aspart to sliding scale

Percentage of saudi male Smokers


30% , overall 20-25%

Percentage of GERD after sleeve


30%

https://t.me/smlemay/8238
SMLE - B, [Oct 25, 2021 at 5:54 PM]
Patient with asymptomatic cirrhosis due to eradicated HCV infection

👍
Best option for follow up ?
US EVERY 6 MONTHS
AFP
MRI

https://t.me/smlemay/8239
SMLE - B, [Oct 25, 2021 at 5:54 PM]
Women in her 50’s with hard non tender immobile lump with tethering and red skin, dx?
Fibroadenoma

👍
Duct ectasia
Carcinoma of breast
Forgot the last obstion.

https://t.me/smlemay/8240
SMLE - B, [Oct 25, 2021 at 5:55 PM]
Pt with productive cough , white sputum then more yellow and bilateral lung infiltration more dense in
right lung ( i think in middle lobe ) what is the cause :
A- atypical pneumonia
B- streptococcus pneumonia
*no pneumococcal pneumonia in choice

https://t.me/smlemay/8241
SMLE - B, [Oct 25, 2021 at 5:55 PM]
Pt with sign of thalassemia ( lab show : A2 and HgF ) what's dx :

👍
A)thalassemia type a
B)thalassemia type b
C) SCD

https://t.me/smlemay/8244
SMLE - B, [Oct 25, 2021 at 5:55 PM]
Long scenario ( pt with tympanostomy tube in left ear, then now perulent discharge and write ( he likes
swimming ) ask what causative organism :
Streptococcus pneumonia

👍
Staph
pseudomonas aeruginosa

https://t.me/smlemay/8246
SMLE - B, [Oct 25, 2021 at 5:56 PM]

👍
Patient with RA on steroid , hydroxchoroquine , MTX no improve his articulatr symptoms what to give?
adalimumab

https://t.me/smlemay/8247
SMLE - B, [Oct 25, 2021 at 5:58 PM]
Site of needle syringe to aspirate pleural effusion:
- midclav. 6th
-mid clav. 9th

👍
- midaxi. 6th
-mid axi 9th

https://t.me/smlemay/8248
SMLE - B, [Oct 25, 2021 at 5:58 PM]
Questions From yesterday :
- child with hx symptoms of DM(polyueara, polydypsia, polyphagia ) upon waiting in the clinic , he never
eat or drink for 3 hours and there sings of dehydration labs: high blood glucose 21mmol what you
suspect this patient has?

👍
1. Increase insulin level
2. Decrease insulin level
3. Increase glucagon
4. Decrease in lipid

https://t.me/smlemay/8250
SMLE - B, [Oct 25, 2021 at 5:58 PM]
- female came to infertility for 2 years , her menerache started at age of 13 she is menstruated regularly
every 28 days and each period lasts for 5 days , medically and surgically free, her husband Sperm
analysis is normal , her Basal body temperature is Monophasic, what the cause of the infertility?
1. PID
2. Coitus something
3. Vaginal something
Anovulatory is the best if its there , if not then A

- child can use tricycle > 3 year milestone

https://t.me/smlemay/8253
SMLE - B, [Oct 25, 2021 at 5:58 PM

- female patient had irregular cycle and using something as a conception method (not IUD)
Her last Menstrual cycle 6weeks ago
Came to with vaginal red-brown stained discharge what to you will do next will confirm the Dx?

👍
1. Huhner’s test
2. BHCG levels
https://t.me/smlemay/8254
SMLE - B, [Oct 25, 2021 at 5:59 PM]
which valvular heart disease has the worst outcome in pregnancy?

👍
A)Mitral regurgitation
B)Mitral stenosis
C)Aortic stenosis
D)Aortic Regurgitation

https://t.me/smlemay/8256
SMLE - B, [Oct 25, 2021 at 5:59 PM]
30 week IFD + placental abruption currently having DIC, 6 cm dilated cervix. Most appropriate
management?
A)observe
B) c/section
C) oxytocin
D) hysterectomy

Fetus with breach presentation flexing hip and knees and elbows and spine is parallel to mothers spine.
What’s the lie?

A- Frank breech.
B- Complete breech

👍
C- Incomplete breech
D- longitudinal
Answer is d

https://t.me/smlemay/8264
SMLE - B, [Oct 25, 2021 at 6:02 PM]
Child with frontal bossing , spleenomegaly
HB F 30
HB A 50

👍
Diagnosis?
B thalasemia
Alpha thalacemia
Sickle cell anemia

https://t.me/smlemay/8266
SMLE - B, [Oct 25, 2021 at 6:03 PM]
Surgeon presenting a new surgical method
And he changed some facts to decrease error resulting from these methods and to convince the audience
What did he do?

👍
plagiarism
Falsification
Fabrication
Forgot the 4th

https://t.me/smlemay/8267
SMLE - B, [Oct 25, 2021 at 6:03 PM]
👍
29 with rheumatic fever she is for dental procedure which anti biotic to give?
Ampicillin
Doxycycline
Forgot the rest

https://t.me/smlemay/8268
SMLE - B, [Oct 25, 2021 at 6:03 PM]
A wife beaten aggressively by her husband comes to your clinic , what to do?
Talk to the husband
Reassurance
Inform ethical comity
Forgot

It should be inform authorities

https://t.me/smlemay/8269
SMLE - B, [Oct 25, 2021 at 6:03 PM]
Pt confirmed of ebola is saying either discharge me or i will leave on my own, what to do?

👍
Let him sign DAMA
Call the security
Call infection control
Inform the ethical comity

https://t.me/smlemay/8270
SMLE - B, [Oct 25, 2021 at 6:03 PM]
Child with asthma come with exacerbation (not sure if there was anything about infection or fever) the

👍
father is smoker and been advice to not smoke around the child, the father is ignoring the dr , what to do?
Call for child abuse
Ignore the father and focus on the child
Talk to the father about importance of immunization
Forgot the 4th

https://t.me/smlemay/8271
SMLE - B, [Oct 25, 2021 at 6:03 PM]
Medical intern in OR and she think is the left kidney not right , what to do?
Tell the nurse to check the system
Tell one of the resident

👍
Go out and check the system yourself
Tell the surgeon about your concern

https://t.me/smlemay/8272
SMLE - B, [Oct 25, 2021 at 6:03 PM]
Pt for appendectomy during surgery no inflammation found, se per protocol remove anyway , the dr fogot

👍
if he told the pt about this before surgey, what to do now?
Tell the pt about the protocol
Don’t tell the pt at all
Ask another dr to tell the pt
Forgot the 4th
https://t.me/smlemay/8273
SMLE - B, [Oct 25, 2021 at 6:04 PM]
Pt, post menopause, asking for hormonal therapy replacement, dr tell her there is not proof it will benefit
her, but she insist , what to do?
Give her the hormonal therapy
Transfer her to another dr

👍
Call the ethical comity
don’t give it just let her leave

https://t.me/smlemay/8274
SMLE - B, [Oct 25, 2021 at 6:04 PM]

👍
Pt with peanut aspiration , where could the peanut be?
Right bronchus
Left bronchus
Forgot the rest

https://t.me/smlemay/8277
SMLE - B, [Oct 25, 2021 at 6:04 PM]
Pt with marfan syndrome features. Whats the most important investigation to r/o life threatening
conditions
A. Echo
B. Chest CT
C. ECG

https://t.me/smlemay/8278
SMLE - B, [Oct 25, 2021 at 6:12 PM]
50 YO plumber came with exertional dyspnea and cough. CXR showed bilateral infiltrates
372-Which of the following has the highest diagnostic value ?
A- CXR

👍
B- HRCT
C-transbronchial biopsy

https://t.me/smlemay/8279
SMLE - B, [Oct 25, 2021 at 6:12 PM]
AAA unstable
A. US
B. CT
C. Ex lap

Depend if dx before then do surgery , if not then us

https://t.me/smlemay/8282
SMLE - B, [Oct 25, 2021 at 6:12 PM]
25 oct:
20 years old females, come with sever RLF pain, tenderness in ex, without fever:
Ovarian torsion
Appendicitis
Repture ectopic pregnancy
Not sure but i will go with A

https://t.me/smlemay/8283
SMLE - B, [Oct 25, 2021 at 6:13 PM]
- 55 year old male patient came to the ER complaining of LLQ abdominal pain, associated with
constipation for 3 days

Vitally all stable except for temp it was 38.5

Asking about inital management:

1. Abx 👍
2. Colonoscopy
3. Laparatomy
4. Reassure and send home

https://t.me/smlemay/8284
SMLE - B, [Oct 25, 2021 at 6:13 PM]
- Old patient with altered lvl of consiousness fell from the stairs, was brought to ER, done CT for him
showing epidural hematoma

Asking about most likely affected artery:

1. Pontine
2. Basilar

👍
3. Anterior cerebral artery
4. Middle meningeal artery

https://t.me/smlemay/8285
SMLE - B, [Oct 25, 2021 at 6:13 PM]
- RTA patient went to the ER, he was stable so they did CT for him, just after they finished the CT report
he immediately detoriorated (BP became 90/60)

CT report showed: thoracic artery was injured/ruptured or something like that + there were fluid
collections around the spleen

asking about most appropriate management:

1. Explaratory laporatomy 👍
2. Emergency thoracotomy
3. Refer to hospital with vascular surgeon

https://t.me/smlemay/8287
SMLE - B, [Oct 25, 2021 at 6:13 PM]
- Patient with uncontrolled heart failure symptoms and slight tachycardia (i think 110)

Echo showed ventricular hypertrophy

Already on ace inhibitors and furesomide, what to add next


1. B blockers 👍
2. Calcium channel blockers

3. Thiazide diuretic

Q about research and at the end What is the Most appropriate way to review the proposal? -
review the proposal by Your colleague

👍
let your colleague to review the research result
There should be ethical approval before the research

https://t.me/smlemay/8293
SMLE - B, [Oct 25, 2021 at 6:19 PM]
7th days after his 4th chemo session developed fever and came to the ER
Labs: low WBC but PMN (75%)
A. NSAID and antifungal

👍
B. Oral abx and discharge
C. IV antibiotics in hospital settings
D. Await work up

https://t.me/smlemay/8294
SMLE - B, [Oct 25, 2021 at 6:28 PM]
Patient with jaundice splenomegaly and history of cholecystectomy for stone. Required many blood
transfusions before.
Labs: microcytic anemia
Most diagnostic test

👍
A. Peripheral blood smear
B. Hemoglobin electrophoresis
C. Bone marrow biopsy

The question came in september they told u parents are relative and mchc is low -> its sickle or
thalassemia so B

https://t.me/smlemay/8295
SMLE - B, [Oct 25, 2021 at 6:28 PM]
Child came from village, short stature, low sat ear, dysmorphic face

👍
What’s the digenesis?
- Noonan syndrome
- Soso syndrome
- marfan syndrome

https://t.me/smlemay/8296
SMLE - B, [Oct 25, 2021 at 8:12 PM]
Patient who have abdominal bloating diarrhea … symptoms worse when he eat bread ( clear easy celiac
case )

Which skins manifestation you expected :


- Psoriasis
- herpetiformis dermatitis
-
-

https://t.me/smlemay/8297
SMLE - B, [Oct 25, 2021 at 8:12 PM]
180-A lady is pregnant with twins. What is the most common risk factor associated with increased
mortality in multiple gestation?
A. Birth defects
B. Birth trauma
C. Prematurity

8 days old newborn for diabetic mother came with SOB and cyanosis, looks irritable and ruddy, no flu like
symptoms
RR 40-49
HR 169
Hb 24.1 (normal: 165-175)
Hct 0.75
Plt 120
What is the most appropriate next management?
A. Echocardiogram
B. Partial exchange transfusion
C. Hydration, oxygenation, suction

8 days old newborn for diabetic mother came with SOB and cyanosis, looks irritable and ruddy, no flu like
symptoms
RR 40-49
HR 169
Hb 24.1 (normal: 165-175)
Hct 0.75
Plt 120
Whats the diagnosis?
A. Polycythemia
B. RDS

https://t.me/smlemay/8301
SMLE - B, [Oct 25, 2021 at 8:13 PM]
I think RDS case 3 hours old with cyanosis grunting asking about management
A. Steroids
B. Intratracheal surfactant

Murmur increase with hand grip and decrease by valsalva manuver


MR
MS
AR
AS

Both AR and MR
https://t.me/smlemay/8303
SMLE - B, [Oct 25, 2021 at 8:13 PM]
Signs of hopelessness
1- suicidal thoughts

I forgot the other option but this what I chose

Hernia lat and inf fo pubic tubericle


Femoral
Direct ingunal
Indirect inguinl

https://t.me/smlemay/8307
SMLE - B, [Oct 25, 2021 at 8:21 PM]
Empiric treatment of pneumonia
Azithromycin and ceftriaxone

https://t.me/smlemay/8308
SMLE - B, [Oct 25, 2021 at 8:21 PM]
Chlymedia STD tx
Azithromycin

‫فيه سؤال جاني مدري جديد او ال بحثت عن بعض الخيارات الموجودة بس مالقيت‬
‫ياليت احد يتذكره‬
‫ فيه مريض سوا‬femoral artery or inguinal ‫ ما اذكر لألسف بعدها بكم يوم صار عنده‬swelling
And pain + numbness
‫اللي اذكره الخيارات كانت‬
Posterior slap
fasciotomy

https://t.me/smlemay/8311
SMLE - B, [Oct 25, 2021 at 9:06 PM]
20 yrs old, has persistent moderate asthma for 6 months, on SABA LABA ICS, he has rhonchi ,
symptoms are especially present at basketball in college
Next best step
-Add leuktrine
-Modify ICS to medium dose
-refer for otolaryngology evaluation

https://t.me/smlemay/8313
SMLE - B, [Oct 25, 2021 at 9:09 PM]
Acute limb ischemia in previous chronic leg pain in smoker what is most appropriate inv:
-MRI angio
-CT angio
-conventional angio
-US
-COPD presented to the ER severe respiratory distress and hypoxia, not improved by bronchodilator, best
next?
A- corticosteroid
B- bipap

U/s pic of endometrial mass presenting with heavy menstrual bleeding


A)endometrial polyp
B)submucosal fibroid
Answer: both can present with heavy bleeding, you need to see and differentiate between the images

You might also like